Slot System
Featured Buckets
Featured Buckets Admin

Prescribing statins for patients with ACS? No need to wait

Article Type
Changed
Mon, 07/13/2020 - 13:33
Display Headline
Prescribing statins for patients with ACS? No need to wait
PRACTICE CHANGER

Prescribe a high-dose statin before any patient with acute coronary syndrome (ACS) undergoes percutaneous coronary intervention (PCI); it may be reasonable to extend this to patients being evaluated for ACS.1

Strength of recommendation

A: Based on a meta-analysis

Navarese EP, Kowalewski M, Andreotti F, et al. Meta-analysis of time-related benefits of statin therapy in patients with acute coronary syndrome undergoing percutaneous coronary intervention. Am J Cardiol. 2014;113:1753-1764.

 

Illustrative case

A 48-year-old man comes to the emergency department with chest pain and is diagnosed with ACS. He is scheduled to have PCI within the next 24 hours. When should you start him on a statin?

Statins are the mainstay pharmaceutical treatment for hyperlipidemia, and are used for primary and secondary prevention of coronary artery disease and stroke.2,3 Well-known for their cholesterol-lowering effect, they also have benefits that are independent of their effects on lipids, including improving endothelial function, decreasing oxidative stress, and decreasing vascular inflammation.4-6

Compared to patients with stable angina, patients with ACS experience markedly higher rates of coronary events, especially immediately before and after PCI and during the subsequent 30 days.1 American College of Cardiology/American Heart Association (ACC/AHA) guidelines for the management of non-ST elevation myocardial infarction (NSTEMI) advocate starting statins before patients are discharged from the hospital, but they don’t specify precisely when.7

Considering the higher risk of coronary events before and after PCI and statins’ pleiotropic effects, it is reasonable to investigate the optimal time for starting statins in patients with ACS.

STUDY SUMMARY: Meta-analysis of 20 RCTs shows
 statins before PCI cuts risk of MI

Navarese et al1 performed a systematic review and meta-analysis of studies comparing the clinical outcomes of patients with ACS who received statins before or after PCI (statins group) vs those who received low-dose statins or no statins (control group). The authors searched PubMed, Cochrane, Google Scholar, and CINAHL databases as well as key conference proceedings for studies published before November 2013. Using reasonable inclusion and exclusion criteria and appropriate statistical methods, they analyzed the results of 20 randomized controlled trials that included 8750 patients. Four studies enrolled only patients with ST elevation MI, 8 were restricted to NSTEMI, and the remaining 8 studies enrolled patients with any type of MI or unstable angina.

For patients who were started on a statin before PCI, the mean timing of administration was 0.53 ± 0.42 days before. For those started after PCI, the average time to administration was 3.18 ± 3.56 days after.

Administering statins before PCI resulted
 in a greater reduction in the odds of MI than starting them afterward. Whether administered before or after PCI, statins reduced the incidence of MIs. The overall 30-day incidence of MIs was 3.4% (123 of 3621) in the statins group and 5% (179 of 3577) in the control group. This resulted in an absolute risk reduction of 1.6% (number needed to treat=62.5), and a reduction of the odds of MI by 33% (odds ratio [OR]=0.67; 95% confidence interval [CI], 0.53-0.84; P=.0007). There was also a trend toward reduced mortality in the statin group (OR=0.66; 95% CI, 0.43-1.02; P=.06).

In addition, administering statins before PCI resulted in a greater reduction in the odds of MI at 30 days (OR=0.38; 95% CI, 0.24-0.59; P<.0001) than starting them post-PCI (OR=0.85; 95% CI, 0.64-1.13; P=.28) when compared to the controls. The difference between the pre-PCI OR and the post-PCI OR was statistically significant (P=.002). These findings persisted past 30 days (P=.06).

 

 

 

WHAT'S NEW: Early statin administration 
is most effective

According to ACC/AHA guidelines, all patients with ACS should be receiving a statin by the time they are discharged. However, when to start the statin is not specified. This meta-analysis is the first report to show that administering a statin before PCI can significantly reduce the risk of subsequent MI.

CAVEATS: Benefits might vary
 with different statins


The studies evaluated in this meta-analysis used various statins and dosing regimens, which could have affected the results. However, sensitivity analyses found similar benefits across different types of statins. In addition, most of the included trials used high doses of statins, which minimized the potential discrepancy in outcomes from various dosing regimens. And while the included studies were not perfect, Navarese et al1 used reasonable methods to identify potential biases.

CHALLENGES TO IMPLEMENTATION: No barriers 
to starting statins earlier


Implementing this intervention may be as simple as editing a standard order. This meta-analysis also suggests that the earlier the intervention, the greater the benefit, which may be an argument for starting a statin when a patient first presents for evaluation for ACS, since the risks of taking a statin are quite low. We believe it would be beneficial if the next update of the ACC/AHA guidelines7 included this recommendation.

ACKNOWLEDGEMENT
The PURLs Surveillance System was supported in part by Grant Number UL1RR024999 from the National Center For Research Resources, a Clinical Translational Science Award to the University of Chicago. The content is solely the responsibility of the authors and does not necessarily represent the official views of the National Center For Research Resources or the National Institutes of Health.

Files
References

1. Navarese EP, Kowalewski M, Andreotti F, et al. Meta-analysis of time-related benefits of statin therapy in patients with acute coronary syndrome undergoing percutaneous coronary intervention. Am J Cardiol. 2014;113:1753-1764.

2. Pignone M, Phillips C, Mulrow C. Use of lipid lowering drugs for primary prevention of coronary heart disease: meta-analysis of randomised trials. BMJ. 2000;321:983-986.

3. Prevention of cardiovascular events and death with pravastatin in patients with coronary heart disease and a broad range of initial cholesterol levels. The Long-Term Intervention with Pravastatin in Ischaemic Disease (LIPID) Study Group. N Engl J Med. 1998;339:1349-1357.

4. Liao JK. Beyond lipid lowering: the role of statins in vascular protection. Int J Cardiol. 2002;86:5-18.

5. Li J, Li JJ, He JG, et al. Atorvastatin decreases C-reactive protein-induced inflammatory response in pulmonary artery smooth muscle cells by inhibiting nuclear factor-kappaB pathway. Cardiovasc Ther. 2010;28:8-14.

6. Tandon V, Bano G, Khajuria V, et al. Pleiotropic effects of statins. Indian J Pharmacol. 2005;37:77-85.

7. Wright RS, Anderson JL, Adams CD, et al; American College of Cardiology Foundation/American Heart Association Task Force on Practice Guidelines. 2011 ACCF/AHA focused update incorporated into the ACC/AHA 2007 Guidelines for the Management of Patients with Unstable Angina/Non-ST-Elevation Myocardial Infarction: a report of the American College of Cardiology Foundation/American Heart Association Task Force on Practice Guidelines developed in collaboration with the American Academy of Family Physicians, Society for Cardiovascular Angiography and Interventions, and the Society of Thoracic Surgeons. J Am Coll Cardiol. 2011;57:e215-e367.

Article PDF
Author and Disclosure Information

Hanna Gov-Ari, MD
James J. Stevermer, MD, MSPH


Department of Family 
and Community Medicine, University of Missouri-Columbia

PURLs EDITOR
Bernard Ewigman, MD, MSPH
Department of Family Medicine, The University of Chicago

Issue
The Journal of Family Practice - 63(12)
Publications
Topics
Page Number
735,738
Legacy Keywords
statins; ACS; acute coronary syndrome; percutaneous coronary intervention; PCI; non-ST elevation myocardial infarction; NSTEMI; MI; myocardial infarction; Hanna Gov-Ari, MD; James J. Stevermer, MD, MSPH
Sections
Files
Files
Author and Disclosure Information

Hanna Gov-Ari, MD
James J. Stevermer, MD, MSPH


Department of Family 
and Community Medicine, University of Missouri-Columbia

PURLs EDITOR
Bernard Ewigman, MD, MSPH
Department of Family Medicine, The University of Chicago

Author and Disclosure Information

Hanna Gov-Ari, MD
James J. Stevermer, MD, MSPH


Department of Family 
and Community Medicine, University of Missouri-Columbia

PURLs EDITOR
Bernard Ewigman, MD, MSPH
Department of Family Medicine, The University of Chicago

Article PDF
Article PDF
Related Articles
PRACTICE CHANGER

Prescribe a high-dose statin before any patient with acute coronary syndrome (ACS) undergoes percutaneous coronary intervention (PCI); it may be reasonable to extend this to patients being evaluated for ACS.1

Strength of recommendation

A: Based on a meta-analysis

Navarese EP, Kowalewski M, Andreotti F, et al. Meta-analysis of time-related benefits of statin therapy in patients with acute coronary syndrome undergoing percutaneous coronary intervention. Am J Cardiol. 2014;113:1753-1764.

 

Illustrative case

A 48-year-old man comes to the emergency department with chest pain and is diagnosed with ACS. He is scheduled to have PCI within the next 24 hours. When should you start him on a statin?

Statins are the mainstay pharmaceutical treatment for hyperlipidemia, and are used for primary and secondary prevention of coronary artery disease and stroke.2,3 Well-known for their cholesterol-lowering effect, they also have benefits that are independent of their effects on lipids, including improving endothelial function, decreasing oxidative stress, and decreasing vascular inflammation.4-6

Compared to patients with stable angina, patients with ACS experience markedly higher rates of coronary events, especially immediately before and after PCI and during the subsequent 30 days.1 American College of Cardiology/American Heart Association (ACC/AHA) guidelines for the management of non-ST elevation myocardial infarction (NSTEMI) advocate starting statins before patients are discharged from the hospital, but they don’t specify precisely when.7

Considering the higher risk of coronary events before and after PCI and statins’ pleiotropic effects, it is reasonable to investigate the optimal time for starting statins in patients with ACS.

STUDY SUMMARY: Meta-analysis of 20 RCTs shows
 statins before PCI cuts risk of MI

Navarese et al1 performed a systematic review and meta-analysis of studies comparing the clinical outcomes of patients with ACS who received statins before or after PCI (statins group) vs those who received low-dose statins or no statins (control group). The authors searched PubMed, Cochrane, Google Scholar, and CINAHL databases as well as key conference proceedings for studies published before November 2013. Using reasonable inclusion and exclusion criteria and appropriate statistical methods, they analyzed the results of 20 randomized controlled trials that included 8750 patients. Four studies enrolled only patients with ST elevation MI, 8 were restricted to NSTEMI, and the remaining 8 studies enrolled patients with any type of MI or unstable angina.

For patients who were started on a statin before PCI, the mean timing of administration was 0.53 ± 0.42 days before. For those started after PCI, the average time to administration was 3.18 ± 3.56 days after.

Administering statins before PCI resulted
 in a greater reduction in the odds of MI than starting them afterward. Whether administered before or after PCI, statins reduced the incidence of MIs. The overall 30-day incidence of MIs was 3.4% (123 of 3621) in the statins group and 5% (179 of 3577) in the control group. This resulted in an absolute risk reduction of 1.6% (number needed to treat=62.5), and a reduction of the odds of MI by 33% (odds ratio [OR]=0.67; 95% confidence interval [CI], 0.53-0.84; P=.0007). There was also a trend toward reduced mortality in the statin group (OR=0.66; 95% CI, 0.43-1.02; P=.06).

In addition, administering statins before PCI resulted in a greater reduction in the odds of MI at 30 days (OR=0.38; 95% CI, 0.24-0.59; P<.0001) than starting them post-PCI (OR=0.85; 95% CI, 0.64-1.13; P=.28) when compared to the controls. The difference between the pre-PCI OR and the post-PCI OR was statistically significant (P=.002). These findings persisted past 30 days (P=.06).

 

 

 

WHAT'S NEW: Early statin administration 
is most effective

According to ACC/AHA guidelines, all patients with ACS should be receiving a statin by the time they are discharged. However, when to start the statin is not specified. This meta-analysis is the first report to show that administering a statin before PCI can significantly reduce the risk of subsequent MI.

CAVEATS: Benefits might vary
 with different statins


The studies evaluated in this meta-analysis used various statins and dosing regimens, which could have affected the results. However, sensitivity analyses found similar benefits across different types of statins. In addition, most of the included trials used high doses of statins, which minimized the potential discrepancy in outcomes from various dosing regimens. And while the included studies were not perfect, Navarese et al1 used reasonable methods to identify potential biases.

CHALLENGES TO IMPLEMENTATION: No barriers 
to starting statins earlier


Implementing this intervention may be as simple as editing a standard order. This meta-analysis also suggests that the earlier the intervention, the greater the benefit, which may be an argument for starting a statin when a patient first presents for evaluation for ACS, since the risks of taking a statin are quite low. We believe it would be beneficial if the next update of the ACC/AHA guidelines7 included this recommendation.

ACKNOWLEDGEMENT
The PURLs Surveillance System was supported in part by Grant Number UL1RR024999 from the National Center For Research Resources, a Clinical Translational Science Award to the University of Chicago. The content is solely the responsibility of the authors and does not necessarily represent the official views of the National Center For Research Resources or the National Institutes of Health.

PRACTICE CHANGER

Prescribe a high-dose statin before any patient with acute coronary syndrome (ACS) undergoes percutaneous coronary intervention (PCI); it may be reasonable to extend this to patients being evaluated for ACS.1

Strength of recommendation

A: Based on a meta-analysis

Navarese EP, Kowalewski M, Andreotti F, et al. Meta-analysis of time-related benefits of statin therapy in patients with acute coronary syndrome undergoing percutaneous coronary intervention. Am J Cardiol. 2014;113:1753-1764.

 

Illustrative case

A 48-year-old man comes to the emergency department with chest pain and is diagnosed with ACS. He is scheduled to have PCI within the next 24 hours. When should you start him on a statin?

Statins are the mainstay pharmaceutical treatment for hyperlipidemia, and are used for primary and secondary prevention of coronary artery disease and stroke.2,3 Well-known for their cholesterol-lowering effect, they also have benefits that are independent of their effects on lipids, including improving endothelial function, decreasing oxidative stress, and decreasing vascular inflammation.4-6

Compared to patients with stable angina, patients with ACS experience markedly higher rates of coronary events, especially immediately before and after PCI and during the subsequent 30 days.1 American College of Cardiology/American Heart Association (ACC/AHA) guidelines for the management of non-ST elevation myocardial infarction (NSTEMI) advocate starting statins before patients are discharged from the hospital, but they don’t specify precisely when.7

Considering the higher risk of coronary events before and after PCI and statins’ pleiotropic effects, it is reasonable to investigate the optimal time for starting statins in patients with ACS.

STUDY SUMMARY: Meta-analysis of 20 RCTs shows
 statins before PCI cuts risk of MI

Navarese et al1 performed a systematic review and meta-analysis of studies comparing the clinical outcomes of patients with ACS who received statins before or after PCI (statins group) vs those who received low-dose statins or no statins (control group). The authors searched PubMed, Cochrane, Google Scholar, and CINAHL databases as well as key conference proceedings for studies published before November 2013. Using reasonable inclusion and exclusion criteria and appropriate statistical methods, they analyzed the results of 20 randomized controlled trials that included 8750 patients. Four studies enrolled only patients with ST elevation MI, 8 were restricted to NSTEMI, and the remaining 8 studies enrolled patients with any type of MI or unstable angina.

For patients who were started on a statin before PCI, the mean timing of administration was 0.53 ± 0.42 days before. For those started after PCI, the average time to administration was 3.18 ± 3.56 days after.

Administering statins before PCI resulted
 in a greater reduction in the odds of MI than starting them afterward. Whether administered before or after PCI, statins reduced the incidence of MIs. The overall 30-day incidence of MIs was 3.4% (123 of 3621) in the statins group and 5% (179 of 3577) in the control group. This resulted in an absolute risk reduction of 1.6% (number needed to treat=62.5), and a reduction of the odds of MI by 33% (odds ratio [OR]=0.67; 95% confidence interval [CI], 0.53-0.84; P=.0007). There was also a trend toward reduced mortality in the statin group (OR=0.66; 95% CI, 0.43-1.02; P=.06).

In addition, administering statins before PCI resulted in a greater reduction in the odds of MI at 30 days (OR=0.38; 95% CI, 0.24-0.59; P<.0001) than starting them post-PCI (OR=0.85; 95% CI, 0.64-1.13; P=.28) when compared to the controls. The difference between the pre-PCI OR and the post-PCI OR was statistically significant (P=.002). These findings persisted past 30 days (P=.06).

 

 

 

WHAT'S NEW: Early statin administration 
is most effective

According to ACC/AHA guidelines, all patients with ACS should be receiving a statin by the time they are discharged. However, when to start the statin is not specified. This meta-analysis is the first report to show that administering a statin before PCI can significantly reduce the risk of subsequent MI.

CAVEATS: Benefits might vary
 with different statins


The studies evaluated in this meta-analysis used various statins and dosing regimens, which could have affected the results. However, sensitivity analyses found similar benefits across different types of statins. In addition, most of the included trials used high doses of statins, which minimized the potential discrepancy in outcomes from various dosing regimens. And while the included studies were not perfect, Navarese et al1 used reasonable methods to identify potential biases.

CHALLENGES TO IMPLEMENTATION: No barriers 
to starting statins earlier


Implementing this intervention may be as simple as editing a standard order. This meta-analysis also suggests that the earlier the intervention, the greater the benefit, which may be an argument for starting a statin when a patient first presents for evaluation for ACS, since the risks of taking a statin are quite low. We believe it would be beneficial if the next update of the ACC/AHA guidelines7 included this recommendation.

ACKNOWLEDGEMENT
The PURLs Surveillance System was supported in part by Grant Number UL1RR024999 from the National Center For Research Resources, a Clinical Translational Science Award to the University of Chicago. The content is solely the responsibility of the authors and does not necessarily represent the official views of the National Center For Research Resources or the National Institutes of Health.

References

1. Navarese EP, Kowalewski M, Andreotti F, et al. Meta-analysis of time-related benefits of statin therapy in patients with acute coronary syndrome undergoing percutaneous coronary intervention. Am J Cardiol. 2014;113:1753-1764.

2. Pignone M, Phillips C, Mulrow C. Use of lipid lowering drugs for primary prevention of coronary heart disease: meta-analysis of randomised trials. BMJ. 2000;321:983-986.

3. Prevention of cardiovascular events and death with pravastatin in patients with coronary heart disease and a broad range of initial cholesterol levels. The Long-Term Intervention with Pravastatin in Ischaemic Disease (LIPID) Study Group. N Engl J Med. 1998;339:1349-1357.

4. Liao JK. Beyond lipid lowering: the role of statins in vascular protection. Int J Cardiol. 2002;86:5-18.

5. Li J, Li JJ, He JG, et al. Atorvastatin decreases C-reactive protein-induced inflammatory response in pulmonary artery smooth muscle cells by inhibiting nuclear factor-kappaB pathway. Cardiovasc Ther. 2010;28:8-14.

6. Tandon V, Bano G, Khajuria V, et al. Pleiotropic effects of statins. Indian J Pharmacol. 2005;37:77-85.

7. Wright RS, Anderson JL, Adams CD, et al; American College of Cardiology Foundation/American Heart Association Task Force on Practice Guidelines. 2011 ACCF/AHA focused update incorporated into the ACC/AHA 2007 Guidelines for the Management of Patients with Unstable Angina/Non-ST-Elevation Myocardial Infarction: a report of the American College of Cardiology Foundation/American Heart Association Task Force on Practice Guidelines developed in collaboration with the American Academy of Family Physicians, Society for Cardiovascular Angiography and Interventions, and the Society of Thoracic Surgeons. J Am Coll Cardiol. 2011;57:e215-e367.

References

1. Navarese EP, Kowalewski M, Andreotti F, et al. Meta-analysis of time-related benefits of statin therapy in patients with acute coronary syndrome undergoing percutaneous coronary intervention. Am J Cardiol. 2014;113:1753-1764.

2. Pignone M, Phillips C, Mulrow C. Use of lipid lowering drugs for primary prevention of coronary heart disease: meta-analysis of randomised trials. BMJ. 2000;321:983-986.

3. Prevention of cardiovascular events and death with pravastatin in patients with coronary heart disease and a broad range of initial cholesterol levels. The Long-Term Intervention with Pravastatin in Ischaemic Disease (LIPID) Study Group. N Engl J Med. 1998;339:1349-1357.

4. Liao JK. Beyond lipid lowering: the role of statins in vascular protection. Int J Cardiol. 2002;86:5-18.

5. Li J, Li JJ, He JG, et al. Atorvastatin decreases C-reactive protein-induced inflammatory response in pulmonary artery smooth muscle cells by inhibiting nuclear factor-kappaB pathway. Cardiovasc Ther. 2010;28:8-14.

6. Tandon V, Bano G, Khajuria V, et al. Pleiotropic effects of statins. Indian J Pharmacol. 2005;37:77-85.

7. Wright RS, Anderson JL, Adams CD, et al; American College of Cardiology Foundation/American Heart Association Task Force on Practice Guidelines. 2011 ACCF/AHA focused update incorporated into the ACC/AHA 2007 Guidelines for the Management of Patients with Unstable Angina/Non-ST-Elevation Myocardial Infarction: a report of the American College of Cardiology Foundation/American Heart Association Task Force on Practice Guidelines developed in collaboration with the American Academy of Family Physicians, Society for Cardiovascular Angiography and Interventions, and the Society of Thoracic Surgeons. J Am Coll Cardiol. 2011;57:e215-e367.

Issue
The Journal of Family Practice - 63(12)
Issue
The Journal of Family Practice - 63(12)
Page Number
735,738
Page Number
735,738
Publications
Publications
Topics
Article Type
Display Headline
Prescribing statins for patients with ACS? No need to wait
Display Headline
Prescribing statins for patients with ACS? No need to wait
Legacy Keywords
statins; ACS; acute coronary syndrome; percutaneous coronary intervention; PCI; non-ST elevation myocardial infarction; NSTEMI; MI; myocardial infarction; Hanna Gov-Ari, MD; James J. Stevermer, MD, MSPH
Legacy Keywords
statins; ACS; acute coronary syndrome; percutaneous coronary intervention; PCI; non-ST elevation myocardial infarction; NSTEMI; MI; myocardial infarction; Hanna Gov-Ari, MD; James J. Stevermer, MD, MSPH
Sections
PURLs Copyright

Copyright © 2014 Family Physicians Inquiries Network. All rights reserved.

Disallow All Ads
Alternative CME
Use ProPublica
Hide sidebar & use full width
render the right sidebar.
Conference Recap Checkbox
Not Conference Recap
Clinical Edge
Display the Slideshow in this Article
Article PDF Media
Media Files

A Simple Way to Reduce Catheter-associated UTIs

Article Type
Changed
Tue, 12/13/2016 - 12:08
Display Headline
A Simple Way to Reduce Catheter-associated UTIs
The administration of a prophylactic antibiotic when a surgical patient’s urinary catheter is removed can cut the rate of urinary tract infections in half.

PRACTICE CHANGER
Ensure that antibiotics are administered to surgical patients when their urinary catheter is removed to reduce the risk for urinary tract infections (UTIs).1

STRENGTH OF RECOMMENDATION
B: Based on a meta-analysis.1

ILLUSTRATIVE CASE
A 49-year-old man was admitted to the hospital for resection of a vertebral mass. He is almost ready for discharge, and soon his urinary catheter will be removed. Should he be given an antibiotic when his catheter is removed to prevent a UTI?

Approximately 15% to 25% of hospitalized patients receive a urinary catheter, typically during the perioperative period.2 UTIs are the most common hospital-acquired infections, and virtually all of these UTIs are caused by instrumentation of the urinary tract, primarily by catheters.2

Although the mortality rate among patients with catheter-associated UTIs (CAUTIs) is just 2.3%, CAUTIs are the leading cause of hospital-acquired bacteremia, which increases morbidity and length of stay.2 The most ­common pathogens for CAUTIs are Escherichia coli (21.4%), Candida species (21%), and Enterococcus species (14.9%).2Pseudomonas aeruginosa, Klebsiella, and Enterobacter species comprise the bulk of the remainder.2

Support for antibiotic prophylaxis has historically been equivocal
Until now, no data clearly supported routine use of prophylactic antibiotics after urinary catheterization. CDC guidelines published in 2009 outline which patients are appropriate candidates for catheterization but do not recommend routine use of antibiotics to prevent CAUTIs.2 A 2014 Infectious Diseases Society of America practice recommendation, which was published after the study reported on here, states the benefit of antibiotics at the time of catheter removal is an unresolved issue.3

STUDY SUMMARY
Analysis shows prophylactic antibiotics reduce UTIs
Marschall et al1 searched multiple databases for studies published between 1947 and 2012 that evaluated prophylactic use of antibiotics at the time of urinary catheter removal. The endpoint for their analysis was symptomatic UTI, which they defined as bacteriuria plus at least one clinical symptom. Trials were excluded if patients had suprapubic catheters or if antibiotics were started shortly after the catheter was inserted.

The authors analyzed seven studies. Six were randomized controlled trials, of which one was unpublished. The seventh trial was a nonrandomized study that compared outcomes of patients of two surgeons, one of whom used prophylactic antibiotics and one who did not. Five studies enrolled surgical patients exclusively, including two that focused on urology patients. In all of the studies, patients had a urinary catheter in place for fewer than 15 days. The duration of antibiotic treatment varied from a single dose to three days. The antibiotics used included trimethoprim/sulfamethoxazole, nitrofurantoin, ciprofloxacin, and a cephalosporin.

Antibiotic prophylaxis significantly reduced the rate of ­CAUTIs. The absolute risk reduction was 5.8%; the rate of CAUTIs was 4.7% in the group treated with antibiotics and 10.5% in the control group. The number needed to treat to prevent one CAUTI was 17, with a risk ratio (RR) of .45. The RR varied only slightly (.36) when the researchers repeated their analysis but excluded the unpublished trial and remained at .45 when they analyzed only studies of surgical patients.

The reduction in CAUTIs remained consistent despite varying lengths of antibiotic administration and choice of antimicrobial agents. However, when the authors looked at pooled results just from the two studies that included both surgical and medical patients, they found no decrease in CAUTIs.

WHAT’S NEW
We now have an effective way to reduce CAUTIs
Prophylactic use of antibiotics when a urinary catheter is removed appears to reduce the rate of CAUTIs by more than 50% in surgical patients. The 2009 CDC guidelines on CAUTI prevention emphasize the use of appropriate infection control measures and limiting the duration of urinary catheter use.2 Now there are data showing a reduction in the incidence of CAUTIs when prophylactic antibiotics are given during catheter removal.

Continue for caveats and challenges >>

 

 

CAVEATS
Results may not apply to ­nonsurgical patients
This meta-analysis does not provide enough information to identify which patients are most likely to benefit from antibiotic prophylaxis. Most patients (92%) in this analysis had undergone surgery, but urinary catheterization is common among medically hospitalized patients. Studies of antibiotic prophylaxis at the time of catheter removal in nonsurgical patients are needed to strengthen the recommendation of this practice for all patients.

Some of the studies analyzed may have been biased. The authors determined that most of the studies in their meta-analysis were at high risk for attrition bias because there was potential for systematic differences in withdrawals between the treatment and control groups. In addition, in most studies, the randomization and allocation appeared to be inadequate, which increased the risk for selection bias.

CHALLENGES TO IMPLEMENTATION
Which antibiotics to use—and for how long—remains unclear
Antibiotic choice depends upon institutional policies and local resistance patterns, which complicates making universal recommendations. The optimal duration of treatment also is unknown, although this meta-analysis suggests that prophylaxis for three days or less can reduce CAUTI risk.

Catheters impregnated with antimicrobials or with microbial resistance barriers may be an alternative to administration of antibiotics at catheter removal, but in preliminary studies, these devices have not been shown to reduce the incidence of CAUTIs.4,5 Increasing antimicrobial resistance also complicates the widespread use of prophylaxis.

REFERENCES
1. Marschall J, Carpenter CR, Fowler S, et al; CDC Prevention Epicenters Program. Antibiotic prophylaxis for urinary tract infections after removal of urinary catheter: meta-analysis. BMJ. 2013;346:f3147.

2. Gould CV, Umscheid CA, Agarwal RK, et al. Guideline for prevention of catheter-associated urinary tract infections 2009. www.cdc.gov/hicpac/pdf/cauti/cautiguideline2009final.pdf. Accessed November 12, 2014.  

3. Lo E, Nicolle LE, Coffin SE, et al. Strategies to prevent catheter-associated urinary tract infections in acute care hospitals: 2014 update. Infect Control Hosp Epidemiol. 2014;35:464-479.

4. Pickard R, Lam T, Maclennan G, et al. Types of urethral catheter for reducing symptomatic urinary tract infections in hospitalised adults requiring short-term catheterisation: multicentre randomised controlled trial and economic evaluation of antimicrobial- and antiseptic-impregnated urethral catheters (the CATHETER trial). Health Technol Assess. 2012;16:1-197.

5. Pickard R, Lam T, MacLennan G, et al. Antimicrobial catheters for reduction of symptomatic urinary tract infection in adults requiring short-term catheterisation in hospital: a multicentre randomised controlled trial. Lancet. 2012;380:1927-1935.

ACKNOWLEDGEMENT
The PURLs Surveillance System was supported in part by Grant Number UL1RR024999 from the National Center For Research Resources, a Clinical Translational Science Award to the University of Chicago. The content is solely the responsibility of the authors and does not necessarily represent the official views of the National Center For Research Resources or the National Institutes of Health.

Copyright © 2014. The Family Physicians Inquiries Network. All rights reserved.

Reprinted with permission from the Family Physicians Inquiries Network and The Journal of Family Practice. 2014;63(5):E10-E12.

References

Article PDF
Author and Disclosure Information

Susan Pereira, MD, Liz Nguyen, MD, James J. Stevermer, MD, MSPH
Susan Pereira and James J. Stevermer are in the Department of Family and Community Medicine at the University of Missouri–Columbia. Liz Nguyen is in the Department of Family Medicine at the University of Chicago.

Issue
Clinician Reviews - 24(12)
Publications
Topics
Page Number
26,28
Legacy Keywords
urinary tract infection, UTI, catheter, antibiotics, surgery, catheter-associated urinary tract infection, CAUTI, trimethoprim/sulfamethoxazole, nitrofurantoin, ciprofloxacin, cephalosporin
Sections
Author and Disclosure Information

Susan Pereira, MD, Liz Nguyen, MD, James J. Stevermer, MD, MSPH
Susan Pereira and James J. Stevermer are in the Department of Family and Community Medicine at the University of Missouri–Columbia. Liz Nguyen is in the Department of Family Medicine at the University of Chicago.

Author and Disclosure Information

Susan Pereira, MD, Liz Nguyen, MD, James J. Stevermer, MD, MSPH
Susan Pereira and James J. Stevermer are in the Department of Family and Community Medicine at the University of Missouri–Columbia. Liz Nguyen is in the Department of Family Medicine at the University of Chicago.

Article PDF
Article PDF
Related Articles
The administration of a prophylactic antibiotic when a surgical patient’s urinary catheter is removed can cut the rate of urinary tract infections in half.
The administration of a prophylactic antibiotic when a surgical patient’s urinary catheter is removed can cut the rate of urinary tract infections in half.

PRACTICE CHANGER
Ensure that antibiotics are administered to surgical patients when their urinary catheter is removed to reduce the risk for urinary tract infections (UTIs).1

STRENGTH OF RECOMMENDATION
B: Based on a meta-analysis.1

ILLUSTRATIVE CASE
A 49-year-old man was admitted to the hospital for resection of a vertebral mass. He is almost ready for discharge, and soon his urinary catheter will be removed. Should he be given an antibiotic when his catheter is removed to prevent a UTI?

Approximately 15% to 25% of hospitalized patients receive a urinary catheter, typically during the perioperative period.2 UTIs are the most common hospital-acquired infections, and virtually all of these UTIs are caused by instrumentation of the urinary tract, primarily by catheters.2

Although the mortality rate among patients with catheter-associated UTIs (CAUTIs) is just 2.3%, CAUTIs are the leading cause of hospital-acquired bacteremia, which increases morbidity and length of stay.2 The most ­common pathogens for CAUTIs are Escherichia coli (21.4%), Candida species (21%), and Enterococcus species (14.9%).2Pseudomonas aeruginosa, Klebsiella, and Enterobacter species comprise the bulk of the remainder.2

Support for antibiotic prophylaxis has historically been equivocal
Until now, no data clearly supported routine use of prophylactic antibiotics after urinary catheterization. CDC guidelines published in 2009 outline which patients are appropriate candidates for catheterization but do not recommend routine use of antibiotics to prevent CAUTIs.2 A 2014 Infectious Diseases Society of America practice recommendation, which was published after the study reported on here, states the benefit of antibiotics at the time of catheter removal is an unresolved issue.3

STUDY SUMMARY
Analysis shows prophylactic antibiotics reduce UTIs
Marschall et al1 searched multiple databases for studies published between 1947 and 2012 that evaluated prophylactic use of antibiotics at the time of urinary catheter removal. The endpoint for their analysis was symptomatic UTI, which they defined as bacteriuria plus at least one clinical symptom. Trials were excluded if patients had suprapubic catheters or if antibiotics were started shortly after the catheter was inserted.

The authors analyzed seven studies. Six were randomized controlled trials, of which one was unpublished. The seventh trial was a nonrandomized study that compared outcomes of patients of two surgeons, one of whom used prophylactic antibiotics and one who did not. Five studies enrolled surgical patients exclusively, including two that focused on urology patients. In all of the studies, patients had a urinary catheter in place for fewer than 15 days. The duration of antibiotic treatment varied from a single dose to three days. The antibiotics used included trimethoprim/sulfamethoxazole, nitrofurantoin, ciprofloxacin, and a cephalosporin.

Antibiotic prophylaxis significantly reduced the rate of ­CAUTIs. The absolute risk reduction was 5.8%; the rate of CAUTIs was 4.7% in the group treated with antibiotics and 10.5% in the control group. The number needed to treat to prevent one CAUTI was 17, with a risk ratio (RR) of .45. The RR varied only slightly (.36) when the researchers repeated their analysis but excluded the unpublished trial and remained at .45 when they analyzed only studies of surgical patients.

The reduction in CAUTIs remained consistent despite varying lengths of antibiotic administration and choice of antimicrobial agents. However, when the authors looked at pooled results just from the two studies that included both surgical and medical patients, they found no decrease in CAUTIs.

WHAT’S NEW
We now have an effective way to reduce CAUTIs
Prophylactic use of antibiotics when a urinary catheter is removed appears to reduce the rate of CAUTIs by more than 50% in surgical patients. The 2009 CDC guidelines on CAUTI prevention emphasize the use of appropriate infection control measures and limiting the duration of urinary catheter use.2 Now there are data showing a reduction in the incidence of CAUTIs when prophylactic antibiotics are given during catheter removal.

Continue for caveats and challenges >>

 

 

CAVEATS
Results may not apply to ­nonsurgical patients
This meta-analysis does not provide enough information to identify which patients are most likely to benefit from antibiotic prophylaxis. Most patients (92%) in this analysis had undergone surgery, but urinary catheterization is common among medically hospitalized patients. Studies of antibiotic prophylaxis at the time of catheter removal in nonsurgical patients are needed to strengthen the recommendation of this practice for all patients.

Some of the studies analyzed may have been biased. The authors determined that most of the studies in their meta-analysis were at high risk for attrition bias because there was potential for systematic differences in withdrawals between the treatment and control groups. In addition, in most studies, the randomization and allocation appeared to be inadequate, which increased the risk for selection bias.

CHALLENGES TO IMPLEMENTATION
Which antibiotics to use—and for how long—remains unclear
Antibiotic choice depends upon institutional policies and local resistance patterns, which complicates making universal recommendations. The optimal duration of treatment also is unknown, although this meta-analysis suggests that prophylaxis for three days or less can reduce CAUTI risk.

Catheters impregnated with antimicrobials or with microbial resistance barriers may be an alternative to administration of antibiotics at catheter removal, but in preliminary studies, these devices have not been shown to reduce the incidence of CAUTIs.4,5 Increasing antimicrobial resistance also complicates the widespread use of prophylaxis.

REFERENCES
1. Marschall J, Carpenter CR, Fowler S, et al; CDC Prevention Epicenters Program. Antibiotic prophylaxis for urinary tract infections after removal of urinary catheter: meta-analysis. BMJ. 2013;346:f3147.

2. Gould CV, Umscheid CA, Agarwal RK, et al. Guideline for prevention of catheter-associated urinary tract infections 2009. www.cdc.gov/hicpac/pdf/cauti/cautiguideline2009final.pdf. Accessed November 12, 2014.  

3. Lo E, Nicolle LE, Coffin SE, et al. Strategies to prevent catheter-associated urinary tract infections in acute care hospitals: 2014 update. Infect Control Hosp Epidemiol. 2014;35:464-479.

4. Pickard R, Lam T, Maclennan G, et al. Types of urethral catheter for reducing symptomatic urinary tract infections in hospitalised adults requiring short-term catheterisation: multicentre randomised controlled trial and economic evaluation of antimicrobial- and antiseptic-impregnated urethral catheters (the CATHETER trial). Health Technol Assess. 2012;16:1-197.

5. Pickard R, Lam T, MacLennan G, et al. Antimicrobial catheters for reduction of symptomatic urinary tract infection in adults requiring short-term catheterisation in hospital: a multicentre randomised controlled trial. Lancet. 2012;380:1927-1935.

ACKNOWLEDGEMENT
The PURLs Surveillance System was supported in part by Grant Number UL1RR024999 from the National Center For Research Resources, a Clinical Translational Science Award to the University of Chicago. The content is solely the responsibility of the authors and does not necessarily represent the official views of the National Center For Research Resources or the National Institutes of Health.

Copyright © 2014. The Family Physicians Inquiries Network. All rights reserved.

Reprinted with permission from the Family Physicians Inquiries Network and The Journal of Family Practice. 2014;63(5):E10-E12.

PRACTICE CHANGER
Ensure that antibiotics are administered to surgical patients when their urinary catheter is removed to reduce the risk for urinary tract infections (UTIs).1

STRENGTH OF RECOMMENDATION
B: Based on a meta-analysis.1

ILLUSTRATIVE CASE
A 49-year-old man was admitted to the hospital for resection of a vertebral mass. He is almost ready for discharge, and soon his urinary catheter will be removed. Should he be given an antibiotic when his catheter is removed to prevent a UTI?

Approximately 15% to 25% of hospitalized patients receive a urinary catheter, typically during the perioperative period.2 UTIs are the most common hospital-acquired infections, and virtually all of these UTIs are caused by instrumentation of the urinary tract, primarily by catheters.2

Although the mortality rate among patients with catheter-associated UTIs (CAUTIs) is just 2.3%, CAUTIs are the leading cause of hospital-acquired bacteremia, which increases morbidity and length of stay.2 The most ­common pathogens for CAUTIs are Escherichia coli (21.4%), Candida species (21%), and Enterococcus species (14.9%).2Pseudomonas aeruginosa, Klebsiella, and Enterobacter species comprise the bulk of the remainder.2

Support for antibiotic prophylaxis has historically been equivocal
Until now, no data clearly supported routine use of prophylactic antibiotics after urinary catheterization. CDC guidelines published in 2009 outline which patients are appropriate candidates for catheterization but do not recommend routine use of antibiotics to prevent CAUTIs.2 A 2014 Infectious Diseases Society of America practice recommendation, which was published after the study reported on here, states the benefit of antibiotics at the time of catheter removal is an unresolved issue.3

STUDY SUMMARY
Analysis shows prophylactic antibiotics reduce UTIs
Marschall et al1 searched multiple databases for studies published between 1947 and 2012 that evaluated prophylactic use of antibiotics at the time of urinary catheter removal. The endpoint for their analysis was symptomatic UTI, which they defined as bacteriuria plus at least one clinical symptom. Trials were excluded if patients had suprapubic catheters or if antibiotics were started shortly after the catheter was inserted.

The authors analyzed seven studies. Six were randomized controlled trials, of which one was unpublished. The seventh trial was a nonrandomized study that compared outcomes of patients of two surgeons, one of whom used prophylactic antibiotics and one who did not. Five studies enrolled surgical patients exclusively, including two that focused on urology patients. In all of the studies, patients had a urinary catheter in place for fewer than 15 days. The duration of antibiotic treatment varied from a single dose to three days. The antibiotics used included trimethoprim/sulfamethoxazole, nitrofurantoin, ciprofloxacin, and a cephalosporin.

Antibiotic prophylaxis significantly reduced the rate of ­CAUTIs. The absolute risk reduction was 5.8%; the rate of CAUTIs was 4.7% in the group treated with antibiotics and 10.5% in the control group. The number needed to treat to prevent one CAUTI was 17, with a risk ratio (RR) of .45. The RR varied only slightly (.36) when the researchers repeated their analysis but excluded the unpublished trial and remained at .45 when they analyzed only studies of surgical patients.

The reduction in CAUTIs remained consistent despite varying lengths of antibiotic administration and choice of antimicrobial agents. However, when the authors looked at pooled results just from the two studies that included both surgical and medical patients, they found no decrease in CAUTIs.

WHAT’S NEW
We now have an effective way to reduce CAUTIs
Prophylactic use of antibiotics when a urinary catheter is removed appears to reduce the rate of CAUTIs by more than 50% in surgical patients. The 2009 CDC guidelines on CAUTI prevention emphasize the use of appropriate infection control measures and limiting the duration of urinary catheter use.2 Now there are data showing a reduction in the incidence of CAUTIs when prophylactic antibiotics are given during catheter removal.

Continue for caveats and challenges >>

 

 

CAVEATS
Results may not apply to ­nonsurgical patients
This meta-analysis does not provide enough information to identify which patients are most likely to benefit from antibiotic prophylaxis. Most patients (92%) in this analysis had undergone surgery, but urinary catheterization is common among medically hospitalized patients. Studies of antibiotic prophylaxis at the time of catheter removal in nonsurgical patients are needed to strengthen the recommendation of this practice for all patients.

Some of the studies analyzed may have been biased. The authors determined that most of the studies in their meta-analysis were at high risk for attrition bias because there was potential for systematic differences in withdrawals between the treatment and control groups. In addition, in most studies, the randomization and allocation appeared to be inadequate, which increased the risk for selection bias.

CHALLENGES TO IMPLEMENTATION
Which antibiotics to use—and for how long—remains unclear
Antibiotic choice depends upon institutional policies and local resistance patterns, which complicates making universal recommendations. The optimal duration of treatment also is unknown, although this meta-analysis suggests that prophylaxis for three days or less can reduce CAUTI risk.

Catheters impregnated with antimicrobials or with microbial resistance barriers may be an alternative to administration of antibiotics at catheter removal, but in preliminary studies, these devices have not been shown to reduce the incidence of CAUTIs.4,5 Increasing antimicrobial resistance also complicates the widespread use of prophylaxis.

REFERENCES
1. Marschall J, Carpenter CR, Fowler S, et al; CDC Prevention Epicenters Program. Antibiotic prophylaxis for urinary tract infections after removal of urinary catheter: meta-analysis. BMJ. 2013;346:f3147.

2. Gould CV, Umscheid CA, Agarwal RK, et al. Guideline for prevention of catheter-associated urinary tract infections 2009. www.cdc.gov/hicpac/pdf/cauti/cautiguideline2009final.pdf. Accessed November 12, 2014.  

3. Lo E, Nicolle LE, Coffin SE, et al. Strategies to prevent catheter-associated urinary tract infections in acute care hospitals: 2014 update. Infect Control Hosp Epidemiol. 2014;35:464-479.

4. Pickard R, Lam T, Maclennan G, et al. Types of urethral catheter for reducing symptomatic urinary tract infections in hospitalised adults requiring short-term catheterisation: multicentre randomised controlled trial and economic evaluation of antimicrobial- and antiseptic-impregnated urethral catheters (the CATHETER trial). Health Technol Assess. 2012;16:1-197.

5. Pickard R, Lam T, MacLennan G, et al. Antimicrobial catheters for reduction of symptomatic urinary tract infection in adults requiring short-term catheterisation in hospital: a multicentre randomised controlled trial. Lancet. 2012;380:1927-1935.

ACKNOWLEDGEMENT
The PURLs Surveillance System was supported in part by Grant Number UL1RR024999 from the National Center For Research Resources, a Clinical Translational Science Award to the University of Chicago. The content is solely the responsibility of the authors and does not necessarily represent the official views of the National Center For Research Resources or the National Institutes of Health.

Copyright © 2014. The Family Physicians Inquiries Network. All rights reserved.

Reprinted with permission from the Family Physicians Inquiries Network and The Journal of Family Practice. 2014;63(5):E10-E12.

References

References

Issue
Clinician Reviews - 24(12)
Issue
Clinician Reviews - 24(12)
Page Number
26,28
Page Number
26,28
Publications
Publications
Topics
Article Type
Display Headline
A Simple Way to Reduce Catheter-associated UTIs
Display Headline
A Simple Way to Reduce Catheter-associated UTIs
Legacy Keywords
urinary tract infection, UTI, catheter, antibiotics, surgery, catheter-associated urinary tract infection, CAUTI, trimethoprim/sulfamethoxazole, nitrofurantoin, ciprofloxacin, cephalosporin
Legacy Keywords
urinary tract infection, UTI, catheter, antibiotics, surgery, catheter-associated urinary tract infection, CAUTI, trimethoprim/sulfamethoxazole, nitrofurantoin, ciprofloxacin, cephalosporin
Sections
Article Source

PURLs Copyright

Inside the Article

Article PDF Media

An Antiemetic for Irritable Bowel Syndrome?

Article Type
Changed
Tue, 12/13/2016 - 12:08
Display Headline
An Antiemetic for Irritable Bowel Syndrome?
A drug used for cancer patients may provide some relief to patients with IBS.

PRACTICE CHANGER
Consider prescribing ondansetron (up to 24 mg/d) for patients who have irritable bowel syndrome with diarrhea (IBS-D).1

STRENGTH OF RECOMMENDATION
B: Based on a well-done double-blind, placebo-controlled randomized controlled trial (RCT).1

ILLUSTRATIVE CASE
A 23-year-old woman who was ­diagnosed with irritable bowel syndrome (IBS) comes to your clinic with complaints of increased frequency of defecation with watery stools and generalized, cramping abdominal pain. She also notes increased passage of mucus and a sensation of incomplete evacuation.

She says the only thing that relieves her pain is defecation. She has tried loperamide, acetaminophen, and ibuprofen without relief. She does not have Crohn disease or ulcerative colitis. What else can you offer her that is safe and effective?

IBS is a chronic, episodic functional gastrointestinal disorder characterized by abdominal pain or discomfort and altered bowel habits: constipation (IBS-C), diarrhea (IBS-D), or alternating periods of both—mixed (IBS-M).2 The diagnosis is based on Rome III criteria, which include recurrent abdominal pain or discomfort on at least three days per month in the past three months associated with two or more of the following: improvement with defecation, onset associated with a change in frequency of stool, and onset associated with a change in form (appearance) of stool.3 IBS often is unrecognized or untreated, and as few as 25% of patients with IBS seek care.4

IBS-D affects approximately 5% of the general population in North America.5,6 IBS-D is associated with a considerably decreased quality of life and is a common cause of work absenteeism.7,8 Because many conditions can cause diarrhea, patients typically undergo numerous tests before receiving an accurate diagnosis, which creates a financial burden.9

For many patients, current IBS treatments—including fiber supplements, laxatives, antidiarrheal medications, antispasmodics, and antidepressants such as tricyclics and selective serotonin reuptake inhibitors—are unsatisfactory.10 Alosetron, a 5-hydroxytryptamine 3 (5-HT3) receptor antagonist, has been used to treat IBS-D,11 but this medication was voluntarily withdrawn from the US market in 2000 due to concerns about ischemic colitis and severe constipation.12 It was reintroduced in 2002 but can be prescribed only by clinicians who enroll in a prescribing program provided by the manufacturer, and there are restrictions on its use.

Ondansetron—another 5-HT3 receptor antagonist used to treat nausea and vomiting caused by chemotherapy—may be another option for treating IBS-D. Garsed et al1 recently conducted an RCT to evaluate the efficacy of ondansetron for patients with IBS-D.

Study summary >>

 

 

STUDY SUMMARY
Ondansetron improves stool consistency, severity of IBS symptoms

In a five-week, double-blind crossover RCT, Garsed et al1 compared ondansetron with placebo for symptom relief in 120 patients who met Rome III criteria for IBS-D. All patients were ages 18 to 75 and had no evidence of inflammatory bowel disease. Exclusion criteria included pregnancy or breastfeeding, unwillingness to stop antidiarrheal medication, prior abdominal surgery other than appendectomy or cholecystectomy, or enrollment in another trial.

Patients were started on ondansetron 4 mg/d with dose titration up to 24 mg/d based on response; no dose adjustments were allowed during the last two weeks of the study. There was a two- to three-week washout between treatment periods.
The primary endpoint was average stool consistency in the last two weeks of treatment, as measured by the Bristol Stool Form (BSF) scale.13 The BSF is a visual scale that depicts stool as hard (type 1) to watery (type 7); types 3 and 4 describe normal stools. The study also looked at urgency and frequency of defecation, bowel transit time, and pain scores.

Treatment with ondansetron resulted in a small but statistically significant improvement in stool consistency. The mean difference in BSF score between ondansetron and placebo was –0.9, indicating slightly more formed stool with use of ondansetron. Scores for IBS severity—mild (a score of 75 to 175 out of 500), moderate (175 to 300), or severe (> 300)—were reduced by more points with ondansetron than with placebo (83 ± 9.8 vs 37 ± 9.7, respectively). Although this mean difference of 46 points fell just short of the 50-point threshold that is considered clinically significant, many patients exceeded this threshold.

Compared to those who received placebo, patients who took ondansetron also had less frequent defecation and lower urgency scores. Gut transit time was lengthened in the ondansetron group by 10 hours more than in the placebo group.

Pain scores did not change significantly for patients taking ondansetron, although they experienced significantly fewer days of urgency and bloating. Symptoms typically improved in as little as seven days but returned after ondansetron use stopped (typically within two weeks). Sixty-five percent of patients reported adequate relief with ondansetron, compared to 14% with placebo.

Patients whose diarrhea was more severe at baseline didn’t respond as well to ondansetron as did those whose diarrhea was less severe. The only frequent adverse effect was constipation, which occurred in 9% of patients receiving ondansetron and 2% of those on placebo.

WHAT’S NEW
Another option for IBS-D
A prior, smaller study of ondansetron that used a lower dosage (12 mg/d) suggested benefit in IBS-D.14 In that study, ondansetron decreased diarrhea and functional dyspepsia. The study by Garsed et al1 is the first large RCT to show significantly improved stool consistency, less frequent defecation, and less urgency and bloating from using ondansetron to treat IBS-D.

CAVEATS
Ondansetron doesn’t appear to reduce pain
In Garsed et al,1 patients who received ondansetron did not experience relief from pain, which is one of the main complaints of IBS. However, this study did find slight improvement in formed stools, symptom relief that approached—but did not quite reach—clinical significance, fewer days with urgency and bloating, and less frequent defecation.

This study did not evaluate the long-term effects of ondansetron use. However, ondansetron has been used for other indications for more than 25 years and has been reported to have a low risk for adverse effects.15

CHALLENGES TO IMPLEMENTATION
Remember ondansetron is not for IBS patients with constipation
Proper use of this drug among patients with IBS is key. The primary benefits of ondansetron are limited to IBS patients who have diarrhea, and not constipation. Ondansetron should not be prescribed to IBS patients who experience constipation or those with mixed symptoms.

REFERENCES
1.  Garsed K, Chernova J, Hastings M, et al. A randomised trial of ondansetron for the treatment of irritable bowel syndrome with diarrhoea. Gut. 2014;63:1617-1625.

2.  Hahn BA, Yan S, Strassels S. Impact of irritable bowel syndrome on quality of life and resource use in the United States and United Kingdom. Digestion. 1999;60:77-81.

3.  Drossman DA, Dumitrascu DL. Rome III: new standard for functional gastrointestinal disorders. J Gastrointestin Liver Dis. 2006;15:237-241.

4.  Luscombe FA. Health-related quality of life and associated psychosocial factors in irritable bowel syndrome: a review. Qual Life Res. 2000;9:161-176.

5.  Saito YA, Locke GR, Talley NJ, et al. A comparison of the Rome and Manning criteria for case identification in epidemiological investigations of irritable bowel syndrome. Am J Gastroenterol. 2000;95:2816-2824.

6.  Thompson WG, Heaton KW, Smyth GT, et al. Irritable bowel syndrome in general practice: prevalence, characteristics, and referral. Gut. 2000;46:78-82.

7.  Tillisch K, Labus JS, Naliboff BD, et al. Characterization of the alternating bowel habit subtype in patients with irritable bowel syndrome. Am J Gastroenterol. 2005;100:896-904.

8.  Schuster MM. Diagnostic evaluation of the irritable bowel syndrome. Gastroenterol Clin North Am. 1991;20:269-278.

9.  Sandler RS, Everhart JE, Donowitz M, et al. The burden of selected digestive diseases in the United States. Gastroenterology. 2002;122:1500-1511.

10.  Talley NJ. Pharmacologic therapy for the irritable bowel syndrome. Am J Gastroenterol. 2003;98:750-758.

11.  Andresen V, Montori VM, Keller J, et al. Effects of 5-hydroxytryptamine (serotonin) type 3 antagonists on symptom relief and constipation in nonconstipated irritable bowel syndrome: a systematic review and meta-analysis of randomized controlled trials. Clin Gastroenterol Hepatol. 2008;6:545-555.

12.  Chang L, Chey WD, Harris L, et al. Incidence of ischemic colitis and serious complications of constipation among patients using alosetron: systematic review of clinical trials and post-marketing surveillance data. Am J Gastroenterol. 2006;101:1069-1079.

13.  Heaton KW, O’Donnell LJ. An office guide to whole-gut transit time. Patients’ recollection of their stool form. J Clin Gastroenterol. 1994;19:28-30.

14.  Maxton DG, Morris J, Whorwell PJ. Selective 5‐hydroxytryptamine antagonism: a role in irritable bowel syndrome and functional dyspepsia? Aliment Pharmacol Ther. 1996;10:595-599.

15.  Gill SK, Einarson A. The safety of drugs for the treatment of nausea and vomiting of pregnancy. Expert Opin Drug Saf. 2007;6:685-694.

ACKNOWLEDGEMENT
The PURLs Surveillance System was supported in part by Grant Number UL1RR024999 from the National Center For Research Resources, a Clinical Translational Science Award to the University of Chicago. The content is solely the responsibility of the authors and does not necessarily represent the official views of the National Center For Research Resources or the National Institutes of Health.

Copyright © 2014. The Family Physicians Inquiries Network. All rights reserved.

Reprinted with permission from the Family Physicians Inquiries Network and The Journal of Family Practice. 2014;63(10):600-602.

References

Article PDF
Author and Disclosure Information

Robert Levy, MD, Jason Corbo, PharmD, BCPS, Shailendra Prasad, MBBS, MPH
Robert Levy and Shailendra Prasad are in the North Memorial
 Family Medicine Residency at the University of Minnesota, Minneapolis. Jason Corbo is in the Family Medicine Residency Program at UPMC St. Margaret in Pittsburgh.

Issue
Clinician Reviews - 24(11)
Publications
Topics
Page Number
16-18
Legacy Keywords
irritable bowel syndrome, diarrhea, ondansetron, antiemetic, cancer treatment, defecation, stools, abdominal pain, IBS, IBS-D
Sections
Author and Disclosure Information

Robert Levy, MD, Jason Corbo, PharmD, BCPS, Shailendra Prasad, MBBS, MPH
Robert Levy and Shailendra Prasad are in the North Memorial
 Family Medicine Residency at the University of Minnesota, Minneapolis. Jason Corbo is in the Family Medicine Residency Program at UPMC St. Margaret in Pittsburgh.

Author and Disclosure Information

Robert Levy, MD, Jason Corbo, PharmD, BCPS, Shailendra Prasad, MBBS, MPH
Robert Levy and Shailendra Prasad are in the North Memorial
 Family Medicine Residency at the University of Minnesota, Minneapolis. Jason Corbo is in the Family Medicine Residency Program at UPMC St. Margaret in Pittsburgh.

Article PDF
Article PDF
A drug used for cancer patients may provide some relief to patients with IBS.
A drug used for cancer patients may provide some relief to patients with IBS.

PRACTICE CHANGER
Consider prescribing ondansetron (up to 24 mg/d) for patients who have irritable bowel syndrome with diarrhea (IBS-D).1

STRENGTH OF RECOMMENDATION
B: Based on a well-done double-blind, placebo-controlled randomized controlled trial (RCT).1

ILLUSTRATIVE CASE
A 23-year-old woman who was ­diagnosed with irritable bowel syndrome (IBS) comes to your clinic with complaints of increased frequency of defecation with watery stools and generalized, cramping abdominal pain. She also notes increased passage of mucus and a sensation of incomplete evacuation.

She says the only thing that relieves her pain is defecation. She has tried loperamide, acetaminophen, and ibuprofen without relief. She does not have Crohn disease or ulcerative colitis. What else can you offer her that is safe and effective?

IBS is a chronic, episodic functional gastrointestinal disorder characterized by abdominal pain or discomfort and altered bowel habits: constipation (IBS-C), diarrhea (IBS-D), or alternating periods of both—mixed (IBS-M).2 The diagnosis is based on Rome III criteria, which include recurrent abdominal pain or discomfort on at least three days per month in the past three months associated with two or more of the following: improvement with defecation, onset associated with a change in frequency of stool, and onset associated with a change in form (appearance) of stool.3 IBS often is unrecognized or untreated, and as few as 25% of patients with IBS seek care.4

IBS-D affects approximately 5% of the general population in North America.5,6 IBS-D is associated with a considerably decreased quality of life and is a common cause of work absenteeism.7,8 Because many conditions can cause diarrhea, patients typically undergo numerous tests before receiving an accurate diagnosis, which creates a financial burden.9

For many patients, current IBS treatments—including fiber supplements, laxatives, antidiarrheal medications, antispasmodics, and antidepressants such as tricyclics and selective serotonin reuptake inhibitors—are unsatisfactory.10 Alosetron, a 5-hydroxytryptamine 3 (5-HT3) receptor antagonist, has been used to treat IBS-D,11 but this medication was voluntarily withdrawn from the US market in 2000 due to concerns about ischemic colitis and severe constipation.12 It was reintroduced in 2002 but can be prescribed only by clinicians who enroll in a prescribing program provided by the manufacturer, and there are restrictions on its use.

Ondansetron—another 5-HT3 receptor antagonist used to treat nausea and vomiting caused by chemotherapy—may be another option for treating IBS-D. Garsed et al1 recently conducted an RCT to evaluate the efficacy of ondansetron for patients with IBS-D.

Study summary >>

 

 

STUDY SUMMARY
Ondansetron improves stool consistency, severity of IBS symptoms

In a five-week, double-blind crossover RCT, Garsed et al1 compared ondansetron with placebo for symptom relief in 120 patients who met Rome III criteria for IBS-D. All patients were ages 18 to 75 and had no evidence of inflammatory bowel disease. Exclusion criteria included pregnancy or breastfeeding, unwillingness to stop antidiarrheal medication, prior abdominal surgery other than appendectomy or cholecystectomy, or enrollment in another trial.

Patients were started on ondansetron 4 mg/d with dose titration up to 24 mg/d based on response; no dose adjustments were allowed during the last two weeks of the study. There was a two- to three-week washout between treatment periods.
The primary endpoint was average stool consistency in the last two weeks of treatment, as measured by the Bristol Stool Form (BSF) scale.13 The BSF is a visual scale that depicts stool as hard (type 1) to watery (type 7); types 3 and 4 describe normal stools. The study also looked at urgency and frequency of defecation, bowel transit time, and pain scores.

Treatment with ondansetron resulted in a small but statistically significant improvement in stool consistency. The mean difference in BSF score between ondansetron and placebo was –0.9, indicating slightly more formed stool with use of ondansetron. Scores for IBS severity—mild (a score of 75 to 175 out of 500), moderate (175 to 300), or severe (> 300)—were reduced by more points with ondansetron than with placebo (83 ± 9.8 vs 37 ± 9.7, respectively). Although this mean difference of 46 points fell just short of the 50-point threshold that is considered clinically significant, many patients exceeded this threshold.

Compared to those who received placebo, patients who took ondansetron also had less frequent defecation and lower urgency scores. Gut transit time was lengthened in the ondansetron group by 10 hours more than in the placebo group.

Pain scores did not change significantly for patients taking ondansetron, although they experienced significantly fewer days of urgency and bloating. Symptoms typically improved in as little as seven days but returned after ondansetron use stopped (typically within two weeks). Sixty-five percent of patients reported adequate relief with ondansetron, compared to 14% with placebo.

Patients whose diarrhea was more severe at baseline didn’t respond as well to ondansetron as did those whose diarrhea was less severe. The only frequent adverse effect was constipation, which occurred in 9% of patients receiving ondansetron and 2% of those on placebo.

WHAT’S NEW
Another option for IBS-D
A prior, smaller study of ondansetron that used a lower dosage (12 mg/d) suggested benefit in IBS-D.14 In that study, ondansetron decreased diarrhea and functional dyspepsia. The study by Garsed et al1 is the first large RCT to show significantly improved stool consistency, less frequent defecation, and less urgency and bloating from using ondansetron to treat IBS-D.

CAVEATS
Ondansetron doesn’t appear to reduce pain
In Garsed et al,1 patients who received ondansetron did not experience relief from pain, which is one of the main complaints of IBS. However, this study did find slight improvement in formed stools, symptom relief that approached—but did not quite reach—clinical significance, fewer days with urgency and bloating, and less frequent defecation.

This study did not evaluate the long-term effects of ondansetron use. However, ondansetron has been used for other indications for more than 25 years and has been reported to have a low risk for adverse effects.15

CHALLENGES TO IMPLEMENTATION
Remember ondansetron is not for IBS patients with constipation
Proper use of this drug among patients with IBS is key. The primary benefits of ondansetron are limited to IBS patients who have diarrhea, and not constipation. Ondansetron should not be prescribed to IBS patients who experience constipation or those with mixed symptoms.

REFERENCES
1.  Garsed K, Chernova J, Hastings M, et al. A randomised trial of ondansetron for the treatment of irritable bowel syndrome with diarrhoea. Gut. 2014;63:1617-1625.

2.  Hahn BA, Yan S, Strassels S. Impact of irritable bowel syndrome on quality of life and resource use in the United States and United Kingdom. Digestion. 1999;60:77-81.

3.  Drossman DA, Dumitrascu DL. Rome III: new standard for functional gastrointestinal disorders. J Gastrointestin Liver Dis. 2006;15:237-241.

4.  Luscombe FA. Health-related quality of life and associated psychosocial factors in irritable bowel syndrome: a review. Qual Life Res. 2000;9:161-176.

5.  Saito YA, Locke GR, Talley NJ, et al. A comparison of the Rome and Manning criteria for case identification in epidemiological investigations of irritable bowel syndrome. Am J Gastroenterol. 2000;95:2816-2824.

6.  Thompson WG, Heaton KW, Smyth GT, et al. Irritable bowel syndrome in general practice: prevalence, characteristics, and referral. Gut. 2000;46:78-82.

7.  Tillisch K, Labus JS, Naliboff BD, et al. Characterization of the alternating bowel habit subtype in patients with irritable bowel syndrome. Am J Gastroenterol. 2005;100:896-904.

8.  Schuster MM. Diagnostic evaluation of the irritable bowel syndrome. Gastroenterol Clin North Am. 1991;20:269-278.

9.  Sandler RS, Everhart JE, Donowitz M, et al. The burden of selected digestive diseases in the United States. Gastroenterology. 2002;122:1500-1511.

10.  Talley NJ. Pharmacologic therapy for the irritable bowel syndrome. Am J Gastroenterol. 2003;98:750-758.

11.  Andresen V, Montori VM, Keller J, et al. Effects of 5-hydroxytryptamine (serotonin) type 3 antagonists on symptom relief and constipation in nonconstipated irritable bowel syndrome: a systematic review and meta-analysis of randomized controlled trials. Clin Gastroenterol Hepatol. 2008;6:545-555.

12.  Chang L, Chey WD, Harris L, et al. Incidence of ischemic colitis and serious complications of constipation among patients using alosetron: systematic review of clinical trials and post-marketing surveillance data. Am J Gastroenterol. 2006;101:1069-1079.

13.  Heaton KW, O’Donnell LJ. An office guide to whole-gut transit time. Patients’ recollection of their stool form. J Clin Gastroenterol. 1994;19:28-30.

14.  Maxton DG, Morris J, Whorwell PJ. Selective 5‐hydroxytryptamine antagonism: a role in irritable bowel syndrome and functional dyspepsia? Aliment Pharmacol Ther. 1996;10:595-599.

15.  Gill SK, Einarson A. The safety of drugs for the treatment of nausea and vomiting of pregnancy. Expert Opin Drug Saf. 2007;6:685-694.

ACKNOWLEDGEMENT
The PURLs Surveillance System was supported in part by Grant Number UL1RR024999 from the National Center For Research Resources, a Clinical Translational Science Award to the University of Chicago. The content is solely the responsibility of the authors and does not necessarily represent the official views of the National Center For Research Resources or the National Institutes of Health.

Copyright © 2014. The Family Physicians Inquiries Network. All rights reserved.

Reprinted with permission from the Family Physicians Inquiries Network and The Journal of Family Practice. 2014;63(10):600-602.

PRACTICE CHANGER
Consider prescribing ondansetron (up to 24 mg/d) for patients who have irritable bowel syndrome with diarrhea (IBS-D).1

STRENGTH OF RECOMMENDATION
B: Based on a well-done double-blind, placebo-controlled randomized controlled trial (RCT).1

ILLUSTRATIVE CASE
A 23-year-old woman who was ­diagnosed with irritable bowel syndrome (IBS) comes to your clinic with complaints of increased frequency of defecation with watery stools and generalized, cramping abdominal pain. She also notes increased passage of mucus and a sensation of incomplete evacuation.

She says the only thing that relieves her pain is defecation. She has tried loperamide, acetaminophen, and ibuprofen without relief. She does not have Crohn disease or ulcerative colitis. What else can you offer her that is safe and effective?

IBS is a chronic, episodic functional gastrointestinal disorder characterized by abdominal pain or discomfort and altered bowel habits: constipation (IBS-C), diarrhea (IBS-D), or alternating periods of both—mixed (IBS-M).2 The diagnosis is based on Rome III criteria, which include recurrent abdominal pain or discomfort on at least three days per month in the past three months associated with two or more of the following: improvement with defecation, onset associated with a change in frequency of stool, and onset associated with a change in form (appearance) of stool.3 IBS often is unrecognized or untreated, and as few as 25% of patients with IBS seek care.4

IBS-D affects approximately 5% of the general population in North America.5,6 IBS-D is associated with a considerably decreased quality of life and is a common cause of work absenteeism.7,8 Because many conditions can cause diarrhea, patients typically undergo numerous tests before receiving an accurate diagnosis, which creates a financial burden.9

For many patients, current IBS treatments—including fiber supplements, laxatives, antidiarrheal medications, antispasmodics, and antidepressants such as tricyclics and selective serotonin reuptake inhibitors—are unsatisfactory.10 Alosetron, a 5-hydroxytryptamine 3 (5-HT3) receptor antagonist, has been used to treat IBS-D,11 but this medication was voluntarily withdrawn from the US market in 2000 due to concerns about ischemic colitis and severe constipation.12 It was reintroduced in 2002 but can be prescribed only by clinicians who enroll in a prescribing program provided by the manufacturer, and there are restrictions on its use.

Ondansetron—another 5-HT3 receptor antagonist used to treat nausea and vomiting caused by chemotherapy—may be another option for treating IBS-D. Garsed et al1 recently conducted an RCT to evaluate the efficacy of ondansetron for patients with IBS-D.

Study summary >>

 

 

STUDY SUMMARY
Ondansetron improves stool consistency, severity of IBS symptoms

In a five-week, double-blind crossover RCT, Garsed et al1 compared ondansetron with placebo for symptom relief in 120 patients who met Rome III criteria for IBS-D. All patients were ages 18 to 75 and had no evidence of inflammatory bowel disease. Exclusion criteria included pregnancy or breastfeeding, unwillingness to stop antidiarrheal medication, prior abdominal surgery other than appendectomy or cholecystectomy, or enrollment in another trial.

Patients were started on ondansetron 4 mg/d with dose titration up to 24 mg/d based on response; no dose adjustments were allowed during the last two weeks of the study. There was a two- to three-week washout between treatment periods.
The primary endpoint was average stool consistency in the last two weeks of treatment, as measured by the Bristol Stool Form (BSF) scale.13 The BSF is a visual scale that depicts stool as hard (type 1) to watery (type 7); types 3 and 4 describe normal stools. The study also looked at urgency and frequency of defecation, bowel transit time, and pain scores.

Treatment with ondansetron resulted in a small but statistically significant improvement in stool consistency. The mean difference in BSF score between ondansetron and placebo was –0.9, indicating slightly more formed stool with use of ondansetron. Scores for IBS severity—mild (a score of 75 to 175 out of 500), moderate (175 to 300), or severe (> 300)—were reduced by more points with ondansetron than with placebo (83 ± 9.8 vs 37 ± 9.7, respectively). Although this mean difference of 46 points fell just short of the 50-point threshold that is considered clinically significant, many patients exceeded this threshold.

Compared to those who received placebo, patients who took ondansetron also had less frequent defecation and lower urgency scores. Gut transit time was lengthened in the ondansetron group by 10 hours more than in the placebo group.

Pain scores did not change significantly for patients taking ondansetron, although they experienced significantly fewer days of urgency and bloating. Symptoms typically improved in as little as seven days but returned after ondansetron use stopped (typically within two weeks). Sixty-five percent of patients reported adequate relief with ondansetron, compared to 14% with placebo.

Patients whose diarrhea was more severe at baseline didn’t respond as well to ondansetron as did those whose diarrhea was less severe. The only frequent adverse effect was constipation, which occurred in 9% of patients receiving ondansetron and 2% of those on placebo.

WHAT’S NEW
Another option for IBS-D
A prior, smaller study of ondansetron that used a lower dosage (12 mg/d) suggested benefit in IBS-D.14 In that study, ondansetron decreased diarrhea and functional dyspepsia. The study by Garsed et al1 is the first large RCT to show significantly improved stool consistency, less frequent defecation, and less urgency and bloating from using ondansetron to treat IBS-D.

CAVEATS
Ondansetron doesn’t appear to reduce pain
In Garsed et al,1 patients who received ondansetron did not experience relief from pain, which is one of the main complaints of IBS. However, this study did find slight improvement in formed stools, symptom relief that approached—but did not quite reach—clinical significance, fewer days with urgency and bloating, and less frequent defecation.

This study did not evaluate the long-term effects of ondansetron use. However, ondansetron has been used for other indications for more than 25 years and has been reported to have a low risk for adverse effects.15

CHALLENGES TO IMPLEMENTATION
Remember ondansetron is not for IBS patients with constipation
Proper use of this drug among patients with IBS is key. The primary benefits of ondansetron are limited to IBS patients who have diarrhea, and not constipation. Ondansetron should not be prescribed to IBS patients who experience constipation or those with mixed symptoms.

REFERENCES
1.  Garsed K, Chernova J, Hastings M, et al. A randomised trial of ondansetron for the treatment of irritable bowel syndrome with diarrhoea. Gut. 2014;63:1617-1625.

2.  Hahn BA, Yan S, Strassels S. Impact of irritable bowel syndrome on quality of life and resource use in the United States and United Kingdom. Digestion. 1999;60:77-81.

3.  Drossman DA, Dumitrascu DL. Rome III: new standard for functional gastrointestinal disorders. J Gastrointestin Liver Dis. 2006;15:237-241.

4.  Luscombe FA. Health-related quality of life and associated psychosocial factors in irritable bowel syndrome: a review. Qual Life Res. 2000;9:161-176.

5.  Saito YA, Locke GR, Talley NJ, et al. A comparison of the Rome and Manning criteria for case identification in epidemiological investigations of irritable bowel syndrome. Am J Gastroenterol. 2000;95:2816-2824.

6.  Thompson WG, Heaton KW, Smyth GT, et al. Irritable bowel syndrome in general practice: prevalence, characteristics, and referral. Gut. 2000;46:78-82.

7.  Tillisch K, Labus JS, Naliboff BD, et al. Characterization of the alternating bowel habit subtype in patients with irritable bowel syndrome. Am J Gastroenterol. 2005;100:896-904.

8.  Schuster MM. Diagnostic evaluation of the irritable bowel syndrome. Gastroenterol Clin North Am. 1991;20:269-278.

9.  Sandler RS, Everhart JE, Donowitz M, et al. The burden of selected digestive diseases in the United States. Gastroenterology. 2002;122:1500-1511.

10.  Talley NJ. Pharmacologic therapy for the irritable bowel syndrome. Am J Gastroenterol. 2003;98:750-758.

11.  Andresen V, Montori VM, Keller J, et al. Effects of 5-hydroxytryptamine (serotonin) type 3 antagonists on symptom relief and constipation in nonconstipated irritable bowel syndrome: a systematic review and meta-analysis of randomized controlled trials. Clin Gastroenterol Hepatol. 2008;6:545-555.

12.  Chang L, Chey WD, Harris L, et al. Incidence of ischemic colitis and serious complications of constipation among patients using alosetron: systematic review of clinical trials and post-marketing surveillance data. Am J Gastroenterol. 2006;101:1069-1079.

13.  Heaton KW, O’Donnell LJ. An office guide to whole-gut transit time. Patients’ recollection of their stool form. J Clin Gastroenterol. 1994;19:28-30.

14.  Maxton DG, Morris J, Whorwell PJ. Selective 5‐hydroxytryptamine antagonism: a role in irritable bowel syndrome and functional dyspepsia? Aliment Pharmacol Ther. 1996;10:595-599.

15.  Gill SK, Einarson A. The safety of drugs for the treatment of nausea and vomiting of pregnancy. Expert Opin Drug Saf. 2007;6:685-694.

ACKNOWLEDGEMENT
The PURLs Surveillance System was supported in part by Grant Number UL1RR024999 from the National Center For Research Resources, a Clinical Translational Science Award to the University of Chicago. The content is solely the responsibility of the authors and does not necessarily represent the official views of the National Center For Research Resources or the National Institutes of Health.

Copyright © 2014. The Family Physicians Inquiries Network. All rights reserved.

Reprinted with permission from the Family Physicians Inquiries Network and The Journal of Family Practice. 2014;63(10):600-602.

References

References

Issue
Clinician Reviews - 24(11)
Issue
Clinician Reviews - 24(11)
Page Number
16-18
Page Number
16-18
Publications
Publications
Topics
Article Type
Display Headline
An Antiemetic for Irritable Bowel Syndrome?
Display Headline
An Antiemetic for Irritable Bowel Syndrome?
Legacy Keywords
irritable bowel syndrome, diarrhea, ondansetron, antiemetic, cancer treatment, defecation, stools, abdominal pain, IBS, IBS-D
Legacy Keywords
irritable bowel syndrome, diarrhea, ondansetron, antiemetic, cancer treatment, defecation, stools, abdominal pain, IBS, IBS-D
Sections
Article Source

PURLs Copyright

Inside the Article

Article PDF Media

Probiotics for colic? A PURL update

Article Type
Changed
Mon, 01/14/2019 - 13:54
Display Headline
Probiotics for colic? A PURL update

In “Colicky baby? Here’s a surprising remedy” (J Fam Pract. 2011;60:34-36), we summarized a 2010 double-blind randomized controlled trial (RCT) that found the probiotic Lactobacillus reuteri DSM 17938 reduced daily crying time in colicky, exclusively breastfed infants.1

A recently published RCT of the same probiotic by Sung et al2 adds to the body of evidence and suggests that the jury may still be out as to the value of probiotics for colicky babies.

The newer study (which also measured colic using modified Wessel’s criteria) included babies who were formula-fed as well as those who were breastfed. When researchers looked at all babies as a single group, those who received probiotics fussed significantly more than those who received placebo at nearly all of the postintervention time points. However, when they delved deeper, the researchers noted that an increase in fussing occurred only among infants on formula. On the other hand, the time that breastfed infants spent crying or fussing did not vary significantly between those who received probiotics and those who received placebo.

Both the 2010 and 2014 studies used valid RCT methods with low risk for bias, so we’re not clear why the results (especially for breastfed infants) differed. The 2010 study was done in Italy and required breastfeeding moms 
to avoid cow’s milk, while the 2014 Sung et al2 study was conducted in Australia and did not have this requirement, so environmental factors may have played a role. The reporting method in the Sung et al2 study—a well-validated, detailed diary of infant behaviors—may have led to less parent recall error than the diary used in the 2010 study. All in all, we can only conclude that it is unclear whether probiotics work to reduce crying in colicky infants.

A safe bet may be to avoid recommending probiotics for colicky formula-fed infants, since no study of this population has shown probiotics are effective, and in the Sung et al2 study, they appeared to worsen symptoms. For breastfed babies, there is no evidence of harm, and mixed evidence on whether probiotics help.

References

1. Savino F, Cordisco L, Tarasco V, et al. Lactobacillus reuteri DSM 17938 in infantile colic: a randomized, double-blind, placebo-controlled trial. Pediatrics. 2010;126:e526-e533.

2. Sung V, Hiscock H, Tang ML, et al. Treating infant colic with the probiotic Lactobacillus reuteri: double blind, placebo controlled randomised trial. BMJ. 2014;348:g2107.

Article PDF
Author and Disclosure Information

Debra B. Stulberg, MD, MA
Kate Rowland, MD, MS

The University of Chicago

Issue
The Journal of Family Practice - 63(10)
Publications
Topics
Page Number
602
Legacy Keywords
probiotics; colic; Lactobacillus reuteri DSM 17938; Debra B. Stulberg, MD, MA; Kate Rowland, MD, MS
Sections
Author and Disclosure Information

Debra B. Stulberg, MD, MA
Kate Rowland, MD, MS

The University of Chicago

Author and Disclosure Information

Debra B. Stulberg, MD, MA
Kate Rowland, MD, MS

The University of Chicago

Article PDF
Article PDF
Related Articles

In “Colicky baby? Here’s a surprising remedy” (J Fam Pract. 2011;60:34-36), we summarized a 2010 double-blind randomized controlled trial (RCT) that found the probiotic Lactobacillus reuteri DSM 17938 reduced daily crying time in colicky, exclusively breastfed infants.1

A recently published RCT of the same probiotic by Sung et al2 adds to the body of evidence and suggests that the jury may still be out as to the value of probiotics for colicky babies.

The newer study (which also measured colic using modified Wessel’s criteria) included babies who were formula-fed as well as those who were breastfed. When researchers looked at all babies as a single group, those who received probiotics fussed significantly more than those who received placebo at nearly all of the postintervention time points. However, when they delved deeper, the researchers noted that an increase in fussing occurred only among infants on formula. On the other hand, the time that breastfed infants spent crying or fussing did not vary significantly between those who received probiotics and those who received placebo.

Both the 2010 and 2014 studies used valid RCT methods with low risk for bias, so we’re not clear why the results (especially for breastfed infants) differed. The 2010 study was done in Italy and required breastfeeding moms 
to avoid cow’s milk, while the 2014 Sung et al2 study was conducted in Australia and did not have this requirement, so environmental factors may have played a role. The reporting method in the Sung et al2 study—a well-validated, detailed diary of infant behaviors—may have led to less parent recall error than the diary used in the 2010 study. All in all, we can only conclude that it is unclear whether probiotics work to reduce crying in colicky infants.

A safe bet may be to avoid recommending probiotics for colicky formula-fed infants, since no study of this population has shown probiotics are effective, and in the Sung et al2 study, they appeared to worsen symptoms. For breastfed babies, there is no evidence of harm, and mixed evidence on whether probiotics help.

In “Colicky baby? Here’s a surprising remedy” (J Fam Pract. 2011;60:34-36), we summarized a 2010 double-blind randomized controlled trial (RCT) that found the probiotic Lactobacillus reuteri DSM 17938 reduced daily crying time in colicky, exclusively breastfed infants.1

A recently published RCT of the same probiotic by Sung et al2 adds to the body of evidence and suggests that the jury may still be out as to the value of probiotics for colicky babies.

The newer study (which also measured colic using modified Wessel’s criteria) included babies who were formula-fed as well as those who were breastfed. When researchers looked at all babies as a single group, those who received probiotics fussed significantly more than those who received placebo at nearly all of the postintervention time points. However, when they delved deeper, the researchers noted that an increase in fussing occurred only among infants on formula. On the other hand, the time that breastfed infants spent crying or fussing did not vary significantly between those who received probiotics and those who received placebo.

Both the 2010 and 2014 studies used valid RCT methods with low risk for bias, so we’re not clear why the results (especially for breastfed infants) differed. The 2010 study was done in Italy and required breastfeeding moms 
to avoid cow’s milk, while the 2014 Sung et al2 study was conducted in Australia and did not have this requirement, so environmental factors may have played a role. The reporting method in the Sung et al2 study—a well-validated, detailed diary of infant behaviors—may have led to less parent recall error than the diary used in the 2010 study. All in all, we can only conclude that it is unclear whether probiotics work to reduce crying in colicky infants.

A safe bet may be to avoid recommending probiotics for colicky formula-fed infants, since no study of this population has shown probiotics are effective, and in the Sung et al2 study, they appeared to worsen symptoms. For breastfed babies, there is no evidence of harm, and mixed evidence on whether probiotics help.

References

1. Savino F, Cordisco L, Tarasco V, et al. Lactobacillus reuteri DSM 17938 in infantile colic: a randomized, double-blind, placebo-controlled trial. Pediatrics. 2010;126:e526-e533.

2. Sung V, Hiscock H, Tang ML, et al. Treating infant colic with the probiotic Lactobacillus reuteri: double blind, placebo controlled randomised trial. BMJ. 2014;348:g2107.

References

1. Savino F, Cordisco L, Tarasco V, et al. Lactobacillus reuteri DSM 17938 in infantile colic: a randomized, double-blind, placebo-controlled trial. Pediatrics. 2010;126:e526-e533.

2. Sung V, Hiscock H, Tang ML, et al. Treating infant colic with the probiotic Lactobacillus reuteri: double blind, placebo controlled randomised trial. BMJ. 2014;348:g2107.

Issue
The Journal of Family Practice - 63(10)
Issue
The Journal of Family Practice - 63(10)
Page Number
602
Page Number
602
Publications
Publications
Topics
Article Type
Display Headline
Probiotics for colic? A PURL update
Display Headline
Probiotics for colic? A PURL update
Legacy Keywords
probiotics; colic; Lactobacillus reuteri DSM 17938; Debra B. Stulberg, MD, MA; Kate Rowland, MD, MS
Legacy Keywords
probiotics; colic; Lactobacillus reuteri DSM 17938; Debra B. Stulberg, MD, MA; Kate Rowland, MD, MS
Sections
PURLs Copyright

Copyright © 2014 Family Physicians Inquiries Network. All rights reserved.

Disallow All Ads
Alternative CME
Article PDF Media

An antiemetic for irritable bowel syndrome?

Article Type
Changed
Mon, 07/13/2020 - 11:53
Display Headline
An antiemetic for irritable bowel syndrome?

 

PRACTICE CHANGER

Consider prescribing ondansetron up to 24 mg/d for patients who have irritable bowel syndrome with diarrhea (IBS-D).1

Strength of recommendation

B: Based on a well-done double-blind, placebo-controlled randomized controlled trial (RCT).

Garsed K, Chernova J, Hastings M, et al. A randomised trial of ondansetron for the treatment of irritable bowel syndrome with diarrhoea. Gut. 2014;63:1617-1625.

Illustrative case

A 23-year-old woman who was diagnosed with irritable bowel syndrome (IBS) comes to your clinic with complaints of increased frequency of defecation with watery stools and generalized, cramping abdominal pain. She also notes increased passage of mucus and a sensation of incomplete evacuation. She says the only thing that relieves her pain is defecation. She has tried loperamide, acetaminophen, and ibuprofen without relief. She does not have Crohn’s disease or ulcerative colitis. What else can you offer her that is safe and effective?

IBS is a chronic, episodic functional gastrointestinal disorder characterized by abdominal pain or discomfort and altered bowel habits (constipation [IBS-C], diarrhea [IBS-D], or alternating periods of both—mixed [IBS-M]).2 It is diagnosed based on Rome III criteria—recurrent abdominal pain or discomfort at least 3 days/month in the last 3 months associated with ≥2 of the following: improvement with defecation, onset associated with a change in frequency of stool, and onset associated with a change in form (appearance) of stool.3 IBS often is unrecognized or untreated, and as few as 25% of patients with IBS seek care.4

IBS-D affects approximately 5% of the general population in North America.5,6 IBS-D is associated with a considerably decreased quality of life and is a common cause of work absenteeism.7,8 Because many conditions can cause diarrhea, patients typically undergo numerous tests before receiving an accurate diagnosis, which creates a financial burden.9

For many patients, current IBS treatments, which include fiber supplements, laxatives, antidiarrheal medications, antispasmodics, and antidepressants such as tricyclics and selective serotonin reuptake inhibitors, are unsatisfactory.10 Alosetron, a 5-hydroxytryptamine 3 (5HT3) receptor antagonist, has been used to treat IBS-D,11 but this medication was voluntarily withdrawn from the US market in 2000 due to concerns of ischemic colitis and severe constipation.12 It was reintroduced in 2002, but can be prescribed only by physicians who enroll in a prescribing program provided by the manufacturer, and the drug has restrictions on its use.

Ondansetron—a different 5HT3 receptor antagonist used to treat nausea and vomiting caused by chemotherapy—may be another option for treating IBS-D. Garsed et al1 recently conducted a RCT to evaluate the efficacy of ondansetron for patients with IBS-D.

STUDY SUMMARY: Ondansetron improves stool consistency, severity of IBS symptoms


In a 5-week, double-blind crossover RCT, Garsed et al1 compared ondansetron vs placebo for symptom relief in 120 patients who met Rome III criteria for IBS-D. All patients were ages 18 to 75 and had no evidence of inflammatory bowel disease. Exclusion criteria were pregnancy or breastfeeding, unwillingness to stop antidiarrheal medication, prior abdominal surgery other than appendectomy or cholecystectomy, or being in another trial. Patients were started on ondansetron 4 mg/d with dose titration up to 24 mg/d based on response; no dose adjustments were allowed during the last 2 weeks of the study. There was a 2- to 3-week washout between treatment periods.

The primary endpoint was average stool consistency in the last 2 weeks of treatment, as measured by the Bristol Stool Form (BSF) scale.13 The BSF is a visual scale that depicts stool as hard (Type 1) to watery (Type 7); types 3 and 4 describe normal stools. The study also looked at urgency and frequency of defecation, bowel transit time, and pain scores.

Treatment with ondansetron resulted in a small but statistically significant improvement in stool consistency. The mean difference in BSF score between ondansetron and placebo was -0.9 (95% confidence interval [CI], -1.1 to -0.6; P<.001), indicating slightly more formed stool with use of ondansetron. The IBS Severity Scoring System score (maximum score 500 points, with mild, moderate, and severe cases indicated by scores of 75-175, 175-300, and >300, respectively) was reduced by more points with ondansetron than placebo (83 ± 9.8 vs 37 ± 9.7; P=.001). Although this mean difference of 46 points fell just short of the 50-point threshold that is considered clinically significant, many patients exceeded this threshold.

For patients with IBS-D, ondansetron reduced frequency of defecation and bloating, but did not relieve pain. Compared to those who received placebo, patients who took ondansetron also had less frequent defecation (P=.002) and lower urgency scores (P<.001). Gut transit time was lengthened in the ondansetron group by 10 hours more than in the placebo group (95% CI, 6-14 hours; P<.001). Pain scores did not change significantly for patients taking ondansetron, although they experienced significantly fewer days of urgency and bloating. Symptoms typically improved in as little as 7 days but returned after stopping ondansetron, typically within 2 weeks. Sixty-five percent of patients reported adequate relief with ondansetron, compared to 14% with placebo.

 

 

Patients whose diarrhea was more severe at baseline didn’t respond as well to ondansetron as did those whose diarrhea was less severe. The only frequent adverse effect was constipation, which occurred in 9% of patients receiving ondansetron and 2% of those on placebo.

WHAT’S NEW: Another option for IBS patients
 with diarrhea

A prior, smaller study of ondansetron that used a lower dosage (12 mg/d) suggested benefit in IBS-D.14 In that study, ondansetron decreased diarrhea and functional dyspepsia. The study by Garsed et al1 is the first large RCT to show significantly improved stool consistency, less frequent defecation, and less urgency and bloating from using ondansetron to treat IBS-D.

CAVEATS: Ondansetron doesn’t appear 
to reduce pain


In Garsed et al,1 patients who received ondansetron did not experience relief from pain, which is one of the main complaints of IBS. However, this study did find slight improvement in formed stools, symptom relief that approached—but did not quite reach—clinical significance, fewer days with urgency and bloating, and less frequent defecation. This study did not evaluate the long-term effects of ondansetron use. However, ondansetron has been used for other indications for more than 25 years and has been reported to have a low risk of adverse effects.15

CHALLENGES TO IMPLEMENTATION: Remember ondansetron 
is not for IBS patients with constipation

Proper use of this drug among patients with IBS is key. The primary benefits of ondansetron are limited to IBS patients who suffer from diarrhea, and not constipation. Ondansetron should not be prescribed to IBS patients who experience constipation, or those with mixed symptoms.

ACKNOWLEDGEMENT
The PURLs Surveillance System was supported in part by Grant Number UL1RR024999 from the National Center For Research Resources, a Clinical Translational Science Award to the University of Chicago. The content is solely the responsibility of the authors and does not necessarily represent the official views of the National Center For Research Resources or the National Institutes of Health.

Click here to view PURL METHODOLOGY

Files
References

 

1. Garsed K, Chernova J, Hastings M, et al. A randomised trial of ondansetron for the treatment of irritable bowel syndrome with diarrhoea. Gut. 2014;63:1617-1625.

2. Hahn BA, Yan S, Strassels S. Impact of irritable bowel syndrome on quality of life and resource use in the United States and United Kingdom. Digestion. 1999;60:77-81.

3. Drossman DA, Dumitrascu DL. Rome III: New standard for functional gastrointestinal disorders. J Gastrointestin Liver Dis. 2006;15:237-241.

4. Luscombe FA. Health-related quality of life and associated psychosocial factors in irritable bowel syndrome: a review. Qual Life Res. 2000;9:161-176.

5. Saito YA, Locke GR, Talley NJ, et al. A comparison of the Rome and Manning criteria for case identification in epidemiological investigations of irritable bowel syndrome. Am J Gastroenterol. 2000;95:2816-2824.

6. Thompson WG, Heaton KW, Smyth GT, et al. Irritable bowel syndrome in general practice: prevalence, characteristics, and referral. Gut. 2000;46:78-82.

7. Tillisch K, Labus JS, Naliboff BD, et al. Characterization of the alternating bowel habit subtype in patients with irritable bowel syndrome. Am J Gastroenterol. 2005;100:896-904.

8. Schuster MM. Diagnostic evaluation of the irritable bowel syndrome. Gastroenterol Clin North Am. 1991;20:269-278.

9. Sandler RS, Everhart JE, Donowitz M, et al. The burden of selected digestive diseases in the United States. Gastroenterology. 2002;122:1500-1511.

10. Talley NJ. Pharmacologic therapy for the irritable bowel syndrome. Am J Gastroenterol. 2003;98:750-758.

11. Andresen V, Montori VM, Keller J, et al. Effects of 5-hydroxytryptamine (serotonin) type 3 antagonists on symptom relief and constipation in nonconstipated irritable bowel syndrome: a systematic review and meta-analysis of randomized controlled trials. Clin Gastroenterol Hepatol. 2008;6:545-555.

12. Chang L, Chey WD, Harris L, et al. Incidence of ischemic colitis and serious complications of constipation among patients using alosetron: systematic review of clinical trials and post-marketing surveillance data. Am J Gastroenterol. 2006;101:1069-1079.

13. Heaton KW, O’Donnell LJ. An office guide to whole-gut transit time. Patients’ recollection of their stool form. J Clin Gastroenterol. 1994;19:28-30.

14. Maxton DG, Morris J, Whorwell PJ. Selective 5‐hydroxytryptamine antagonism: a role in irritable bowel syndrome and functional dyspepsia? Aliment Pharmacol Ther. 1996;10:595-599.

15. Gill SK, Einarson A. The safety of drugs for the treatment of nausea and vomiting of pregnancy. Expert Opin Drug Saf. 2007;6:685-694.

Article PDF
Author and Disclosure Information

 

Robert Levy, MD
Jason Corbo, PharmD, BCPS
Shailendra Prasad, MBBS, MPH

North Memorial
 Family Medicine Residency, University of Minnesota, Minneapolis (Drs. Levy and Prasad); Family Medicine Residency Program, UPMC St. Margaret, Pittsburgh, Pa (Dr. Corbo)

PURLs EDITOR
Bernard Ewigman, MD, MSPH

Department of Family Medicine, The University of Chicago

Issue
The Journal of Family Practice - 63(10)
Publications
Topics
Page Number
600-602
Legacy Keywords
irritable bowel syndrome; ondansetron; IBS; Robert Levy, MD; Jason Corbo, PharmD, BCPS; Shailendra Prasad, MBBS, MPH
Sections
Files
Files
Author and Disclosure Information

 

Robert Levy, MD
Jason Corbo, PharmD, BCPS
Shailendra Prasad, MBBS, MPH

North Memorial
 Family Medicine Residency, University of Minnesota, Minneapolis (Drs. Levy and Prasad); Family Medicine Residency Program, UPMC St. Margaret, Pittsburgh, Pa (Dr. Corbo)

PURLs EDITOR
Bernard Ewigman, MD, MSPH

Department of Family Medicine, The University of Chicago

Author and Disclosure Information

 

Robert Levy, MD
Jason Corbo, PharmD, BCPS
Shailendra Prasad, MBBS, MPH

North Memorial
 Family Medicine Residency, University of Minnesota, Minneapolis (Drs. Levy and Prasad); Family Medicine Residency Program, UPMC St. Margaret, Pittsburgh, Pa (Dr. Corbo)

PURLs EDITOR
Bernard Ewigman, MD, MSPH

Department of Family Medicine, The University of Chicago

Article PDF
Article PDF
Related Articles

 

PRACTICE CHANGER

Consider prescribing ondansetron up to 24 mg/d for patients who have irritable bowel syndrome with diarrhea (IBS-D).1

Strength of recommendation

B: Based on a well-done double-blind, placebo-controlled randomized controlled trial (RCT).

Garsed K, Chernova J, Hastings M, et al. A randomised trial of ondansetron for the treatment of irritable bowel syndrome with diarrhoea. Gut. 2014;63:1617-1625.

Illustrative case

A 23-year-old woman who was diagnosed with irritable bowel syndrome (IBS) comes to your clinic with complaints of increased frequency of defecation with watery stools and generalized, cramping abdominal pain. She also notes increased passage of mucus and a sensation of incomplete evacuation. She says the only thing that relieves her pain is defecation. She has tried loperamide, acetaminophen, and ibuprofen without relief. She does not have Crohn’s disease or ulcerative colitis. What else can you offer her that is safe and effective?

IBS is a chronic, episodic functional gastrointestinal disorder characterized by abdominal pain or discomfort and altered bowel habits (constipation [IBS-C], diarrhea [IBS-D], or alternating periods of both—mixed [IBS-M]).2 It is diagnosed based on Rome III criteria—recurrent abdominal pain or discomfort at least 3 days/month in the last 3 months associated with ≥2 of the following: improvement with defecation, onset associated with a change in frequency of stool, and onset associated with a change in form (appearance) of stool.3 IBS often is unrecognized or untreated, and as few as 25% of patients with IBS seek care.4

IBS-D affects approximately 5% of the general population in North America.5,6 IBS-D is associated with a considerably decreased quality of life and is a common cause of work absenteeism.7,8 Because many conditions can cause diarrhea, patients typically undergo numerous tests before receiving an accurate diagnosis, which creates a financial burden.9

For many patients, current IBS treatments, which include fiber supplements, laxatives, antidiarrheal medications, antispasmodics, and antidepressants such as tricyclics and selective serotonin reuptake inhibitors, are unsatisfactory.10 Alosetron, a 5-hydroxytryptamine 3 (5HT3) receptor antagonist, has been used to treat IBS-D,11 but this medication was voluntarily withdrawn from the US market in 2000 due to concerns of ischemic colitis and severe constipation.12 It was reintroduced in 2002, but can be prescribed only by physicians who enroll in a prescribing program provided by the manufacturer, and the drug has restrictions on its use.

Ondansetron—a different 5HT3 receptor antagonist used to treat nausea and vomiting caused by chemotherapy—may be another option for treating IBS-D. Garsed et al1 recently conducted a RCT to evaluate the efficacy of ondansetron for patients with IBS-D.

STUDY SUMMARY: Ondansetron improves stool consistency, severity of IBS symptoms


In a 5-week, double-blind crossover RCT, Garsed et al1 compared ondansetron vs placebo for symptom relief in 120 patients who met Rome III criteria for IBS-D. All patients were ages 18 to 75 and had no evidence of inflammatory bowel disease. Exclusion criteria were pregnancy or breastfeeding, unwillingness to stop antidiarrheal medication, prior abdominal surgery other than appendectomy or cholecystectomy, or being in another trial. Patients were started on ondansetron 4 mg/d with dose titration up to 24 mg/d based on response; no dose adjustments were allowed during the last 2 weeks of the study. There was a 2- to 3-week washout between treatment periods.

The primary endpoint was average stool consistency in the last 2 weeks of treatment, as measured by the Bristol Stool Form (BSF) scale.13 The BSF is a visual scale that depicts stool as hard (Type 1) to watery (Type 7); types 3 and 4 describe normal stools. The study also looked at urgency and frequency of defecation, bowel transit time, and pain scores.

Treatment with ondansetron resulted in a small but statistically significant improvement in stool consistency. The mean difference in BSF score between ondansetron and placebo was -0.9 (95% confidence interval [CI], -1.1 to -0.6; P<.001), indicating slightly more formed stool with use of ondansetron. The IBS Severity Scoring System score (maximum score 500 points, with mild, moderate, and severe cases indicated by scores of 75-175, 175-300, and >300, respectively) was reduced by more points with ondansetron than placebo (83 ± 9.8 vs 37 ± 9.7; P=.001). Although this mean difference of 46 points fell just short of the 50-point threshold that is considered clinically significant, many patients exceeded this threshold.

For patients with IBS-D, ondansetron reduced frequency of defecation and bloating, but did not relieve pain. Compared to those who received placebo, patients who took ondansetron also had less frequent defecation (P=.002) and lower urgency scores (P<.001). Gut transit time was lengthened in the ondansetron group by 10 hours more than in the placebo group (95% CI, 6-14 hours; P<.001). Pain scores did not change significantly for patients taking ondansetron, although they experienced significantly fewer days of urgency and bloating. Symptoms typically improved in as little as 7 days but returned after stopping ondansetron, typically within 2 weeks. Sixty-five percent of patients reported adequate relief with ondansetron, compared to 14% with placebo.

 

 

Patients whose diarrhea was more severe at baseline didn’t respond as well to ondansetron as did those whose diarrhea was less severe. The only frequent adverse effect was constipation, which occurred in 9% of patients receiving ondansetron and 2% of those on placebo.

WHAT’S NEW: Another option for IBS patients
 with diarrhea

A prior, smaller study of ondansetron that used a lower dosage (12 mg/d) suggested benefit in IBS-D.14 In that study, ondansetron decreased diarrhea and functional dyspepsia. The study by Garsed et al1 is the first large RCT to show significantly improved stool consistency, less frequent defecation, and less urgency and bloating from using ondansetron to treat IBS-D.

CAVEATS: Ondansetron doesn’t appear 
to reduce pain


In Garsed et al,1 patients who received ondansetron did not experience relief from pain, which is one of the main complaints of IBS. However, this study did find slight improvement in formed stools, symptom relief that approached—but did not quite reach—clinical significance, fewer days with urgency and bloating, and less frequent defecation. This study did not evaluate the long-term effects of ondansetron use. However, ondansetron has been used for other indications for more than 25 years and has been reported to have a low risk of adverse effects.15

CHALLENGES TO IMPLEMENTATION: Remember ondansetron 
is not for IBS patients with constipation

Proper use of this drug among patients with IBS is key. The primary benefits of ondansetron are limited to IBS patients who suffer from diarrhea, and not constipation. Ondansetron should not be prescribed to IBS patients who experience constipation, or those with mixed symptoms.

ACKNOWLEDGEMENT
The PURLs Surveillance System was supported in part by Grant Number UL1RR024999 from the National Center For Research Resources, a Clinical Translational Science Award to the University of Chicago. The content is solely the responsibility of the authors and does not necessarily represent the official views of the National Center For Research Resources or the National Institutes of Health.

Click here to view PURL METHODOLOGY

 

PRACTICE CHANGER

Consider prescribing ondansetron up to 24 mg/d for patients who have irritable bowel syndrome with diarrhea (IBS-D).1

Strength of recommendation

B: Based on a well-done double-blind, placebo-controlled randomized controlled trial (RCT).

Garsed K, Chernova J, Hastings M, et al. A randomised trial of ondansetron for the treatment of irritable bowel syndrome with diarrhoea. Gut. 2014;63:1617-1625.

Illustrative case

A 23-year-old woman who was diagnosed with irritable bowel syndrome (IBS) comes to your clinic with complaints of increased frequency of defecation with watery stools and generalized, cramping abdominal pain. She also notes increased passage of mucus and a sensation of incomplete evacuation. She says the only thing that relieves her pain is defecation. She has tried loperamide, acetaminophen, and ibuprofen without relief. She does not have Crohn’s disease or ulcerative colitis. What else can you offer her that is safe and effective?

IBS is a chronic, episodic functional gastrointestinal disorder characterized by abdominal pain or discomfort and altered bowel habits (constipation [IBS-C], diarrhea [IBS-D], or alternating periods of both—mixed [IBS-M]).2 It is diagnosed based on Rome III criteria—recurrent abdominal pain or discomfort at least 3 days/month in the last 3 months associated with ≥2 of the following: improvement with defecation, onset associated with a change in frequency of stool, and onset associated with a change in form (appearance) of stool.3 IBS often is unrecognized or untreated, and as few as 25% of patients with IBS seek care.4

IBS-D affects approximately 5% of the general population in North America.5,6 IBS-D is associated with a considerably decreased quality of life and is a common cause of work absenteeism.7,8 Because many conditions can cause diarrhea, patients typically undergo numerous tests before receiving an accurate diagnosis, which creates a financial burden.9

For many patients, current IBS treatments, which include fiber supplements, laxatives, antidiarrheal medications, antispasmodics, and antidepressants such as tricyclics and selective serotonin reuptake inhibitors, are unsatisfactory.10 Alosetron, a 5-hydroxytryptamine 3 (5HT3) receptor antagonist, has been used to treat IBS-D,11 but this medication was voluntarily withdrawn from the US market in 2000 due to concerns of ischemic colitis and severe constipation.12 It was reintroduced in 2002, but can be prescribed only by physicians who enroll in a prescribing program provided by the manufacturer, and the drug has restrictions on its use.

Ondansetron—a different 5HT3 receptor antagonist used to treat nausea and vomiting caused by chemotherapy—may be another option for treating IBS-D. Garsed et al1 recently conducted a RCT to evaluate the efficacy of ondansetron for patients with IBS-D.

STUDY SUMMARY: Ondansetron improves stool consistency, severity of IBS symptoms


In a 5-week, double-blind crossover RCT, Garsed et al1 compared ondansetron vs placebo for symptom relief in 120 patients who met Rome III criteria for IBS-D. All patients were ages 18 to 75 and had no evidence of inflammatory bowel disease. Exclusion criteria were pregnancy or breastfeeding, unwillingness to stop antidiarrheal medication, prior abdominal surgery other than appendectomy or cholecystectomy, or being in another trial. Patients were started on ondansetron 4 mg/d with dose titration up to 24 mg/d based on response; no dose adjustments were allowed during the last 2 weeks of the study. There was a 2- to 3-week washout between treatment periods.

The primary endpoint was average stool consistency in the last 2 weeks of treatment, as measured by the Bristol Stool Form (BSF) scale.13 The BSF is a visual scale that depicts stool as hard (Type 1) to watery (Type 7); types 3 and 4 describe normal stools. The study also looked at urgency and frequency of defecation, bowel transit time, and pain scores.

Treatment with ondansetron resulted in a small but statistically significant improvement in stool consistency. The mean difference in BSF score between ondansetron and placebo was -0.9 (95% confidence interval [CI], -1.1 to -0.6; P<.001), indicating slightly more formed stool with use of ondansetron. The IBS Severity Scoring System score (maximum score 500 points, with mild, moderate, and severe cases indicated by scores of 75-175, 175-300, and >300, respectively) was reduced by more points with ondansetron than placebo (83 ± 9.8 vs 37 ± 9.7; P=.001). Although this mean difference of 46 points fell just short of the 50-point threshold that is considered clinically significant, many patients exceeded this threshold.

For patients with IBS-D, ondansetron reduced frequency of defecation and bloating, but did not relieve pain. Compared to those who received placebo, patients who took ondansetron also had less frequent defecation (P=.002) and lower urgency scores (P<.001). Gut transit time was lengthened in the ondansetron group by 10 hours more than in the placebo group (95% CI, 6-14 hours; P<.001). Pain scores did not change significantly for patients taking ondansetron, although they experienced significantly fewer days of urgency and bloating. Symptoms typically improved in as little as 7 days but returned after stopping ondansetron, typically within 2 weeks. Sixty-five percent of patients reported adequate relief with ondansetron, compared to 14% with placebo.

 

 

Patients whose diarrhea was more severe at baseline didn’t respond as well to ondansetron as did those whose diarrhea was less severe. The only frequent adverse effect was constipation, which occurred in 9% of patients receiving ondansetron and 2% of those on placebo.

WHAT’S NEW: Another option for IBS patients
 with diarrhea

A prior, smaller study of ondansetron that used a lower dosage (12 mg/d) suggested benefit in IBS-D.14 In that study, ondansetron decreased diarrhea and functional dyspepsia. The study by Garsed et al1 is the first large RCT to show significantly improved stool consistency, less frequent defecation, and less urgency and bloating from using ondansetron to treat IBS-D.

CAVEATS: Ondansetron doesn’t appear 
to reduce pain


In Garsed et al,1 patients who received ondansetron did not experience relief from pain, which is one of the main complaints of IBS. However, this study did find slight improvement in formed stools, symptom relief that approached—but did not quite reach—clinical significance, fewer days with urgency and bloating, and less frequent defecation. This study did not evaluate the long-term effects of ondansetron use. However, ondansetron has been used for other indications for more than 25 years and has been reported to have a low risk of adverse effects.15

CHALLENGES TO IMPLEMENTATION: Remember ondansetron 
is not for IBS patients with constipation

Proper use of this drug among patients with IBS is key. The primary benefits of ondansetron are limited to IBS patients who suffer from diarrhea, and not constipation. Ondansetron should not be prescribed to IBS patients who experience constipation, or those with mixed symptoms.

ACKNOWLEDGEMENT
The PURLs Surveillance System was supported in part by Grant Number UL1RR024999 from the National Center For Research Resources, a Clinical Translational Science Award to the University of Chicago. The content is solely the responsibility of the authors and does not necessarily represent the official views of the National Center For Research Resources or the National Institutes of Health.

Click here to view PURL METHODOLOGY

References

 

1. Garsed K, Chernova J, Hastings M, et al. A randomised trial of ondansetron for the treatment of irritable bowel syndrome with diarrhoea. Gut. 2014;63:1617-1625.

2. Hahn BA, Yan S, Strassels S. Impact of irritable bowel syndrome on quality of life and resource use in the United States and United Kingdom. Digestion. 1999;60:77-81.

3. Drossman DA, Dumitrascu DL. Rome III: New standard for functional gastrointestinal disorders. J Gastrointestin Liver Dis. 2006;15:237-241.

4. Luscombe FA. Health-related quality of life and associated psychosocial factors in irritable bowel syndrome: a review. Qual Life Res. 2000;9:161-176.

5. Saito YA, Locke GR, Talley NJ, et al. A comparison of the Rome and Manning criteria for case identification in epidemiological investigations of irritable bowel syndrome. Am J Gastroenterol. 2000;95:2816-2824.

6. Thompson WG, Heaton KW, Smyth GT, et al. Irritable bowel syndrome in general practice: prevalence, characteristics, and referral. Gut. 2000;46:78-82.

7. Tillisch K, Labus JS, Naliboff BD, et al. Characterization of the alternating bowel habit subtype in patients with irritable bowel syndrome. Am J Gastroenterol. 2005;100:896-904.

8. Schuster MM. Diagnostic evaluation of the irritable bowel syndrome. Gastroenterol Clin North Am. 1991;20:269-278.

9. Sandler RS, Everhart JE, Donowitz M, et al. The burden of selected digestive diseases in the United States. Gastroenterology. 2002;122:1500-1511.

10. Talley NJ. Pharmacologic therapy for the irritable bowel syndrome. Am J Gastroenterol. 2003;98:750-758.

11. Andresen V, Montori VM, Keller J, et al. Effects of 5-hydroxytryptamine (serotonin) type 3 antagonists on symptom relief and constipation in nonconstipated irritable bowel syndrome: a systematic review and meta-analysis of randomized controlled trials. Clin Gastroenterol Hepatol. 2008;6:545-555.

12. Chang L, Chey WD, Harris L, et al. Incidence of ischemic colitis and serious complications of constipation among patients using alosetron: systematic review of clinical trials and post-marketing surveillance data. Am J Gastroenterol. 2006;101:1069-1079.

13. Heaton KW, O’Donnell LJ. An office guide to whole-gut transit time. Patients’ recollection of their stool form. J Clin Gastroenterol. 1994;19:28-30.

14. Maxton DG, Morris J, Whorwell PJ. Selective 5‐hydroxytryptamine antagonism: a role in irritable bowel syndrome and functional dyspepsia? Aliment Pharmacol Ther. 1996;10:595-599.

15. Gill SK, Einarson A. The safety of drugs for the treatment of nausea and vomiting of pregnancy. Expert Opin Drug Saf. 2007;6:685-694.

References

 

1. Garsed K, Chernova J, Hastings M, et al. A randomised trial of ondansetron for the treatment of irritable bowel syndrome with diarrhoea. Gut. 2014;63:1617-1625.

2. Hahn BA, Yan S, Strassels S. Impact of irritable bowel syndrome on quality of life and resource use in the United States and United Kingdom. Digestion. 1999;60:77-81.

3. Drossman DA, Dumitrascu DL. Rome III: New standard for functional gastrointestinal disorders. J Gastrointestin Liver Dis. 2006;15:237-241.

4. Luscombe FA. Health-related quality of life and associated psychosocial factors in irritable bowel syndrome: a review. Qual Life Res. 2000;9:161-176.

5. Saito YA, Locke GR, Talley NJ, et al. A comparison of the Rome and Manning criteria for case identification in epidemiological investigations of irritable bowel syndrome. Am J Gastroenterol. 2000;95:2816-2824.

6. Thompson WG, Heaton KW, Smyth GT, et al. Irritable bowel syndrome in general practice: prevalence, characteristics, and referral. Gut. 2000;46:78-82.

7. Tillisch K, Labus JS, Naliboff BD, et al. Characterization of the alternating bowel habit subtype in patients with irritable bowel syndrome. Am J Gastroenterol. 2005;100:896-904.

8. Schuster MM. Diagnostic evaluation of the irritable bowel syndrome. Gastroenterol Clin North Am. 1991;20:269-278.

9. Sandler RS, Everhart JE, Donowitz M, et al. The burden of selected digestive diseases in the United States. Gastroenterology. 2002;122:1500-1511.

10. Talley NJ. Pharmacologic therapy for the irritable bowel syndrome. Am J Gastroenterol. 2003;98:750-758.

11. Andresen V, Montori VM, Keller J, et al. Effects of 5-hydroxytryptamine (serotonin) type 3 antagonists on symptom relief and constipation in nonconstipated irritable bowel syndrome: a systematic review and meta-analysis of randomized controlled trials. Clin Gastroenterol Hepatol. 2008;6:545-555.

12. Chang L, Chey WD, Harris L, et al. Incidence of ischemic colitis and serious complications of constipation among patients using alosetron: systematic review of clinical trials and post-marketing surveillance data. Am J Gastroenterol. 2006;101:1069-1079.

13. Heaton KW, O’Donnell LJ. An office guide to whole-gut transit time. Patients’ recollection of their stool form. J Clin Gastroenterol. 1994;19:28-30.

14. Maxton DG, Morris J, Whorwell PJ. Selective 5‐hydroxytryptamine antagonism: a role in irritable bowel syndrome and functional dyspepsia? Aliment Pharmacol Ther. 1996;10:595-599.

15. Gill SK, Einarson A. The safety of drugs for the treatment of nausea and vomiting of pregnancy. Expert Opin Drug Saf. 2007;6:685-694.

Issue
The Journal of Family Practice - 63(10)
Issue
The Journal of Family Practice - 63(10)
Page Number
600-602
Page Number
600-602
Publications
Publications
Topics
Article Type
Display Headline
An antiemetic for irritable bowel syndrome?
Display Headline
An antiemetic for irritable bowel syndrome?
Legacy Keywords
irritable bowel syndrome; ondansetron; IBS; Robert Levy, MD; Jason Corbo, PharmD, BCPS; Shailendra Prasad, MBBS, MPH
Legacy Keywords
irritable bowel syndrome; ondansetron; IBS; Robert Levy, MD; Jason Corbo, PharmD, BCPS; Shailendra Prasad, MBBS, MPH
Sections
PURLs Copyright

Copyright © 2014 Family Physicians Inquiries Network. All rights reserved.

Disallow All Ads
Alternative CME
Use ProPublica
Hide sidebar & use full width
render the right sidebar.
Conference Recap Checkbox
Not Conference Recap
Clinical Edge
Display the Slideshow in this Article
Article PDF Media
Media Files

Surgery for Persistent Knee Pain? Not So Fast

Article Type
Changed
Tue, 12/13/2016 - 12:08
Display Headline
Surgery for Persistent Knee Pain? Not So Fast
For patients with knee pain from a torn medial meniscus but no osteoarthritis, arthroscopic partial meniscectomy may not be necessary.

PRACTICE CHANGER
Do not refer patients with a degenerative medial meniscus tear for arthroscopic partial meniscectomy, because surgery outcomes are no better than those of conservative treatment.1

Strength of ­recommendation
B: Based on a single high-quality randomized controlled trial.1

Illustrative case
A 40-year-old man comes to your office for follow-up of medial left knee pain he’s had for three months that hasn’t responded to conservative treatment. The pain developed gradually, without a history of trauma. The patient has no signs of degenerative joint disease on x-ray, but MRI reveals a tear of the medial meniscus. Should you refer him for meniscectomy?

Study summary>>

 

 

Patients and doctors alike tend to look for a treatment that will “fix” the problem, which may be why we have continued to use arthroscopic partial meniscectomy to attempt to relieve symptoms of meniscal tears despite a lack of evidence to support the practice.

Guidelines from the American Academy of Orthopaedic Surgeons state that the evidence for medial meniscectomy in patients with a torn meniscus and osteoarthritis (OA) is inconclusive; the organization offers no guidelines for patients with a torn meniscus who don’t have OA.2 The American College of Occupational and Environmental Medicine states that there is insufficient evidence to support arthroscopic partial meniscectomy for symptomatic, torn medial menisci for select patients and “the vast majority of patients [with medial meniscal tears] do not require surgery.”3 Previous studies have concluded that arthroscopic surgery for OA of the knee provides no additional benefit to optimized physical and medical therapy.4 Furthermore, research by Katz et al5 shows that meniscectomy provides no benefit over conservative treatment in functional status at six months in patients with OA and a medial meniscal tear.

That said, arthroscopic partial meniscectomy is still the most common orthopedic procedure in the United States.1 Although its use has decreased in the past 15 years, it is performed nearly 700,000 times annually at a cost of approximately $4 billion.1,6,7 Like any surgical procedure, meniscectomy carries a risk for complications. In the double-blind, randomized trial reported on here, Sihvonen et al1 compared meniscectomy to a sham procedure for patients with knee pain but not OA.

STUDY SUMMARY
Meniscectomy and sham­surgery are equally effective

Sihvonen et al1 conducted a randomized, double-blind, sham-controlled trial at five orthopedic clinics in Finland. Patients ages 35 to 65 were enrolled if they had clinical findings of a medial meniscus tear and knee pain for more than three months that wasn’t relieved by conservative treatment. The trial excluded patients who had an obvious traumatic onset of symptoms; clinical or radiologic evidence of knee OA; a locked knee that could not be straightened; knee instability or decreased range of motion; previous surgery on the affected knee; fracture within the past 12 months on the affected limb; or other notable pathology on MRI or during arthroscopy.

Before randomization, 160 patients underwent diagnostic arthroscopy. Fourteen patients were excluded: six because they did not actually have a medial meniscal tear, one because he also had a lateral meniscus tear, three due to a major chondral flap, two who had already undergone meniscal repair, and two due to an osteochondral microfracture.

At the end of the diagnostic arthroscopy, each patient was blindly randomized to arthroscopic partial meniscectomy or sham surgery. To simulate the meniscectomy procedure, the surgeon similarly manipulated the knee, made comparable noise and vibration using tools and suction, and ensured that the patient was kept in the operating room (OR) for a comparable time. Only the orthopedic surgeon and OR staff were aware of which surgery the patient underwent, and these staff members were not included in further treatment or follow-up. After the procedure, all patients received the same walking aids and instructions for a graduated exercise program.

The 70 patients in the meniscectomy group and the 76 in the sham surgery group were similar in age (mean: 52 years), sex, BMI, and duration of pain (mean: 10 months). Patients in both groups also had similar tears noted on arthroscopy.

Three primary outcomes were measured before surgery and at 12 months: knee pain, knee symptoms and function, and quality of life (QoL). Knee pain after exercise was evaluated on a scale of 0 to 10, with 0 indicating no pain. The validated Lysholm knee score was used to assess knee symptoms and function, and the Western Ontario Meniscal Evaluation Tool (WOMET) was utilized to evaluate QoL; both are 100-point scales in which lower scores indicate more severe symptoms.

Both groups had marked improvement in pain and function from baseline to 12 months, and there was no significant difference between the two groups. Knee pain scores improved by 3.1 points in the meniscectomy group and 3.3 points in the sham surgery group.
 Lysholm symptom and function scores improved 21.7 points in the meniscectomy group and 23.3 points in the sham surgery group (a change of 11.5 points would have been considered clinically significant). The mean between-group difference was –1.6 points.

WOMET QoL scores improved 24.6 points in the meniscectomy group and 27.1 points in the sham surgery group (a change of 15.5 points would have been considered clinically significant). The mean between-group difference was –2.5 points.

There were no significant between-group differences in serious adverse events or number of patients who required subsequent knee surgery. Similar proportions in each group thought they had sham surgery, which confirmed the effectiveness of the blinding. Ninety-six percent of patients in the sham procedure group and 93% in the meniscectomy group reported they would be willing to repeat the procedure.

What's new and challenges to implementation >>

 

 


WHAT’S NEW
Recommend physical therapy,
exercise instead of surgery
Previous studies of arthroscopic partial meniscectomy to treat degenerative meniscal tears in patients with knee OA found no benefit.6,8 This study specifically examined patients without OA and found arthroscopic partial meniscectomy offered no benefit over sham surgery.

In addition to fewer referrals for meniscectomy, these findings could lead to another change in practice: Clinicians may be less likely to order MRI to confirm the diagnosis of a medial meniscal tear, since doing so will not change their therapeutic approach. This approach centers on recommending that patients with a degenerative meniscal tear start and stick with physical therapy and their designated exercise regimen.

CAVEATS
Surgery might be effective for more active patients

This study, as well as previous research, did not look at surgery for an acute medial meniscus tear following a traumatic incident, such as a fall or direct blow. Additionally, these results are based on improved outcomes in activities of daily living, and may not extend to patients who engage in high-level functioning, such as sports or strenuous work. The sham surgery group received lavage, which could be considered an active treatment, although a previous trial found lavage had no benefit over conservative treatment in patients with knee OA.4

CHALLENGES TO IMPLEMENTATION
It might be hard to convince patients they don’t need surgery

Some patients expect immediate intervention with surgery. It may be difficult to convince such patients that active participation in physical therapy can lead to the same outcomes as surgery. Spending time with your patient to explain the injury, what happens during surgery, and the evidence that shows a lack of difference in outcomes can lead to fewer surgeries. Most patients and clinicians will want to do an MRI after three months of persistent pain to determine the diagnosis, although some may be comfortable with continuing conservative treatment.

References
1. Sihvonen R, Paavola M, Malmivaara A, et al; Finnish Degenerative Meniscal Lesion Study (FIDELITY) Group. Arthroscopic partial meniscectomy versus sham surgery for a degenerative meniscal tear. N Engl J Med. 2013; 369:2515-2524.

2. American Academy of Orthopaedic Surgeons. Treatment of Osteoarthritis of the Knee. Evidence-Based Guideline. 2nd ed. Rosemont, IL: American Academy of Orthopaedic Surgeons; 2013.

3. Knee disorders. In: Hegmann KT, ed. Occupational Medicine Practice Guidelines. Evaluation and Management of Common Health Problems and Functional Recovery in Workers. 3rd ed. Elk Grove Village, IL: American College of Occupational and Environmental Medicine; 2011:1-503.

4. Kirkley A, Birmingham TB, Litchfield RB, et al. A randomized trial for arthroscopic surgery for osteoarthritis of the knee. N Engl J Med. 2008; 359:1097-1107.

5. Katz JN, Brophy RH, Chaisson CE, et al. Surgery versus physical therapy for a meniscal tear and osteoarthritis. N Engl J Med. 2013;368:1675-1684.

6. Cullen KA, Hall MJ, Golosinskiy A. Ambulatory surgery in the United States, 2006. Natl Health State Report. 2009;11:1-25.

7. Salzler MJ, Lin A, Miller CD, et al. Complications after arthroscopic knee surgery. Am J Sports Med. 2014;42:292-296.

8. Moseley JB, O’Malley K, Petersen NJ, et al. A controlled trial of arthroscopic surgery for osteoarthritis of the knee. N Engl J Med. 2002; 347:81-88.

Acknowledgement
The PURLs Surveillance System was supported in part by Grant Number UL1RR024999 from the National Center For Research Resources, a Clinical Translational Science Award to the University of Chicago. The content is solely the responsibility of the authors and does not necessarily represent the official views of the National Center For Research Resources or the National Institutes of Health.

Copyright © 2014. The Family Physicians Inquiries Network. All rights reserved.

Reprinted with permission from the Family Physicians Inquiries Network and The Journal of Family Practice. 2014;63(9):534-536.

References

Article PDF
Author and Disclosure Information

Mallory McClester Brown, MD, Anne Mounsey, MD
Mallory McClester Brown and Anne Mounsey are in the Department of Family Medicine at the University of North Carolina at Chapel Hill.

Issue
Clinician Reviews - 24(10)
Publications
Topics
Page Number
49,52,55
Legacy Keywords
knee pain, torn medial meniscus, medial meniscal tear, arthroscopic partial meniscectomy, medial meniscectomy
Sections
Author and Disclosure Information

Mallory McClester Brown, MD, Anne Mounsey, MD
Mallory McClester Brown and Anne Mounsey are in the Department of Family Medicine at the University of North Carolina at Chapel Hill.

Author and Disclosure Information

Mallory McClester Brown, MD, Anne Mounsey, MD
Mallory McClester Brown and Anne Mounsey are in the Department of Family Medicine at the University of North Carolina at Chapel Hill.

Article PDF
Article PDF
For patients with knee pain from a torn medial meniscus but no osteoarthritis, arthroscopic partial meniscectomy may not be necessary.
For patients with knee pain from a torn medial meniscus but no osteoarthritis, arthroscopic partial meniscectomy may not be necessary.

PRACTICE CHANGER
Do not refer patients with a degenerative medial meniscus tear for arthroscopic partial meniscectomy, because surgery outcomes are no better than those of conservative treatment.1

Strength of ­recommendation
B: Based on a single high-quality randomized controlled trial.1

Illustrative case
A 40-year-old man comes to your office for follow-up of medial left knee pain he’s had for three months that hasn’t responded to conservative treatment. The pain developed gradually, without a history of trauma. The patient has no signs of degenerative joint disease on x-ray, but MRI reveals a tear of the medial meniscus. Should you refer him for meniscectomy?

Study summary>>

 

 

Patients and doctors alike tend to look for a treatment that will “fix” the problem, which may be why we have continued to use arthroscopic partial meniscectomy to attempt to relieve symptoms of meniscal tears despite a lack of evidence to support the practice.

Guidelines from the American Academy of Orthopaedic Surgeons state that the evidence for medial meniscectomy in patients with a torn meniscus and osteoarthritis (OA) is inconclusive; the organization offers no guidelines for patients with a torn meniscus who don’t have OA.2 The American College of Occupational and Environmental Medicine states that there is insufficient evidence to support arthroscopic partial meniscectomy for symptomatic, torn medial menisci for select patients and “the vast majority of patients [with medial meniscal tears] do not require surgery.”3 Previous studies have concluded that arthroscopic surgery for OA of the knee provides no additional benefit to optimized physical and medical therapy.4 Furthermore, research by Katz et al5 shows that meniscectomy provides no benefit over conservative treatment in functional status at six months in patients with OA and a medial meniscal tear.

That said, arthroscopic partial meniscectomy is still the most common orthopedic procedure in the United States.1 Although its use has decreased in the past 15 years, it is performed nearly 700,000 times annually at a cost of approximately $4 billion.1,6,7 Like any surgical procedure, meniscectomy carries a risk for complications. In the double-blind, randomized trial reported on here, Sihvonen et al1 compared meniscectomy to a sham procedure for patients with knee pain but not OA.

STUDY SUMMARY
Meniscectomy and sham­surgery are equally effective

Sihvonen et al1 conducted a randomized, double-blind, sham-controlled trial at five orthopedic clinics in Finland. Patients ages 35 to 65 were enrolled if they had clinical findings of a medial meniscus tear and knee pain for more than three months that wasn’t relieved by conservative treatment. The trial excluded patients who had an obvious traumatic onset of symptoms; clinical or radiologic evidence of knee OA; a locked knee that could not be straightened; knee instability or decreased range of motion; previous surgery on the affected knee; fracture within the past 12 months on the affected limb; or other notable pathology on MRI or during arthroscopy.

Before randomization, 160 patients underwent diagnostic arthroscopy. Fourteen patients were excluded: six because they did not actually have a medial meniscal tear, one because he also had a lateral meniscus tear, three due to a major chondral flap, two who had already undergone meniscal repair, and two due to an osteochondral microfracture.

At the end of the diagnostic arthroscopy, each patient was blindly randomized to arthroscopic partial meniscectomy or sham surgery. To simulate the meniscectomy procedure, the surgeon similarly manipulated the knee, made comparable noise and vibration using tools and suction, and ensured that the patient was kept in the operating room (OR) for a comparable time. Only the orthopedic surgeon and OR staff were aware of which surgery the patient underwent, and these staff members were not included in further treatment or follow-up. After the procedure, all patients received the same walking aids and instructions for a graduated exercise program.

The 70 patients in the meniscectomy group and the 76 in the sham surgery group were similar in age (mean: 52 years), sex, BMI, and duration of pain (mean: 10 months). Patients in both groups also had similar tears noted on arthroscopy.

Three primary outcomes were measured before surgery and at 12 months: knee pain, knee symptoms and function, and quality of life (QoL). Knee pain after exercise was evaluated on a scale of 0 to 10, with 0 indicating no pain. The validated Lysholm knee score was used to assess knee symptoms and function, and the Western Ontario Meniscal Evaluation Tool (WOMET) was utilized to evaluate QoL; both are 100-point scales in which lower scores indicate more severe symptoms.

Both groups had marked improvement in pain and function from baseline to 12 months, and there was no significant difference between the two groups. Knee pain scores improved by 3.1 points in the meniscectomy group and 3.3 points in the sham surgery group.
 Lysholm symptom and function scores improved 21.7 points in the meniscectomy group and 23.3 points in the sham surgery group (a change of 11.5 points would have been considered clinically significant). The mean between-group difference was –1.6 points.

WOMET QoL scores improved 24.6 points in the meniscectomy group and 27.1 points in the sham surgery group (a change of 15.5 points would have been considered clinically significant). The mean between-group difference was –2.5 points.

There were no significant between-group differences in serious adverse events or number of patients who required subsequent knee surgery. Similar proportions in each group thought they had sham surgery, which confirmed the effectiveness of the blinding. Ninety-six percent of patients in the sham procedure group and 93% in the meniscectomy group reported they would be willing to repeat the procedure.

What's new and challenges to implementation >>

 

 


WHAT’S NEW
Recommend physical therapy,
exercise instead of surgery
Previous studies of arthroscopic partial meniscectomy to treat degenerative meniscal tears in patients with knee OA found no benefit.6,8 This study specifically examined patients without OA and found arthroscopic partial meniscectomy offered no benefit over sham surgery.

In addition to fewer referrals for meniscectomy, these findings could lead to another change in practice: Clinicians may be less likely to order MRI to confirm the diagnosis of a medial meniscal tear, since doing so will not change their therapeutic approach. This approach centers on recommending that patients with a degenerative meniscal tear start and stick with physical therapy and their designated exercise regimen.

CAVEATS
Surgery might be effective for more active patients

This study, as well as previous research, did not look at surgery for an acute medial meniscus tear following a traumatic incident, such as a fall or direct blow. Additionally, these results are based on improved outcomes in activities of daily living, and may not extend to patients who engage in high-level functioning, such as sports or strenuous work. The sham surgery group received lavage, which could be considered an active treatment, although a previous trial found lavage had no benefit over conservative treatment in patients with knee OA.4

CHALLENGES TO IMPLEMENTATION
It might be hard to convince patients they don’t need surgery

Some patients expect immediate intervention with surgery. It may be difficult to convince such patients that active participation in physical therapy can lead to the same outcomes as surgery. Spending time with your patient to explain the injury, what happens during surgery, and the evidence that shows a lack of difference in outcomes can lead to fewer surgeries. Most patients and clinicians will want to do an MRI after three months of persistent pain to determine the diagnosis, although some may be comfortable with continuing conservative treatment.

References
1. Sihvonen R, Paavola M, Malmivaara A, et al; Finnish Degenerative Meniscal Lesion Study (FIDELITY) Group. Arthroscopic partial meniscectomy versus sham surgery for a degenerative meniscal tear. N Engl J Med. 2013; 369:2515-2524.

2. American Academy of Orthopaedic Surgeons. Treatment of Osteoarthritis of the Knee. Evidence-Based Guideline. 2nd ed. Rosemont, IL: American Academy of Orthopaedic Surgeons; 2013.

3. Knee disorders. In: Hegmann KT, ed. Occupational Medicine Practice Guidelines. Evaluation and Management of Common Health Problems and Functional Recovery in Workers. 3rd ed. Elk Grove Village, IL: American College of Occupational and Environmental Medicine; 2011:1-503.

4. Kirkley A, Birmingham TB, Litchfield RB, et al. A randomized trial for arthroscopic surgery for osteoarthritis of the knee. N Engl J Med. 2008; 359:1097-1107.

5. Katz JN, Brophy RH, Chaisson CE, et al. Surgery versus physical therapy for a meniscal tear and osteoarthritis. N Engl J Med. 2013;368:1675-1684.

6. Cullen KA, Hall MJ, Golosinskiy A. Ambulatory surgery in the United States, 2006. Natl Health State Report. 2009;11:1-25.

7. Salzler MJ, Lin A, Miller CD, et al. Complications after arthroscopic knee surgery. Am J Sports Med. 2014;42:292-296.

8. Moseley JB, O’Malley K, Petersen NJ, et al. A controlled trial of arthroscopic surgery for osteoarthritis of the knee. N Engl J Med. 2002; 347:81-88.

Acknowledgement
The PURLs Surveillance System was supported in part by Grant Number UL1RR024999 from the National Center For Research Resources, a Clinical Translational Science Award to the University of Chicago. The content is solely the responsibility of the authors and does not necessarily represent the official views of the National Center For Research Resources or the National Institutes of Health.

Copyright © 2014. The Family Physicians Inquiries Network. All rights reserved.

Reprinted with permission from the Family Physicians Inquiries Network and The Journal of Family Practice. 2014;63(9):534-536.

PRACTICE CHANGER
Do not refer patients with a degenerative medial meniscus tear for arthroscopic partial meniscectomy, because surgery outcomes are no better than those of conservative treatment.1

Strength of ­recommendation
B: Based on a single high-quality randomized controlled trial.1

Illustrative case
A 40-year-old man comes to your office for follow-up of medial left knee pain he’s had for three months that hasn’t responded to conservative treatment. The pain developed gradually, without a history of trauma. The patient has no signs of degenerative joint disease on x-ray, but MRI reveals a tear of the medial meniscus. Should you refer him for meniscectomy?

Study summary>>

 

 

Patients and doctors alike tend to look for a treatment that will “fix” the problem, which may be why we have continued to use arthroscopic partial meniscectomy to attempt to relieve symptoms of meniscal tears despite a lack of evidence to support the practice.

Guidelines from the American Academy of Orthopaedic Surgeons state that the evidence for medial meniscectomy in patients with a torn meniscus and osteoarthritis (OA) is inconclusive; the organization offers no guidelines for patients with a torn meniscus who don’t have OA.2 The American College of Occupational and Environmental Medicine states that there is insufficient evidence to support arthroscopic partial meniscectomy for symptomatic, torn medial menisci for select patients and “the vast majority of patients [with medial meniscal tears] do not require surgery.”3 Previous studies have concluded that arthroscopic surgery for OA of the knee provides no additional benefit to optimized physical and medical therapy.4 Furthermore, research by Katz et al5 shows that meniscectomy provides no benefit over conservative treatment in functional status at six months in patients with OA and a medial meniscal tear.

That said, arthroscopic partial meniscectomy is still the most common orthopedic procedure in the United States.1 Although its use has decreased in the past 15 years, it is performed nearly 700,000 times annually at a cost of approximately $4 billion.1,6,7 Like any surgical procedure, meniscectomy carries a risk for complications. In the double-blind, randomized trial reported on here, Sihvonen et al1 compared meniscectomy to a sham procedure for patients with knee pain but not OA.

STUDY SUMMARY
Meniscectomy and sham­surgery are equally effective

Sihvonen et al1 conducted a randomized, double-blind, sham-controlled trial at five orthopedic clinics in Finland. Patients ages 35 to 65 were enrolled if they had clinical findings of a medial meniscus tear and knee pain for more than three months that wasn’t relieved by conservative treatment. The trial excluded patients who had an obvious traumatic onset of symptoms; clinical or radiologic evidence of knee OA; a locked knee that could not be straightened; knee instability or decreased range of motion; previous surgery on the affected knee; fracture within the past 12 months on the affected limb; or other notable pathology on MRI or during arthroscopy.

Before randomization, 160 patients underwent diagnostic arthroscopy. Fourteen patients were excluded: six because they did not actually have a medial meniscal tear, one because he also had a lateral meniscus tear, three due to a major chondral flap, two who had already undergone meniscal repair, and two due to an osteochondral microfracture.

At the end of the diagnostic arthroscopy, each patient was blindly randomized to arthroscopic partial meniscectomy or sham surgery. To simulate the meniscectomy procedure, the surgeon similarly manipulated the knee, made comparable noise and vibration using tools and suction, and ensured that the patient was kept in the operating room (OR) for a comparable time. Only the orthopedic surgeon and OR staff were aware of which surgery the patient underwent, and these staff members were not included in further treatment or follow-up. After the procedure, all patients received the same walking aids and instructions for a graduated exercise program.

The 70 patients in the meniscectomy group and the 76 in the sham surgery group were similar in age (mean: 52 years), sex, BMI, and duration of pain (mean: 10 months). Patients in both groups also had similar tears noted on arthroscopy.

Three primary outcomes were measured before surgery and at 12 months: knee pain, knee symptoms and function, and quality of life (QoL). Knee pain after exercise was evaluated on a scale of 0 to 10, with 0 indicating no pain. The validated Lysholm knee score was used to assess knee symptoms and function, and the Western Ontario Meniscal Evaluation Tool (WOMET) was utilized to evaluate QoL; both are 100-point scales in which lower scores indicate more severe symptoms.

Both groups had marked improvement in pain and function from baseline to 12 months, and there was no significant difference between the two groups. Knee pain scores improved by 3.1 points in the meniscectomy group and 3.3 points in the sham surgery group.
 Lysholm symptom and function scores improved 21.7 points in the meniscectomy group and 23.3 points in the sham surgery group (a change of 11.5 points would have been considered clinically significant). The mean between-group difference was –1.6 points.

WOMET QoL scores improved 24.6 points in the meniscectomy group and 27.1 points in the sham surgery group (a change of 15.5 points would have been considered clinically significant). The mean between-group difference was –2.5 points.

There were no significant between-group differences in serious adverse events or number of patients who required subsequent knee surgery. Similar proportions in each group thought they had sham surgery, which confirmed the effectiveness of the blinding. Ninety-six percent of patients in the sham procedure group and 93% in the meniscectomy group reported they would be willing to repeat the procedure.

What's new and challenges to implementation >>

 

 


WHAT’S NEW
Recommend physical therapy,
exercise instead of surgery
Previous studies of arthroscopic partial meniscectomy to treat degenerative meniscal tears in patients with knee OA found no benefit.6,8 This study specifically examined patients without OA and found arthroscopic partial meniscectomy offered no benefit over sham surgery.

In addition to fewer referrals for meniscectomy, these findings could lead to another change in practice: Clinicians may be less likely to order MRI to confirm the diagnosis of a medial meniscal tear, since doing so will not change their therapeutic approach. This approach centers on recommending that patients with a degenerative meniscal tear start and stick with physical therapy and their designated exercise regimen.

CAVEATS
Surgery might be effective for more active patients

This study, as well as previous research, did not look at surgery for an acute medial meniscus tear following a traumatic incident, such as a fall or direct blow. Additionally, these results are based on improved outcomes in activities of daily living, and may not extend to patients who engage in high-level functioning, such as sports or strenuous work. The sham surgery group received lavage, which could be considered an active treatment, although a previous trial found lavage had no benefit over conservative treatment in patients with knee OA.4

CHALLENGES TO IMPLEMENTATION
It might be hard to convince patients they don’t need surgery

Some patients expect immediate intervention with surgery. It may be difficult to convince such patients that active participation in physical therapy can lead to the same outcomes as surgery. Spending time with your patient to explain the injury, what happens during surgery, and the evidence that shows a lack of difference in outcomes can lead to fewer surgeries. Most patients and clinicians will want to do an MRI after three months of persistent pain to determine the diagnosis, although some may be comfortable with continuing conservative treatment.

References
1. Sihvonen R, Paavola M, Malmivaara A, et al; Finnish Degenerative Meniscal Lesion Study (FIDELITY) Group. Arthroscopic partial meniscectomy versus sham surgery for a degenerative meniscal tear. N Engl J Med. 2013; 369:2515-2524.

2. American Academy of Orthopaedic Surgeons. Treatment of Osteoarthritis of the Knee. Evidence-Based Guideline. 2nd ed. Rosemont, IL: American Academy of Orthopaedic Surgeons; 2013.

3. Knee disorders. In: Hegmann KT, ed. Occupational Medicine Practice Guidelines. Evaluation and Management of Common Health Problems and Functional Recovery in Workers. 3rd ed. Elk Grove Village, IL: American College of Occupational and Environmental Medicine; 2011:1-503.

4. Kirkley A, Birmingham TB, Litchfield RB, et al. A randomized trial for arthroscopic surgery for osteoarthritis of the knee. N Engl J Med. 2008; 359:1097-1107.

5. Katz JN, Brophy RH, Chaisson CE, et al. Surgery versus physical therapy for a meniscal tear and osteoarthritis. N Engl J Med. 2013;368:1675-1684.

6. Cullen KA, Hall MJ, Golosinskiy A. Ambulatory surgery in the United States, 2006. Natl Health State Report. 2009;11:1-25.

7. Salzler MJ, Lin A, Miller CD, et al. Complications after arthroscopic knee surgery. Am J Sports Med. 2014;42:292-296.

8. Moseley JB, O’Malley K, Petersen NJ, et al. A controlled trial of arthroscopic surgery for osteoarthritis of the knee. N Engl J Med. 2002; 347:81-88.

Acknowledgement
The PURLs Surveillance System was supported in part by Grant Number UL1RR024999 from the National Center For Research Resources, a Clinical Translational Science Award to the University of Chicago. The content is solely the responsibility of the authors and does not necessarily represent the official views of the National Center For Research Resources or the National Institutes of Health.

Copyright © 2014. The Family Physicians Inquiries Network. All rights reserved.

Reprinted with permission from the Family Physicians Inquiries Network and The Journal of Family Practice. 2014;63(9):534-536.

References

References

Issue
Clinician Reviews - 24(10)
Issue
Clinician Reviews - 24(10)
Page Number
49,52,55
Page Number
49,52,55
Publications
Publications
Topics
Article Type
Display Headline
Surgery for Persistent Knee Pain? Not So Fast
Display Headline
Surgery for Persistent Knee Pain? Not So Fast
Legacy Keywords
knee pain, torn medial meniscus, medial meniscal tear, arthroscopic partial meniscectomy, medial meniscectomy
Legacy Keywords
knee pain, torn medial meniscus, medial meniscal tear, arthroscopic partial meniscectomy, medial meniscectomy
Sections
Article Source

PURLs Copyright

Inside the Article

Article PDF Media

Surgery for persistent knee pain? Not so fast

Article Type
Changed
Mon, 01/14/2019 - 13:52
Display Headline
Surgery for persistent knee pain? Not so fast

 

PRACTICE CHANGER

Do not refer patients with a degenerative medial meniscus tear for arthroscopic partial meniscectomy because outcomes are no better than those of conservative treatment.1

Strength of recommendation

B: Based on a single high-quality randomized control trial.

Sihvonen R, Paavola M, Malmivaara A, et al; Finnish Degenerative Meniscal Lesion Study (FIDELITY) Group. Arthroscopic partial meniscectomy versus sham surgery for a degenerative meniscal tear. N Engl J Med. 2013;369:2515-2524.

Illustrative case

A 40-year-old man comes to your office for follow-up of medial left knee pain he’s had for 3 months that hasn’t responded to conservative treatment. The pain developed gradually, without a history of trauma. The patient has no signs of degenerative joint disease on x-ray but magnetic resonance imaging (MRI) reveals a tear of the medial meniscus. Should you refer him for meniscectomy?

Patients and doctors alike tend to look for a treatment that will “fix” the problem, which may be why we have continued to use arthroscopic partial meniscectomy to attempt to relieve symptoms of meniscal tears despite a lack of evidence to support the practice.

Guidelines from the American Academy of Orthopaedic Surgeons state that the evidence for medial meniscectomy in patients with a torn meniscus and osteoarthritis (OA) is inconclusive; the organization offers no guidelines for patients with a torn meniscus who don’t have OA.2 The American College of Occupational and Environmental Medicine states that there is insufficient evidence to support arthroscopic partial meniscectomy for symptomatic, torn medial menisci for select patients and “the vast majority of patients [with medial meniscal tears] do not require surgery.”3 Previous studies have concluded that arthroscopic surgery for OA of the knee provides no additional benefit to optimized physical and medical therapy.4 Furthermore, research by Katz et al5 shows that meniscectomy provides no benefit over conservative treatment in functional status at 6 months in patients with OA and a medial meniscal tear.

That said, arthroscopic partial meniscectomy is still the most common orthopedic procedure in the United States.1 Although its use has decreased over the last 15 years, it is performed nearly 700,000 times annually at a cost of approximately $4 billion.1,6,7 Like any surgical procedure, meniscectomy carries a risk of complications. In the double-blind, randomized trial reported on here, Sihvonen et al1 compared meniscectomy to a sham procedure for patients with knee pain, but not OA.

STUDY SUMMARY: Meniscectomy and sham surgery 
are equally effective


Sihvonen et al1 conducted a randomized, double-blind, sham-controlled trial at 5 orthopedic clinics in Finland. Patients ages 35 to 65 years were enrolled if they had clinical findings of a medial meniscus tear and knee pain for >3 months that wasn’t relieved by conservative treatment. The trial excluded patients who had an obvious traumatic onset of symptoms; clinical or radiological evidence of knee OA; a locked knee that could not be straightened; knee instability or decreased range of motion; previous surgery on the affected knee; fracture within the past 12 months on the affected limb; or other notable pathology on MRI or during arthroscopy.

Before randomization, 160 patients underwent diagnostic arthroscopy. Fourteen patients were excluded: 6 because they did not actually have a medial meniscal tear, one because he also had a lateral meniscus tear, 3 due to a major chondral flap, 2 who had already undergone meniscal repair, and 2 due to an osteochondral microfracture.

At the end of the diagnostic arthroscopy, each patient was blindly randomized to arthroscopic partial meniscectomy or sham surgery. To simulate the meniscectomy procedure, the surgeon similarly manipulated the knee, made comparable noise and vibration using tools and suction, and ensured that the patient was kept in the operating room (OR) for a comparable time. Only the orthopedic surgeon and OR staff were aware of which surgery the patient underwent, and these staff members were not included in further treatment or follow-up. After the procedure, all patients received the same walking aids and instructions for a graduated exercise program.

The 70 patients in the meniscectomy group and the 76 in the sham surgery group were similar in age (mean: 52 years), sex, body mass index, and duration of pain (mean: 10 months). Patients in both groups also had similar tears noted on arthroscopy.

Three primary outcomes were measured before surgery and at 12 months: knee pain, knee symptoms and function, and quality of life. Knee pain after exercise was evaluated on a 0 to 10 scale, with 0 indicating no pain. The validated Lysholm knee score was used to assess knee symptoms and function and the Western Ontario Meniscal Evaluation Tool (WOMET) was utilized to evaluate quality of life; both are 100-point scales in which lower scores indicate more severe symptoms.

 

 

Both groups had marked improvement in pain and function from baseline to 12 months, and there was no significant difference between the 2 groups. Knee pain scores improved by 3.1 in the meniscectomy group and 3.3 in the sham surgery group.


Lysholm symptom and function scores improved 21.7 points in the meniscectomy group and 23.3 points in the sham surgery group (a change of 11.5 points would have been considered clinically significant). The mean between-group difference was -1.6 points (95% confidence interval [CI], -7.2 to 4.0).

Both meniscectomy and sham surgery led
 to marked improvement at 12 months, with no significant differences in outcomes. WOMET quality of life scores improved 24.6 points in the meniscectomy group and 27.1 points in the sham surgery (a change of 15.5 points would have been considered clinically significant). The mean between-group difference was -2.5 points (95% CI, -9.2 to 4.1).

There were no significant between-group differences in serious adverse events or number of patients who required subsequent knee surgery. Similar proportions in each group thought they had sham surgery, which confirmed the effectiveness of the blinding. Ninety-six percent of patients in the sham procedure group and 93% in the meniscectomy group reported they would be willing to repeat the procedure.

WHAT'S NEW: Recommend physical therapy,
 exercise instead of surgery


Previous studies of arthroscopic partial meniscectomy to treat degenerative meniscal tears in patients with knee OA found no benefit.6,8 This study specifically examined patients without OA and found arthroscopic partial meniscectomy offered no benefit over sham surgery.

In addition to fewer referrals for meniscectomy, these findings could lead to another change in practice: Physicians may be less likely to order an MRI to confirm the diagnosis of a medial meniscal tear, since doing so will not change their therapeutic approach. This approach centers on recommending that patients with a degenerative meniscal tear start and stick with physical therapy and their designated exercise regimen.

CAVEATS: Surgery might be effective for more active patients

This study, as well as previous research, did not look at surgery for an acute medial meniscus tear following a traumatic incident, such as a fall or direct blow. Additionally, these results are based on improved outcomes in activities of daily living, and may not extend to patients who engage in high-level functioning, such as sports or strenuous work. The sham surgery group received lavage, which could be considered an active treatment, although a previous trial found lavage had no benefit over conservative treatment in patients with knee OA.4

CHALLENGES TO IMPLEMENTATION: It might be hard to convince patients they don’t need surgery

Some patients expect immediate intervention with surgery. It may be difficult to convince such patients that active participation in physical therapy can lead to the same outcomes as surgery. Spending time with your patient to explain the injury, what happens during surgery, and the evidence that shows a lack of difference in outcomes can lead to fewer surgeries. Most patients and physicians will want to do an MRI after 3 months of persistent pain to determine the diagnosis, although some may be comfortable with continuing conservative treatment.

Acknowledgement
The PURLs Surveillance System was supported in part by Grant Number UL1RR024999 from the National Center For Research Resources, a Clinical Translational Science Award to the University of Chicago. The content is solely the responsibility of the authors and does not necessarily represent the official views of the National Center For Research Resources or the National Institutes of Health.

Click here to view PURL METHODOLOGY

References

 

1. Sihvonen R, Paavola M, Malmivaara A, et al; Finnish Degenerative Meniscal Lesion Study (FIDELITY) Group. Arthroscopic partial meniscectomy versus sham surgery for a degenerative meniscal tear. N Engl J Med. 2013;369:2515-2524.

2. American Academy of Orthopaedic Surgeons. Treatment of Osteoarthritis of the Knee. Evidence-Based Guideline. 2nd ed. Rosemont, IL: American Academy of Orthopaedic Surgeons; 2013.

3. Knee disorders. In: Hegmann KT, ed. Occupational Medicine Practice Guidelines. Evaluation and Management of Common Health Problems and Functional Recovery in Workers. 3rd ed. Elk Grove Village, IL: American College of Occupational and Environmental Medicine; 2011:1-503.

4. Kirkley A, Birmingham TB, Litchfield RB, et al. A randomized trial for arthroscopic surgery for osteoarthritis of the knee. N Engl J Med. 2008;359:1097-1107.

5. Katz JN, Brophy RH, Chaisson CE, et al. Surgery versus physical therapy for a meniscal tear and osteoarthritis. N Engl J Med. 2013;368:1675-1684.

6. Cullen KA, Hall MJ, Golosinskiy A. Ambulatory surgery in the United States, 2006. Natl Health State Report. 2009;11:1-25.

7. Salzler MJ, Lin A, Miller CD, et al. Complications after arthroscopic knee surgery. Am J Sports Med. 2014;42:292-296.

8. Moseley JB, O’Malley K, Petersen NJ, et al. A controlled trial of arthroscopic surgery for osteoarthritis of the knee. N Engl J Med. 2002;347:81-88.

Article PDF
Author and Disclosure Information

 

Mallory McClester Brown, MD; Anne Mounsey, MD
Department of Family Medicine, University of North Carolina at Chapel Hill

PURLs EDITOR
Jim Stevermer, MD, MSPH

Department of Family and Community Medicine, University of Missouri-Columbia

Issue
The Journal of Family Practice - 63(9)
Publications
Topics
Page Number
534-536
Legacy Keywords
torn medial meniscus; meniscectomy; physical therapy; Mallory McClester Brown, MD; Anne Mounsey, MD
Sections
Author and Disclosure Information

 

Mallory McClester Brown, MD; Anne Mounsey, MD
Department of Family Medicine, University of North Carolina at Chapel Hill

PURLs EDITOR
Jim Stevermer, MD, MSPH

Department of Family and Community Medicine, University of Missouri-Columbia

Author and Disclosure Information

 

Mallory McClester Brown, MD; Anne Mounsey, MD
Department of Family Medicine, University of North Carolina at Chapel Hill

PURLs EDITOR
Jim Stevermer, MD, MSPH

Department of Family and Community Medicine, University of Missouri-Columbia

Article PDF
Article PDF
Related Articles

 

PRACTICE CHANGER

Do not refer patients with a degenerative medial meniscus tear for arthroscopic partial meniscectomy because outcomes are no better than those of conservative treatment.1

Strength of recommendation

B: Based on a single high-quality randomized control trial.

Sihvonen R, Paavola M, Malmivaara A, et al; Finnish Degenerative Meniscal Lesion Study (FIDELITY) Group. Arthroscopic partial meniscectomy versus sham surgery for a degenerative meniscal tear. N Engl J Med. 2013;369:2515-2524.

Illustrative case

A 40-year-old man comes to your office for follow-up of medial left knee pain he’s had for 3 months that hasn’t responded to conservative treatment. The pain developed gradually, without a history of trauma. The patient has no signs of degenerative joint disease on x-ray but magnetic resonance imaging (MRI) reveals a tear of the medial meniscus. Should you refer him for meniscectomy?

Patients and doctors alike tend to look for a treatment that will “fix” the problem, which may be why we have continued to use arthroscopic partial meniscectomy to attempt to relieve symptoms of meniscal tears despite a lack of evidence to support the practice.

Guidelines from the American Academy of Orthopaedic Surgeons state that the evidence for medial meniscectomy in patients with a torn meniscus and osteoarthritis (OA) is inconclusive; the organization offers no guidelines for patients with a torn meniscus who don’t have OA.2 The American College of Occupational and Environmental Medicine states that there is insufficient evidence to support arthroscopic partial meniscectomy for symptomatic, torn medial menisci for select patients and “the vast majority of patients [with medial meniscal tears] do not require surgery.”3 Previous studies have concluded that arthroscopic surgery for OA of the knee provides no additional benefit to optimized physical and medical therapy.4 Furthermore, research by Katz et al5 shows that meniscectomy provides no benefit over conservative treatment in functional status at 6 months in patients with OA and a medial meniscal tear.

That said, arthroscopic partial meniscectomy is still the most common orthopedic procedure in the United States.1 Although its use has decreased over the last 15 years, it is performed nearly 700,000 times annually at a cost of approximately $4 billion.1,6,7 Like any surgical procedure, meniscectomy carries a risk of complications. In the double-blind, randomized trial reported on here, Sihvonen et al1 compared meniscectomy to a sham procedure for patients with knee pain, but not OA.

STUDY SUMMARY: Meniscectomy and sham surgery 
are equally effective


Sihvonen et al1 conducted a randomized, double-blind, sham-controlled trial at 5 orthopedic clinics in Finland. Patients ages 35 to 65 years were enrolled if they had clinical findings of a medial meniscus tear and knee pain for >3 months that wasn’t relieved by conservative treatment. The trial excluded patients who had an obvious traumatic onset of symptoms; clinical or radiological evidence of knee OA; a locked knee that could not be straightened; knee instability or decreased range of motion; previous surgery on the affected knee; fracture within the past 12 months on the affected limb; or other notable pathology on MRI or during arthroscopy.

Before randomization, 160 patients underwent diagnostic arthroscopy. Fourteen patients were excluded: 6 because they did not actually have a medial meniscal tear, one because he also had a lateral meniscus tear, 3 due to a major chondral flap, 2 who had already undergone meniscal repair, and 2 due to an osteochondral microfracture.

At the end of the diagnostic arthroscopy, each patient was blindly randomized to arthroscopic partial meniscectomy or sham surgery. To simulate the meniscectomy procedure, the surgeon similarly manipulated the knee, made comparable noise and vibration using tools and suction, and ensured that the patient was kept in the operating room (OR) for a comparable time. Only the orthopedic surgeon and OR staff were aware of which surgery the patient underwent, and these staff members were not included in further treatment or follow-up. After the procedure, all patients received the same walking aids and instructions for a graduated exercise program.

The 70 patients in the meniscectomy group and the 76 in the sham surgery group were similar in age (mean: 52 years), sex, body mass index, and duration of pain (mean: 10 months). Patients in both groups also had similar tears noted on arthroscopy.

Three primary outcomes were measured before surgery and at 12 months: knee pain, knee symptoms and function, and quality of life. Knee pain after exercise was evaluated on a 0 to 10 scale, with 0 indicating no pain. The validated Lysholm knee score was used to assess knee symptoms and function and the Western Ontario Meniscal Evaluation Tool (WOMET) was utilized to evaluate quality of life; both are 100-point scales in which lower scores indicate more severe symptoms.

 

 

Both groups had marked improvement in pain and function from baseline to 12 months, and there was no significant difference between the 2 groups. Knee pain scores improved by 3.1 in the meniscectomy group and 3.3 in the sham surgery group.


Lysholm symptom and function scores improved 21.7 points in the meniscectomy group and 23.3 points in the sham surgery group (a change of 11.5 points would have been considered clinically significant). The mean between-group difference was -1.6 points (95% confidence interval [CI], -7.2 to 4.0).

Both meniscectomy and sham surgery led
 to marked improvement at 12 months, with no significant differences in outcomes. WOMET quality of life scores improved 24.6 points in the meniscectomy group and 27.1 points in the sham surgery (a change of 15.5 points would have been considered clinically significant). The mean between-group difference was -2.5 points (95% CI, -9.2 to 4.1).

There were no significant between-group differences in serious adverse events or number of patients who required subsequent knee surgery. Similar proportions in each group thought they had sham surgery, which confirmed the effectiveness of the blinding. Ninety-six percent of patients in the sham procedure group and 93% in the meniscectomy group reported they would be willing to repeat the procedure.

WHAT'S NEW: Recommend physical therapy,
 exercise instead of surgery


Previous studies of arthroscopic partial meniscectomy to treat degenerative meniscal tears in patients with knee OA found no benefit.6,8 This study specifically examined patients without OA and found arthroscopic partial meniscectomy offered no benefit over sham surgery.

In addition to fewer referrals for meniscectomy, these findings could lead to another change in practice: Physicians may be less likely to order an MRI to confirm the diagnosis of a medial meniscal tear, since doing so will not change their therapeutic approach. This approach centers on recommending that patients with a degenerative meniscal tear start and stick with physical therapy and their designated exercise regimen.

CAVEATS: Surgery might be effective for more active patients

This study, as well as previous research, did not look at surgery for an acute medial meniscus tear following a traumatic incident, such as a fall or direct blow. Additionally, these results are based on improved outcomes in activities of daily living, and may not extend to patients who engage in high-level functioning, such as sports or strenuous work. The sham surgery group received lavage, which could be considered an active treatment, although a previous trial found lavage had no benefit over conservative treatment in patients with knee OA.4

CHALLENGES TO IMPLEMENTATION: It might be hard to convince patients they don’t need surgery

Some patients expect immediate intervention with surgery. It may be difficult to convince such patients that active participation in physical therapy can lead to the same outcomes as surgery. Spending time with your patient to explain the injury, what happens during surgery, and the evidence that shows a lack of difference in outcomes can lead to fewer surgeries. Most patients and physicians will want to do an MRI after 3 months of persistent pain to determine the diagnosis, although some may be comfortable with continuing conservative treatment.

Acknowledgement
The PURLs Surveillance System was supported in part by Grant Number UL1RR024999 from the National Center For Research Resources, a Clinical Translational Science Award to the University of Chicago. The content is solely the responsibility of the authors and does not necessarily represent the official views of the National Center For Research Resources or the National Institutes of Health.

Click here to view PURL METHODOLOGY

 

PRACTICE CHANGER

Do not refer patients with a degenerative medial meniscus tear for arthroscopic partial meniscectomy because outcomes are no better than those of conservative treatment.1

Strength of recommendation

B: Based on a single high-quality randomized control trial.

Sihvonen R, Paavola M, Malmivaara A, et al; Finnish Degenerative Meniscal Lesion Study (FIDELITY) Group. Arthroscopic partial meniscectomy versus sham surgery for a degenerative meniscal tear. N Engl J Med. 2013;369:2515-2524.

Illustrative case

A 40-year-old man comes to your office for follow-up of medial left knee pain he’s had for 3 months that hasn’t responded to conservative treatment. The pain developed gradually, without a history of trauma. The patient has no signs of degenerative joint disease on x-ray but magnetic resonance imaging (MRI) reveals a tear of the medial meniscus. Should you refer him for meniscectomy?

Patients and doctors alike tend to look for a treatment that will “fix” the problem, which may be why we have continued to use arthroscopic partial meniscectomy to attempt to relieve symptoms of meniscal tears despite a lack of evidence to support the practice.

Guidelines from the American Academy of Orthopaedic Surgeons state that the evidence for medial meniscectomy in patients with a torn meniscus and osteoarthritis (OA) is inconclusive; the organization offers no guidelines for patients with a torn meniscus who don’t have OA.2 The American College of Occupational and Environmental Medicine states that there is insufficient evidence to support arthroscopic partial meniscectomy for symptomatic, torn medial menisci for select patients and “the vast majority of patients [with medial meniscal tears] do not require surgery.”3 Previous studies have concluded that arthroscopic surgery for OA of the knee provides no additional benefit to optimized physical and medical therapy.4 Furthermore, research by Katz et al5 shows that meniscectomy provides no benefit over conservative treatment in functional status at 6 months in patients with OA and a medial meniscal tear.

That said, arthroscopic partial meniscectomy is still the most common orthopedic procedure in the United States.1 Although its use has decreased over the last 15 years, it is performed nearly 700,000 times annually at a cost of approximately $4 billion.1,6,7 Like any surgical procedure, meniscectomy carries a risk of complications. In the double-blind, randomized trial reported on here, Sihvonen et al1 compared meniscectomy to a sham procedure for patients with knee pain, but not OA.

STUDY SUMMARY: Meniscectomy and sham surgery 
are equally effective


Sihvonen et al1 conducted a randomized, double-blind, sham-controlled trial at 5 orthopedic clinics in Finland. Patients ages 35 to 65 years were enrolled if they had clinical findings of a medial meniscus tear and knee pain for >3 months that wasn’t relieved by conservative treatment. The trial excluded patients who had an obvious traumatic onset of symptoms; clinical or radiological evidence of knee OA; a locked knee that could not be straightened; knee instability or decreased range of motion; previous surgery on the affected knee; fracture within the past 12 months on the affected limb; or other notable pathology on MRI or during arthroscopy.

Before randomization, 160 patients underwent diagnostic arthroscopy. Fourteen patients were excluded: 6 because they did not actually have a medial meniscal tear, one because he also had a lateral meniscus tear, 3 due to a major chondral flap, 2 who had already undergone meniscal repair, and 2 due to an osteochondral microfracture.

At the end of the diagnostic arthroscopy, each patient was blindly randomized to arthroscopic partial meniscectomy or sham surgery. To simulate the meniscectomy procedure, the surgeon similarly manipulated the knee, made comparable noise and vibration using tools and suction, and ensured that the patient was kept in the operating room (OR) for a comparable time. Only the orthopedic surgeon and OR staff were aware of which surgery the patient underwent, and these staff members were not included in further treatment or follow-up. After the procedure, all patients received the same walking aids and instructions for a graduated exercise program.

The 70 patients in the meniscectomy group and the 76 in the sham surgery group were similar in age (mean: 52 years), sex, body mass index, and duration of pain (mean: 10 months). Patients in both groups also had similar tears noted on arthroscopy.

Three primary outcomes were measured before surgery and at 12 months: knee pain, knee symptoms and function, and quality of life. Knee pain after exercise was evaluated on a 0 to 10 scale, with 0 indicating no pain. The validated Lysholm knee score was used to assess knee symptoms and function and the Western Ontario Meniscal Evaluation Tool (WOMET) was utilized to evaluate quality of life; both are 100-point scales in which lower scores indicate more severe symptoms.

 

 

Both groups had marked improvement in pain and function from baseline to 12 months, and there was no significant difference between the 2 groups. Knee pain scores improved by 3.1 in the meniscectomy group and 3.3 in the sham surgery group.


Lysholm symptom and function scores improved 21.7 points in the meniscectomy group and 23.3 points in the sham surgery group (a change of 11.5 points would have been considered clinically significant). The mean between-group difference was -1.6 points (95% confidence interval [CI], -7.2 to 4.0).

Both meniscectomy and sham surgery led
 to marked improvement at 12 months, with no significant differences in outcomes. WOMET quality of life scores improved 24.6 points in the meniscectomy group and 27.1 points in the sham surgery (a change of 15.5 points would have been considered clinically significant). The mean between-group difference was -2.5 points (95% CI, -9.2 to 4.1).

There were no significant between-group differences in serious adverse events or number of patients who required subsequent knee surgery. Similar proportions in each group thought they had sham surgery, which confirmed the effectiveness of the blinding. Ninety-six percent of patients in the sham procedure group and 93% in the meniscectomy group reported they would be willing to repeat the procedure.

WHAT'S NEW: Recommend physical therapy,
 exercise instead of surgery


Previous studies of arthroscopic partial meniscectomy to treat degenerative meniscal tears in patients with knee OA found no benefit.6,8 This study specifically examined patients without OA and found arthroscopic partial meniscectomy offered no benefit over sham surgery.

In addition to fewer referrals for meniscectomy, these findings could lead to another change in practice: Physicians may be less likely to order an MRI to confirm the diagnosis of a medial meniscal tear, since doing so will not change their therapeutic approach. This approach centers on recommending that patients with a degenerative meniscal tear start and stick with physical therapy and their designated exercise regimen.

CAVEATS: Surgery might be effective for more active patients

This study, as well as previous research, did not look at surgery for an acute medial meniscus tear following a traumatic incident, such as a fall or direct blow. Additionally, these results are based on improved outcomes in activities of daily living, and may not extend to patients who engage in high-level functioning, such as sports or strenuous work. The sham surgery group received lavage, which could be considered an active treatment, although a previous trial found lavage had no benefit over conservative treatment in patients with knee OA.4

CHALLENGES TO IMPLEMENTATION: It might be hard to convince patients they don’t need surgery

Some patients expect immediate intervention with surgery. It may be difficult to convince such patients that active participation in physical therapy can lead to the same outcomes as surgery. Spending time with your patient to explain the injury, what happens during surgery, and the evidence that shows a lack of difference in outcomes can lead to fewer surgeries. Most patients and physicians will want to do an MRI after 3 months of persistent pain to determine the diagnosis, although some may be comfortable with continuing conservative treatment.

Acknowledgement
The PURLs Surveillance System was supported in part by Grant Number UL1RR024999 from the National Center For Research Resources, a Clinical Translational Science Award to the University of Chicago. The content is solely the responsibility of the authors and does not necessarily represent the official views of the National Center For Research Resources or the National Institutes of Health.

Click here to view PURL METHODOLOGY

References

 

1. Sihvonen R, Paavola M, Malmivaara A, et al; Finnish Degenerative Meniscal Lesion Study (FIDELITY) Group. Arthroscopic partial meniscectomy versus sham surgery for a degenerative meniscal tear. N Engl J Med. 2013;369:2515-2524.

2. American Academy of Orthopaedic Surgeons. Treatment of Osteoarthritis of the Knee. Evidence-Based Guideline. 2nd ed. Rosemont, IL: American Academy of Orthopaedic Surgeons; 2013.

3. Knee disorders. In: Hegmann KT, ed. Occupational Medicine Practice Guidelines. Evaluation and Management of Common Health Problems and Functional Recovery in Workers. 3rd ed. Elk Grove Village, IL: American College of Occupational and Environmental Medicine; 2011:1-503.

4. Kirkley A, Birmingham TB, Litchfield RB, et al. A randomized trial for arthroscopic surgery for osteoarthritis of the knee. N Engl J Med. 2008;359:1097-1107.

5. Katz JN, Brophy RH, Chaisson CE, et al. Surgery versus physical therapy for a meniscal tear and osteoarthritis. N Engl J Med. 2013;368:1675-1684.

6. Cullen KA, Hall MJ, Golosinskiy A. Ambulatory surgery in the United States, 2006. Natl Health State Report. 2009;11:1-25.

7. Salzler MJ, Lin A, Miller CD, et al. Complications after arthroscopic knee surgery. Am J Sports Med. 2014;42:292-296.

8. Moseley JB, O’Malley K, Petersen NJ, et al. A controlled trial of arthroscopic surgery for osteoarthritis of the knee. N Engl J Med. 2002;347:81-88.

References

 

1. Sihvonen R, Paavola M, Malmivaara A, et al; Finnish Degenerative Meniscal Lesion Study (FIDELITY) Group. Arthroscopic partial meniscectomy versus sham surgery for a degenerative meniscal tear. N Engl J Med. 2013;369:2515-2524.

2. American Academy of Orthopaedic Surgeons. Treatment of Osteoarthritis of the Knee. Evidence-Based Guideline. 2nd ed. Rosemont, IL: American Academy of Orthopaedic Surgeons; 2013.

3. Knee disorders. In: Hegmann KT, ed. Occupational Medicine Practice Guidelines. Evaluation and Management of Common Health Problems and Functional Recovery in Workers. 3rd ed. Elk Grove Village, IL: American College of Occupational and Environmental Medicine; 2011:1-503.

4. Kirkley A, Birmingham TB, Litchfield RB, et al. A randomized trial for arthroscopic surgery for osteoarthritis of the knee. N Engl J Med. 2008;359:1097-1107.

5. Katz JN, Brophy RH, Chaisson CE, et al. Surgery versus physical therapy for a meniscal tear and osteoarthritis. N Engl J Med. 2013;368:1675-1684.

6. Cullen KA, Hall MJ, Golosinskiy A. Ambulatory surgery in the United States, 2006. Natl Health State Report. 2009;11:1-25.

7. Salzler MJ, Lin A, Miller CD, et al. Complications after arthroscopic knee surgery. Am J Sports Med. 2014;42:292-296.

8. Moseley JB, O’Malley K, Petersen NJ, et al. A controlled trial of arthroscopic surgery for osteoarthritis of the knee. N Engl J Med. 2002;347:81-88.

Issue
The Journal of Family Practice - 63(9)
Issue
The Journal of Family Practice - 63(9)
Page Number
534-536
Page Number
534-536
Publications
Publications
Topics
Article Type
Display Headline
Surgery for persistent knee pain? Not so fast
Display Headline
Surgery for persistent knee pain? Not so fast
Legacy Keywords
torn medial meniscus; meniscectomy; physical therapy; Mallory McClester Brown, MD; Anne Mounsey, MD
Legacy Keywords
torn medial meniscus; meniscectomy; physical therapy; Mallory McClester Brown, MD; Anne Mounsey, MD
Sections
PURLs Copyright

Copyright © 2014 Family Physicians Inquiries Network. All rights reserved.

Disallow All Ads
Alternative CME
Article PDF Media

Why You Shouldn’t Start β-Blockers Before Surgery

Article Type
Changed
Tue, 12/13/2016 - 12:08
Display Headline
Why You Shouldn’t Start β-Blockers Before Surgery
A new meta-analysis finds that initiating β-blockers before surgery increases patients’ risk for death.

PRACTICE CHANGER

Do not routinely initiate β-blockers in patients undergoing intermediate- or high-risk noncardiac surgery. β-Blockers appear to increase the 30-day risk for all-cause mortality.1

STRENGTH OF RECOMMENDATION

A: Based on meta-analysis of nine randomized controlled trials (RCTs).1

ILLUSTRATIVE CASE
A 67-year-old woman with diabetes, hypertension, and hyperlipidemia presents for evaluation prior to a total hip arthroplasty. She is not taking a β-blocker. Should you prescribe one?

Study summary >>

 

 

Current guidelines from the American College of Cardiology Foundation (ACCF) and the American Heart Association (AHA) recommend starting 

β-blockers to prevent cardiac events in patients about to undergo intermediate- or high-risk surgery or vascular surgery who have a history of inducible ischemia, coronary artery disease (CAD), or at least one risk factor for CAD.2 However, the majority of the evidence for these guidelines, which were published in 2009 and are in the process of being updated, came from the DECREASE (Dutch Echocardiographic Cardiac Risk Evaluation Applying Stress Echocardiography) trials. These trials  have been discredited due to serious methodologic flaws, including falsified descriptions of how outcomes were determined and fictitious databases.3

A new meta-analysis conducted by Bouri et al1 that excluded the DECREASE trials found that, although preoperative β-blockers reduce the rate of certain nonfatal outcomes, they increase the risk for death and stroke.

STUDY SUMMARY
Preop β-blockers do more harm than good

Bouri et al1 conducted a meta-analysis of published RCTs evaluating preoperative β-blockers versus placebo for patients undergoing noncardiac surgery. Of the 11 studies that met eligibility criteria, two were the discredited DECREASE trials. Thus, Bouri et al1 analyzed nine high-quality RCTs that included 10,529 patients.

Most studies included patients undergoing vascular surgery. Some studies also included intra-abdominal, intrathoracic, neurosurgic, orthopedic, urologic, and gynecologic surgeries. β-Blockers were started no more than a day before surgery and were discontinued at hospital discharge or up to 30 days postop. Metoprolol was used in five trials, bisoprolol in one trial, atenolol in two trials, and propranolol in one trial. The primary endpoint was all-cause mortality within 30 days.

A total of 5,264 patients were randomly assigned to receive β-blockers and 5,265 to placebo. There were 162 deaths in the β-blocker group and 129 deaths in the placebo group. Patients who received β-blockers had a 27% increased risk for all-cause mortality (risk ratio [RR] = 1.27). The number needed to harm was 160.

Six of the studies also evaluated rates of nonfatal MI, nonfatal stroke, and hypotension. β-Blockers lowered the risk for nonfatal MI (RR = 0.73) but increased the risk for nonfatal stroke (RR = 1.73) and hypotension (RR = 1.51).

This meta-analysis was dominated by the 2008 Peri-Operative ISchemic Evaluation (POISE) trial, an RCT that compared placebo to extended-release metoprolol (100 mg 2 to 4 h before surgery, followed by 200 mg/d for 30 d), in 8,351 patients with, or at risk for, atherosclerotic disease.4 While β-blockers reduced the risk for MI and atrial fibrillation, they increased the risk for mortality and stroke, likely due to drug-induced hypotension. The slightly larger-than-typical doses of β-blockers used in this study may have contributed to the excess mortality.

What's new and challenges to implementation >> 

 

 

WHAT’S NEW
Avoiding β-blockers in surgery patients will prevent deaths

Bouri et al1 found that while β-blockers protect against nonfatal MIs, they increase the risk for nonfatal strokes and death. This new meta-analysis challenges the ACCF/AHA recommendations by suggesting that abandoning the use of β-blockers for preoperative patients who aren’t already taking them will prevent a substantial number of perioperative deaths. Bouri et al1 estimate that in the United Kingdom, where 47,286 deaths occur annually within 30 days of intermediate- or high-risk procedures, the number of iatrogenic deaths would drop by approximately 10,000 if β-blockers were not used.1

CAVEATS
Don’t stop β-blockers in patients who already take them

This meta-analysis did not evaluate outcomes in patients who were already taking β-blockers. These patients should continue to take them in the perioperative period, which is in line with current ACCF/AHA guidelines.

CHALLENGES TO IMPLEMENTATION
Reluctance to disregard published guidelines 
Some clinicians may not be comfortable ignoring the current ACCF/AHA guidelines that make a Class IIA recommendation (it is reasonable to administer this treatment) for the use of preoperative β-blockade for patients at risk for cardiovascular events who were not previously taking a β-blocker. This updated meta-analysis excludes the discredited DECREASE trials and challenges us to act against these current guidelines while we await updated recommendations.       

REFERENCES
1. Bouri S, Shun-Shin MJ, Cole GD, et al. Meta-analysis of secure randomised controlled trials of ß-blockade to prevent perioperative death in non-cardiac surgery. Heart. 2014;100:456-464.

2. American College of Cardiology Foundation/American Heart Association Task Force on Practice Guidelines; American Society of Echocardiography; American Society of Nuclear Cardiology; Heart Rhythm Society; Society of Cardiovascular Anesthesiologists; Society for Cardiovascular Angiography and Interventions; Society for Vascular Medicine; Society for Vascular Surgery; Fleisher LA, Beckman JA, Brown KA, et al. 2009 ACCF/AHA focused update on perioperative beta blockade incorporated into the ACC/AHA 2007 guidelines on perioperative cardiovascular evaluation and care for noncardiac surgery. J Am Coll Cardiol. 2009;54:e13-e118.

3. Erasmus Medical Center Follow-up Investigation Committee. Report on the 2012 follow-up investigation of possible breaches of academic integrity (September 30, 2012). CardioBrief. Available at: http://cardiobrief.files.wordpress.com/2012/10/integrity-report-2012-10-english-translation.pdf. Accessed August 14, 2014.

4. Devereaux PJ, Yang H, Yusuf S, et al; POISE Study Group. Effects of extended-release metoprolol succinate in patients undergoing non-cardiac surgery (POISE trial): a randomised controlled trial. Lancet. 2008;
371:1839-1847.

ACKNOWLEDGEMENT
The PURLs Surveillance System was supported in part by Grant Number UL1RR024999 from the National Center For Research Resources, a Clinical Translational Science Award to the University of Chicago. The content is solely the responsibility of the authors and does not necessarily represent the official views of the National Center For Research Resources or the National Institutes of Health.

Copyright © 2014. The Family Physicians Inquiries Network. All rights reserved.

Reprinted with permission from the Family Physicians Inquiries Network and The Journal of Family Practice. 2014;63(6):E15-E16.

References

Article PDF
Author and Disclosure Information

Anne Mounsey, MD, Jodi M. Roque, MD, Mari Egan, MD
Anne Mounsey and Jodi M. Roque are in the Department of Family Medicine at the University of North Carolina at Chapel Hill. Mari Egan is in the Department of Family Medicine at the University of Chicago.

Issue
Clinician Reviews - 24(9)
Publications
Topics
Page Number
21-22
Legacy Keywords
beta blockers, β-Blockers, surgery, non cardiac, noncardiac surgery, PURLs
Sections
Author and Disclosure Information

Anne Mounsey, MD, Jodi M. Roque, MD, Mari Egan, MD
Anne Mounsey and Jodi M. Roque are in the Department of Family Medicine at the University of North Carolina at Chapel Hill. Mari Egan is in the Department of Family Medicine at the University of Chicago.

Author and Disclosure Information

Anne Mounsey, MD, Jodi M. Roque, MD, Mari Egan, MD
Anne Mounsey and Jodi M. Roque are in the Department of Family Medicine at the University of North Carolina at Chapel Hill. Mari Egan is in the Department of Family Medicine at the University of Chicago.

Article PDF
Article PDF
Related Articles
A new meta-analysis finds that initiating β-blockers before surgery increases patients’ risk for death.
A new meta-analysis finds that initiating β-blockers before surgery increases patients’ risk for death.

PRACTICE CHANGER

Do not routinely initiate β-blockers in patients undergoing intermediate- or high-risk noncardiac surgery. β-Blockers appear to increase the 30-day risk for all-cause mortality.1

STRENGTH OF RECOMMENDATION

A: Based on meta-analysis of nine randomized controlled trials (RCTs).1

ILLUSTRATIVE CASE
A 67-year-old woman with diabetes, hypertension, and hyperlipidemia presents for evaluation prior to a total hip arthroplasty. She is not taking a β-blocker. Should you prescribe one?

Study summary >>

 

 

Current guidelines from the American College of Cardiology Foundation (ACCF) and the American Heart Association (AHA) recommend starting 

β-blockers to prevent cardiac events in patients about to undergo intermediate- or high-risk surgery or vascular surgery who have a history of inducible ischemia, coronary artery disease (CAD), or at least one risk factor for CAD.2 However, the majority of the evidence for these guidelines, which were published in 2009 and are in the process of being updated, came from the DECREASE (Dutch Echocardiographic Cardiac Risk Evaluation Applying Stress Echocardiography) trials. These trials  have been discredited due to serious methodologic flaws, including falsified descriptions of how outcomes were determined and fictitious databases.3

A new meta-analysis conducted by Bouri et al1 that excluded the DECREASE trials found that, although preoperative β-blockers reduce the rate of certain nonfatal outcomes, they increase the risk for death and stroke.

STUDY SUMMARY
Preop β-blockers do more harm than good

Bouri et al1 conducted a meta-analysis of published RCTs evaluating preoperative β-blockers versus placebo for patients undergoing noncardiac surgery. Of the 11 studies that met eligibility criteria, two were the discredited DECREASE trials. Thus, Bouri et al1 analyzed nine high-quality RCTs that included 10,529 patients.

Most studies included patients undergoing vascular surgery. Some studies also included intra-abdominal, intrathoracic, neurosurgic, orthopedic, urologic, and gynecologic surgeries. β-Blockers were started no more than a day before surgery and were discontinued at hospital discharge or up to 30 days postop. Metoprolol was used in five trials, bisoprolol in one trial, atenolol in two trials, and propranolol in one trial. The primary endpoint was all-cause mortality within 30 days.

A total of 5,264 patients were randomly assigned to receive β-blockers and 5,265 to placebo. There were 162 deaths in the β-blocker group and 129 deaths in the placebo group. Patients who received β-blockers had a 27% increased risk for all-cause mortality (risk ratio [RR] = 1.27). The number needed to harm was 160.

Six of the studies also evaluated rates of nonfatal MI, nonfatal stroke, and hypotension. β-Blockers lowered the risk for nonfatal MI (RR = 0.73) but increased the risk for nonfatal stroke (RR = 1.73) and hypotension (RR = 1.51).

This meta-analysis was dominated by the 2008 Peri-Operative ISchemic Evaluation (POISE) trial, an RCT that compared placebo to extended-release metoprolol (100 mg 2 to 4 h before surgery, followed by 200 mg/d for 30 d), in 8,351 patients with, or at risk for, atherosclerotic disease.4 While β-blockers reduced the risk for MI and atrial fibrillation, they increased the risk for mortality and stroke, likely due to drug-induced hypotension. The slightly larger-than-typical doses of β-blockers used in this study may have contributed to the excess mortality.

What's new and challenges to implementation >> 

 

 

WHAT’S NEW
Avoiding β-blockers in surgery patients will prevent deaths

Bouri et al1 found that while β-blockers protect against nonfatal MIs, they increase the risk for nonfatal strokes and death. This new meta-analysis challenges the ACCF/AHA recommendations by suggesting that abandoning the use of β-blockers for preoperative patients who aren’t already taking them will prevent a substantial number of perioperative deaths. Bouri et al1 estimate that in the United Kingdom, where 47,286 deaths occur annually within 30 days of intermediate- or high-risk procedures, the number of iatrogenic deaths would drop by approximately 10,000 if β-blockers were not used.1

CAVEATS
Don’t stop β-blockers in patients who already take them

This meta-analysis did not evaluate outcomes in patients who were already taking β-blockers. These patients should continue to take them in the perioperative period, which is in line with current ACCF/AHA guidelines.

CHALLENGES TO IMPLEMENTATION
Reluctance to disregard published guidelines 
Some clinicians may not be comfortable ignoring the current ACCF/AHA guidelines that make a Class IIA recommendation (it is reasonable to administer this treatment) for the use of preoperative β-blockade for patients at risk for cardiovascular events who were not previously taking a β-blocker. This updated meta-analysis excludes the discredited DECREASE trials and challenges us to act against these current guidelines while we await updated recommendations.       

REFERENCES
1. Bouri S, Shun-Shin MJ, Cole GD, et al. Meta-analysis of secure randomised controlled trials of ß-blockade to prevent perioperative death in non-cardiac surgery. Heart. 2014;100:456-464.

2. American College of Cardiology Foundation/American Heart Association Task Force on Practice Guidelines; American Society of Echocardiography; American Society of Nuclear Cardiology; Heart Rhythm Society; Society of Cardiovascular Anesthesiologists; Society for Cardiovascular Angiography and Interventions; Society for Vascular Medicine; Society for Vascular Surgery; Fleisher LA, Beckman JA, Brown KA, et al. 2009 ACCF/AHA focused update on perioperative beta blockade incorporated into the ACC/AHA 2007 guidelines on perioperative cardiovascular evaluation and care for noncardiac surgery. J Am Coll Cardiol. 2009;54:e13-e118.

3. Erasmus Medical Center Follow-up Investigation Committee. Report on the 2012 follow-up investigation of possible breaches of academic integrity (September 30, 2012). CardioBrief. Available at: http://cardiobrief.files.wordpress.com/2012/10/integrity-report-2012-10-english-translation.pdf. Accessed August 14, 2014.

4. Devereaux PJ, Yang H, Yusuf S, et al; POISE Study Group. Effects of extended-release metoprolol succinate in patients undergoing non-cardiac surgery (POISE trial): a randomised controlled trial. Lancet. 2008;
371:1839-1847.

ACKNOWLEDGEMENT
The PURLs Surveillance System was supported in part by Grant Number UL1RR024999 from the National Center For Research Resources, a Clinical Translational Science Award to the University of Chicago. The content is solely the responsibility of the authors and does not necessarily represent the official views of the National Center For Research Resources or the National Institutes of Health.

Copyright © 2014. The Family Physicians Inquiries Network. All rights reserved.

Reprinted with permission from the Family Physicians Inquiries Network and The Journal of Family Practice. 2014;63(6):E15-E16.

PRACTICE CHANGER

Do not routinely initiate β-blockers in patients undergoing intermediate- or high-risk noncardiac surgery. β-Blockers appear to increase the 30-day risk for all-cause mortality.1

STRENGTH OF RECOMMENDATION

A: Based on meta-analysis of nine randomized controlled trials (RCTs).1

ILLUSTRATIVE CASE
A 67-year-old woman with diabetes, hypertension, and hyperlipidemia presents for evaluation prior to a total hip arthroplasty. She is not taking a β-blocker. Should you prescribe one?

Study summary >>

 

 

Current guidelines from the American College of Cardiology Foundation (ACCF) and the American Heart Association (AHA) recommend starting 

β-blockers to prevent cardiac events in patients about to undergo intermediate- or high-risk surgery or vascular surgery who have a history of inducible ischemia, coronary artery disease (CAD), or at least one risk factor for CAD.2 However, the majority of the evidence for these guidelines, which were published in 2009 and are in the process of being updated, came from the DECREASE (Dutch Echocardiographic Cardiac Risk Evaluation Applying Stress Echocardiography) trials. These trials  have been discredited due to serious methodologic flaws, including falsified descriptions of how outcomes were determined and fictitious databases.3

A new meta-analysis conducted by Bouri et al1 that excluded the DECREASE trials found that, although preoperative β-blockers reduce the rate of certain nonfatal outcomes, they increase the risk for death and stroke.

STUDY SUMMARY
Preop β-blockers do more harm than good

Bouri et al1 conducted a meta-analysis of published RCTs evaluating preoperative β-blockers versus placebo for patients undergoing noncardiac surgery. Of the 11 studies that met eligibility criteria, two were the discredited DECREASE trials. Thus, Bouri et al1 analyzed nine high-quality RCTs that included 10,529 patients.

Most studies included patients undergoing vascular surgery. Some studies also included intra-abdominal, intrathoracic, neurosurgic, orthopedic, urologic, and gynecologic surgeries. β-Blockers were started no more than a day before surgery and were discontinued at hospital discharge or up to 30 days postop. Metoprolol was used in five trials, bisoprolol in one trial, atenolol in two trials, and propranolol in one trial. The primary endpoint was all-cause mortality within 30 days.

A total of 5,264 patients were randomly assigned to receive β-blockers and 5,265 to placebo. There were 162 deaths in the β-blocker group and 129 deaths in the placebo group. Patients who received β-blockers had a 27% increased risk for all-cause mortality (risk ratio [RR] = 1.27). The number needed to harm was 160.

Six of the studies also evaluated rates of nonfatal MI, nonfatal stroke, and hypotension. β-Blockers lowered the risk for nonfatal MI (RR = 0.73) but increased the risk for nonfatal stroke (RR = 1.73) and hypotension (RR = 1.51).

This meta-analysis was dominated by the 2008 Peri-Operative ISchemic Evaluation (POISE) trial, an RCT that compared placebo to extended-release metoprolol (100 mg 2 to 4 h before surgery, followed by 200 mg/d for 30 d), in 8,351 patients with, or at risk for, atherosclerotic disease.4 While β-blockers reduced the risk for MI and atrial fibrillation, they increased the risk for mortality and stroke, likely due to drug-induced hypotension. The slightly larger-than-typical doses of β-blockers used in this study may have contributed to the excess mortality.

What's new and challenges to implementation >> 

 

 

WHAT’S NEW
Avoiding β-blockers in surgery patients will prevent deaths

Bouri et al1 found that while β-blockers protect against nonfatal MIs, they increase the risk for nonfatal strokes and death. This new meta-analysis challenges the ACCF/AHA recommendations by suggesting that abandoning the use of β-blockers for preoperative patients who aren’t already taking them will prevent a substantial number of perioperative deaths. Bouri et al1 estimate that in the United Kingdom, where 47,286 deaths occur annually within 30 days of intermediate- or high-risk procedures, the number of iatrogenic deaths would drop by approximately 10,000 if β-blockers were not used.1

CAVEATS
Don’t stop β-blockers in patients who already take them

This meta-analysis did not evaluate outcomes in patients who were already taking β-blockers. These patients should continue to take them in the perioperative period, which is in line with current ACCF/AHA guidelines.

CHALLENGES TO IMPLEMENTATION
Reluctance to disregard published guidelines 
Some clinicians may not be comfortable ignoring the current ACCF/AHA guidelines that make a Class IIA recommendation (it is reasonable to administer this treatment) for the use of preoperative β-blockade for patients at risk for cardiovascular events who were not previously taking a β-blocker. This updated meta-analysis excludes the discredited DECREASE trials and challenges us to act against these current guidelines while we await updated recommendations.       

REFERENCES
1. Bouri S, Shun-Shin MJ, Cole GD, et al. Meta-analysis of secure randomised controlled trials of ß-blockade to prevent perioperative death in non-cardiac surgery. Heart. 2014;100:456-464.

2. American College of Cardiology Foundation/American Heart Association Task Force on Practice Guidelines; American Society of Echocardiography; American Society of Nuclear Cardiology; Heart Rhythm Society; Society of Cardiovascular Anesthesiologists; Society for Cardiovascular Angiography and Interventions; Society for Vascular Medicine; Society for Vascular Surgery; Fleisher LA, Beckman JA, Brown KA, et al. 2009 ACCF/AHA focused update on perioperative beta blockade incorporated into the ACC/AHA 2007 guidelines on perioperative cardiovascular evaluation and care for noncardiac surgery. J Am Coll Cardiol. 2009;54:e13-e118.

3. Erasmus Medical Center Follow-up Investigation Committee. Report on the 2012 follow-up investigation of possible breaches of academic integrity (September 30, 2012). CardioBrief. Available at: http://cardiobrief.files.wordpress.com/2012/10/integrity-report-2012-10-english-translation.pdf. Accessed August 14, 2014.

4. Devereaux PJ, Yang H, Yusuf S, et al; POISE Study Group. Effects of extended-release metoprolol succinate in patients undergoing non-cardiac surgery (POISE trial): a randomised controlled trial. Lancet. 2008;
371:1839-1847.

ACKNOWLEDGEMENT
The PURLs Surveillance System was supported in part by Grant Number UL1RR024999 from the National Center For Research Resources, a Clinical Translational Science Award to the University of Chicago. The content is solely the responsibility of the authors and does not necessarily represent the official views of the National Center For Research Resources or the National Institutes of Health.

Copyright © 2014. The Family Physicians Inquiries Network. All rights reserved.

Reprinted with permission from the Family Physicians Inquiries Network and The Journal of Family Practice. 2014;63(6):E15-E16.

References

References

Issue
Clinician Reviews - 24(9)
Issue
Clinician Reviews - 24(9)
Page Number
21-22
Page Number
21-22
Publications
Publications
Topics
Article Type
Display Headline
Why You Shouldn’t Start β-Blockers Before Surgery
Display Headline
Why You Shouldn’t Start β-Blockers Before Surgery
Legacy Keywords
beta blockers, β-Blockers, surgery, non cardiac, noncardiac surgery, PURLs
Legacy Keywords
beta blockers, β-Blockers, surgery, non cardiac, noncardiac surgery, PURLs
Sections
Article Source

PURLs Copyright

Inside the Article

Article PDF Media

Suctioning Neonates at Birth: Time to Change Our Approach

Article Type
Changed
Tue, 12/13/2016 - 12:08
Display Headline
Suctioning Neonates at Birth: Time to Change Our Approach

PRACTICE CHANGER
Stop suctioning neonates at birth. There is no benefit to this practice, and it can cause bradycardia and apnea. Instead, wipe the baby’s mouth and nose with a towel to clear excess secretions and stimulate respiration.1

Strength of recommendation
B:
Based on a single randomized equivalency trial.

Kelleher J, Bhat, R, Salas AA, et al. Oronasopharyngeal suction versus wiping of the mouth and nose at birth: a randomised equivalency trial. Lancet. 2013;382:326-330.

Illustrative case
A healthy neonate is born through clear amniotic fluid with no meconium. She is vigorous and has no major congenital anomalies. Does she need oronasopharyngeal suctioning?

No, she does not need suctioning. Although it is still standard practice to perform oronasopharyngeal suctioning with a bulb syringe immediately after delivery, multiple studies have found no benefit to routine suctioning.2-7 Guidelines from the Neonatal Resuscitation Program (NRP) and other organizations recommend against the practice, even for neonates born through meconium-stained amniotic fluid.8,9 Suctioning is done because some clinicians believe it reduces the risk of aspiration, especially if there is meconium, and to stimulate breathing, but the evidence suggests that suctioning can stimulate the vagus nerve, which can lead to bradycardia.2 Studies that compared babies who did and didn’t receive suctioning found that those who received it had lower Apgar scores and oxygen saturation levels.2-4

Wiping the neonate’s mouth and nose with a towel is an alternative to suctioning, but until now no trials have compared the outcomes of these 2 methods. Kelleher et al1 conducted an equivalency trial to determine if wiping the mouth and nose is as effective as oronasopharyngeal suctioning.

STUDY SUMMARY: No difference in breathing 
after wiping or suctioning

Kelleher et al1 studied neonates born after at least 35 weeks gestation, excluding those who had major congenital anomalies or were non-vigorous (depressed muscle tone or respiration, heart rate <100 beats/min, or both) and born into meconium-stained amniotic fluid, as well as those whom they anticipated would need advanced resuscitation. Neonates were randomly assigned to receive either oronasopharyngeal suctioning with a bulb syringe or wiping of the face and mouth with a towel, starting immediately after the umbilical cord was cut and lasting as long as needed while in the delivery room. The primary outcome was the mean respiratory rate in the first 24 hours after birth. The predefined range of clinical equivalence between the 2 groups was a respiratory rate within 4 breaths/min.

Of 506 neonates randomized, 15 were excluded because they were not vigorous and had meconium-stained fluid, and 3 were excluded when their parents withdrew consent. Baseline characteristics for the 2 groups—including maternal age, presence of chronic medical conditions, and body mass index; vaginal vs cesarean delivery; umbilical artery pH; and neonatal sex, ethnic origin, and birth weight—were similar.

In the first 24 hours after birth, the average respiratory rate in the wiping group was 51 breaths/min (standard deviation [SD] ± 8) vs 50 breaths/min (SD ± 6) in the suctioning group. There was no difference in respiratory rates between the 2 groups at 1, 8, or 16 hours after birth. There was also no difference between the 2 groups in Apgar scores or need for advanced resuscitation. More neonates in the wiping group than in the suctioning group were admitted to the neonatal intensive care unit (45 of 246 [18%] vs 30 of 242 [12%]; P=.07), but the study was not powered to assess this outcome.

WHAT'S NEW: Wiping is as effective as suctioning,
 but there are no adverse effects

This study gives us evidence that wiping the face, mouth, and nose is equivalent to suctioning newborns at delivery, and it supports the NRP recommendation against routine suctioning in vigorous neonates born at term. Wiping avoids the potential adverse effects on the respiratory mucosa, bradycardia, and lower Apgar scores associated with suctioning via bulb syringes.

CAVEATS: Wiping is not best 
if a neonate’s airway is obstructed

This study looked only at neonates born after 35 weeks’ gestation who did not have meconium-stained amniotic fluid or congenital abnormalities. Also, NRP guidelines do recommend clearing the airways with a bulb syringe or suction catheter if airway obstruction is evident or positive-pressure ventilation is required.8

Another caveat ... In this study,1 there were 98 treatment crossovers: 64 of the 246 neonates in the wiping group received suctioning, and 34 of the 242 neonates in the suctioning group received wiping. However, this was not likely to change the study’s overall conclusion because a per-treatment analysis also found that wiping and suctioning were equivalent.

CHALLENGES TO IMPLEMENTATION: “We’ve always done it this way”
Practice patterns in a delivery room can be difficult to change. As we work on improving our delivery room environment and changing ingrained habits, the evidence from this study should help support the use of wiping in place of suctioning. The transition from suctioning to wiping also would be facilitated by having easily accessible towels designated for wiping.

 

 

Acknowledgement
The PURLs Surveillance System was supported in part by Grant Number UL1RR024999 from the National Center For Research Resources, a Clinical Translational Science Award to the University of Chicago. The content is solely the responsibility of the authors and does not necessarily represent the official views of the National Center For Research Resources or the National Institutes of Health.

PURL METHODOLOGY

…..

References


1. Kelleher J, Bhat R, Salas AA, et al. Oronasopharyngeal suction versus wiping of the mouth and nose at birth: a randomised equivalency trial. Lancet. 2013;382:326-330.

2. Gungor S, Kurt E, Teksoz E, et al. Oronasopharyngeal suction versus no suction in normal and term infants delivered by elective cesarean section: a prospective randomized controlled trial. Gynecol Obstet Invest. 2006;61:9-14.

3. Gungor S, Teksoz E, Ceyhan T, et al. Oronasopharyngeal suction versus no suction in normal, term and vaginally born infants: a prospective randomized controlled trial. Aust N Z J Obstet Gynaecol. 2005;45:453-456.

4. Carrasco M, Martell M, Estol PC. Oronasopharyngeal suction at birth: effects on arterial oxygen saturation. J Pediatr. 1997;130:832-834.

5. Estol PC, Piriz H, Basalo S, et al. Oro-naso-pharyngeal suction at birth: effects on respiratory adaptation of normal term vaginally born infants. J Perinat Med. 1992;20:297-305.

6. Wiswell TE, Gannon CM, Jacob J, et al. Delivery room management of the apparently vigorous meconium-stained neonate: results of the multicenter, international collaborative trial. Pediatrics. 2000;105(1 pt 1):1-7.

7. Vain NE, Szyld EG, Prudent LM, et al. Oropharyngeal and nasopharyngeal suctioning of meconium-stained neonates before delivery of their shoulders: multicentre, randomized controlled trial. Lancet. 2004;364:597-602.

8. Kattwinkel J, Perlman JM, Aziz K, et al. Part 15: neonatal resuscitation: 2010 American Heart Association guidelines for cardiopulmonary resuscitation and emergency cardiovascular care. Circulation. 2010;122(18 suppl 3):S909-S919.

9. Perlman JM, Wyllie J, Kattwinkel J, et al; Neonatal Resuscitation Chapter Collaborators. Neonatal resuscitation: 2010 International Consensus on Cardiopulmonary Resuscitation and Emergency Cardiovascular Care Science with Treatment Recommendations. Pediatrics. 2010;126:e1319-1344.

Article PDF
Author and Disclosure Information

Iliana Neumann, MD
Anne Mounsey, MD
Niladri Das, MD
Department of Family Medicine, University of North Carolina at Chapel Hill (Drs. Neumann and Mounsey); Family Medicine Residency Program, University of Pittsburgh St. Margaret, Pa (Dr. Das)

PURLs EDITOR
James J. Stevermer, MD, MSPH

Department of Family and Community Medicine, University of Missouri-Columbia

Issue
Clinician Reviews - 24(8)
Publications
Topics
Page Number
461-462
Legacy Keywords
respiratory; obstetrics; pediatrics; Iliana Neumann; MD; Anne Mounsey; MD; Niladri Das; MD; bradycardia; apnea; Neonatal Resuscitation Program; oronasopharyngeal suctioning; bulb syringe; meconium
Sections
Author and Disclosure Information

Iliana Neumann, MD
Anne Mounsey, MD
Niladri Das, MD
Department of Family Medicine, University of North Carolina at Chapel Hill (Drs. Neumann and Mounsey); Family Medicine Residency Program, University of Pittsburgh St. Margaret, Pa (Dr. Das)

PURLs EDITOR
James J. Stevermer, MD, MSPH

Department of Family and Community Medicine, University of Missouri-Columbia

Author and Disclosure Information

Iliana Neumann, MD
Anne Mounsey, MD
Niladri Das, MD
Department of Family Medicine, University of North Carolina at Chapel Hill (Drs. Neumann and Mounsey); Family Medicine Residency Program, University of Pittsburgh St. Margaret, Pa (Dr. Das)

PURLs EDITOR
James J. Stevermer, MD, MSPH

Department of Family and Community Medicine, University of Missouri-Columbia

Article PDF
Article PDF

PRACTICE CHANGER
Stop suctioning neonates at birth. There is no benefit to this practice, and it can cause bradycardia and apnea. Instead, wipe the baby’s mouth and nose with a towel to clear excess secretions and stimulate respiration.1

Strength of recommendation
B:
Based on a single randomized equivalency trial.

Kelleher J, Bhat, R, Salas AA, et al. Oronasopharyngeal suction versus wiping of the mouth and nose at birth: a randomised equivalency trial. Lancet. 2013;382:326-330.

Illustrative case
A healthy neonate is born through clear amniotic fluid with no meconium. She is vigorous and has no major congenital anomalies. Does she need oronasopharyngeal suctioning?

No, she does not need suctioning. Although it is still standard practice to perform oronasopharyngeal suctioning with a bulb syringe immediately after delivery, multiple studies have found no benefit to routine suctioning.2-7 Guidelines from the Neonatal Resuscitation Program (NRP) and other organizations recommend against the practice, even for neonates born through meconium-stained amniotic fluid.8,9 Suctioning is done because some clinicians believe it reduces the risk of aspiration, especially if there is meconium, and to stimulate breathing, but the evidence suggests that suctioning can stimulate the vagus nerve, which can lead to bradycardia.2 Studies that compared babies who did and didn’t receive suctioning found that those who received it had lower Apgar scores and oxygen saturation levels.2-4

Wiping the neonate’s mouth and nose with a towel is an alternative to suctioning, but until now no trials have compared the outcomes of these 2 methods. Kelleher et al1 conducted an equivalency trial to determine if wiping the mouth and nose is as effective as oronasopharyngeal suctioning.

STUDY SUMMARY: No difference in breathing 
after wiping or suctioning

Kelleher et al1 studied neonates born after at least 35 weeks gestation, excluding those who had major congenital anomalies or were non-vigorous (depressed muscle tone or respiration, heart rate <100 beats/min, or both) and born into meconium-stained amniotic fluid, as well as those whom they anticipated would need advanced resuscitation. Neonates were randomly assigned to receive either oronasopharyngeal suctioning with a bulb syringe or wiping of the face and mouth with a towel, starting immediately after the umbilical cord was cut and lasting as long as needed while in the delivery room. The primary outcome was the mean respiratory rate in the first 24 hours after birth. The predefined range of clinical equivalence between the 2 groups was a respiratory rate within 4 breaths/min.

Of 506 neonates randomized, 15 were excluded because they were not vigorous and had meconium-stained fluid, and 3 were excluded when their parents withdrew consent. Baseline characteristics for the 2 groups—including maternal age, presence of chronic medical conditions, and body mass index; vaginal vs cesarean delivery; umbilical artery pH; and neonatal sex, ethnic origin, and birth weight—were similar.

In the first 24 hours after birth, the average respiratory rate in the wiping group was 51 breaths/min (standard deviation [SD] ± 8) vs 50 breaths/min (SD ± 6) in the suctioning group. There was no difference in respiratory rates between the 2 groups at 1, 8, or 16 hours after birth. There was also no difference between the 2 groups in Apgar scores or need for advanced resuscitation. More neonates in the wiping group than in the suctioning group were admitted to the neonatal intensive care unit (45 of 246 [18%] vs 30 of 242 [12%]; P=.07), but the study was not powered to assess this outcome.

WHAT'S NEW: Wiping is as effective as suctioning,
 but there are no adverse effects

This study gives us evidence that wiping the face, mouth, and nose is equivalent to suctioning newborns at delivery, and it supports the NRP recommendation against routine suctioning in vigorous neonates born at term. Wiping avoids the potential adverse effects on the respiratory mucosa, bradycardia, and lower Apgar scores associated with suctioning via bulb syringes.

CAVEATS: Wiping is not best 
if a neonate’s airway is obstructed

This study looked only at neonates born after 35 weeks’ gestation who did not have meconium-stained amniotic fluid or congenital abnormalities. Also, NRP guidelines do recommend clearing the airways with a bulb syringe or suction catheter if airway obstruction is evident or positive-pressure ventilation is required.8

Another caveat ... In this study,1 there were 98 treatment crossovers: 64 of the 246 neonates in the wiping group received suctioning, and 34 of the 242 neonates in the suctioning group received wiping. However, this was not likely to change the study’s overall conclusion because a per-treatment analysis also found that wiping and suctioning were equivalent.

CHALLENGES TO IMPLEMENTATION: “We’ve always done it this way”
Practice patterns in a delivery room can be difficult to change. As we work on improving our delivery room environment and changing ingrained habits, the evidence from this study should help support the use of wiping in place of suctioning. The transition from suctioning to wiping also would be facilitated by having easily accessible towels designated for wiping.

 

 

Acknowledgement
The PURLs Surveillance System was supported in part by Grant Number UL1RR024999 from the National Center For Research Resources, a Clinical Translational Science Award to the University of Chicago. The content is solely the responsibility of the authors and does not necessarily represent the official views of the National Center For Research Resources or the National Institutes of Health.

PURL METHODOLOGY

…..

PRACTICE CHANGER
Stop suctioning neonates at birth. There is no benefit to this practice, and it can cause bradycardia and apnea. Instead, wipe the baby’s mouth and nose with a towel to clear excess secretions and stimulate respiration.1

Strength of recommendation
B:
Based on a single randomized equivalency trial.

Kelleher J, Bhat, R, Salas AA, et al. Oronasopharyngeal suction versus wiping of the mouth and nose at birth: a randomised equivalency trial. Lancet. 2013;382:326-330.

Illustrative case
A healthy neonate is born through clear amniotic fluid with no meconium. She is vigorous and has no major congenital anomalies. Does she need oronasopharyngeal suctioning?

No, she does not need suctioning. Although it is still standard practice to perform oronasopharyngeal suctioning with a bulb syringe immediately after delivery, multiple studies have found no benefit to routine suctioning.2-7 Guidelines from the Neonatal Resuscitation Program (NRP) and other organizations recommend against the practice, even for neonates born through meconium-stained amniotic fluid.8,9 Suctioning is done because some clinicians believe it reduces the risk of aspiration, especially if there is meconium, and to stimulate breathing, but the evidence suggests that suctioning can stimulate the vagus nerve, which can lead to bradycardia.2 Studies that compared babies who did and didn’t receive suctioning found that those who received it had lower Apgar scores and oxygen saturation levels.2-4

Wiping the neonate’s mouth and nose with a towel is an alternative to suctioning, but until now no trials have compared the outcomes of these 2 methods. Kelleher et al1 conducted an equivalency trial to determine if wiping the mouth and nose is as effective as oronasopharyngeal suctioning.

STUDY SUMMARY: No difference in breathing 
after wiping or suctioning

Kelleher et al1 studied neonates born after at least 35 weeks gestation, excluding those who had major congenital anomalies or were non-vigorous (depressed muscle tone or respiration, heart rate <100 beats/min, or both) and born into meconium-stained amniotic fluid, as well as those whom they anticipated would need advanced resuscitation. Neonates were randomly assigned to receive either oronasopharyngeal suctioning with a bulb syringe or wiping of the face and mouth with a towel, starting immediately after the umbilical cord was cut and lasting as long as needed while in the delivery room. The primary outcome was the mean respiratory rate in the first 24 hours after birth. The predefined range of clinical equivalence between the 2 groups was a respiratory rate within 4 breaths/min.

Of 506 neonates randomized, 15 were excluded because they were not vigorous and had meconium-stained fluid, and 3 were excluded when their parents withdrew consent. Baseline characteristics for the 2 groups—including maternal age, presence of chronic medical conditions, and body mass index; vaginal vs cesarean delivery; umbilical artery pH; and neonatal sex, ethnic origin, and birth weight—were similar.

In the first 24 hours after birth, the average respiratory rate in the wiping group was 51 breaths/min (standard deviation [SD] ± 8) vs 50 breaths/min (SD ± 6) in the suctioning group. There was no difference in respiratory rates between the 2 groups at 1, 8, or 16 hours after birth. There was also no difference between the 2 groups in Apgar scores or need for advanced resuscitation. More neonates in the wiping group than in the suctioning group were admitted to the neonatal intensive care unit (45 of 246 [18%] vs 30 of 242 [12%]; P=.07), but the study was not powered to assess this outcome.

WHAT'S NEW: Wiping is as effective as suctioning,
 but there are no adverse effects

This study gives us evidence that wiping the face, mouth, and nose is equivalent to suctioning newborns at delivery, and it supports the NRP recommendation against routine suctioning in vigorous neonates born at term. Wiping avoids the potential adverse effects on the respiratory mucosa, bradycardia, and lower Apgar scores associated with suctioning via bulb syringes.

CAVEATS: Wiping is not best 
if a neonate’s airway is obstructed

This study looked only at neonates born after 35 weeks’ gestation who did not have meconium-stained amniotic fluid or congenital abnormalities. Also, NRP guidelines do recommend clearing the airways with a bulb syringe or suction catheter if airway obstruction is evident or positive-pressure ventilation is required.8

Another caveat ... In this study,1 there were 98 treatment crossovers: 64 of the 246 neonates in the wiping group received suctioning, and 34 of the 242 neonates in the suctioning group received wiping. However, this was not likely to change the study’s overall conclusion because a per-treatment analysis also found that wiping and suctioning were equivalent.

CHALLENGES TO IMPLEMENTATION: “We’ve always done it this way”
Practice patterns in a delivery room can be difficult to change. As we work on improving our delivery room environment and changing ingrained habits, the evidence from this study should help support the use of wiping in place of suctioning. The transition from suctioning to wiping also would be facilitated by having easily accessible towels designated for wiping.

 

 

Acknowledgement
The PURLs Surveillance System was supported in part by Grant Number UL1RR024999 from the National Center For Research Resources, a Clinical Translational Science Award to the University of Chicago. The content is solely the responsibility of the authors and does not necessarily represent the official views of the National Center For Research Resources or the National Institutes of Health.

PURL METHODOLOGY

…..

References


1. Kelleher J, Bhat R, Salas AA, et al. Oronasopharyngeal suction versus wiping of the mouth and nose at birth: a randomised equivalency trial. Lancet. 2013;382:326-330.

2. Gungor S, Kurt E, Teksoz E, et al. Oronasopharyngeal suction versus no suction in normal and term infants delivered by elective cesarean section: a prospective randomized controlled trial. Gynecol Obstet Invest. 2006;61:9-14.

3. Gungor S, Teksoz E, Ceyhan T, et al. Oronasopharyngeal suction versus no suction in normal, term and vaginally born infants: a prospective randomized controlled trial. Aust N Z J Obstet Gynaecol. 2005;45:453-456.

4. Carrasco M, Martell M, Estol PC. Oronasopharyngeal suction at birth: effects on arterial oxygen saturation. J Pediatr. 1997;130:832-834.

5. Estol PC, Piriz H, Basalo S, et al. Oro-naso-pharyngeal suction at birth: effects on respiratory adaptation of normal term vaginally born infants. J Perinat Med. 1992;20:297-305.

6. Wiswell TE, Gannon CM, Jacob J, et al. Delivery room management of the apparently vigorous meconium-stained neonate: results of the multicenter, international collaborative trial. Pediatrics. 2000;105(1 pt 1):1-7.

7. Vain NE, Szyld EG, Prudent LM, et al. Oropharyngeal and nasopharyngeal suctioning of meconium-stained neonates before delivery of their shoulders: multicentre, randomized controlled trial. Lancet. 2004;364:597-602.

8. Kattwinkel J, Perlman JM, Aziz K, et al. Part 15: neonatal resuscitation: 2010 American Heart Association guidelines for cardiopulmonary resuscitation and emergency cardiovascular care. Circulation. 2010;122(18 suppl 3):S909-S919.

9. Perlman JM, Wyllie J, Kattwinkel J, et al; Neonatal Resuscitation Chapter Collaborators. Neonatal resuscitation: 2010 International Consensus on Cardiopulmonary Resuscitation and Emergency Cardiovascular Care Science with Treatment Recommendations. Pediatrics. 2010;126:e1319-1344.

References


1. Kelleher J, Bhat R, Salas AA, et al. Oronasopharyngeal suction versus wiping of the mouth and nose at birth: a randomised equivalency trial. Lancet. 2013;382:326-330.

2. Gungor S, Kurt E, Teksoz E, et al. Oronasopharyngeal suction versus no suction in normal and term infants delivered by elective cesarean section: a prospective randomized controlled trial. Gynecol Obstet Invest. 2006;61:9-14.

3. Gungor S, Teksoz E, Ceyhan T, et al. Oronasopharyngeal suction versus no suction in normal, term and vaginally born infants: a prospective randomized controlled trial. Aust N Z J Obstet Gynaecol. 2005;45:453-456.

4. Carrasco M, Martell M, Estol PC. Oronasopharyngeal suction at birth: effects on arterial oxygen saturation. J Pediatr. 1997;130:832-834.

5. Estol PC, Piriz H, Basalo S, et al. Oro-naso-pharyngeal suction at birth: effects on respiratory adaptation of normal term vaginally born infants. J Perinat Med. 1992;20:297-305.

6. Wiswell TE, Gannon CM, Jacob J, et al. Delivery room management of the apparently vigorous meconium-stained neonate: results of the multicenter, international collaborative trial. Pediatrics. 2000;105(1 pt 1):1-7.

7. Vain NE, Szyld EG, Prudent LM, et al. Oropharyngeal and nasopharyngeal suctioning of meconium-stained neonates before delivery of their shoulders: multicentre, randomized controlled trial. Lancet. 2004;364:597-602.

8. Kattwinkel J, Perlman JM, Aziz K, et al. Part 15: neonatal resuscitation: 2010 American Heart Association guidelines for cardiopulmonary resuscitation and emergency cardiovascular care. Circulation. 2010;122(18 suppl 3):S909-S919.

9. Perlman JM, Wyllie J, Kattwinkel J, et al; Neonatal Resuscitation Chapter Collaborators. Neonatal resuscitation: 2010 International Consensus on Cardiopulmonary Resuscitation and Emergency Cardiovascular Care Science with Treatment Recommendations. Pediatrics. 2010;126:e1319-1344.

Issue
Clinician Reviews - 24(8)
Issue
Clinician Reviews - 24(8)
Page Number
461-462
Page Number
461-462
Publications
Publications
Topics
Article Type
Display Headline
Suctioning Neonates at Birth: Time to Change Our Approach
Display Headline
Suctioning Neonates at Birth: Time to Change Our Approach
Legacy Keywords
respiratory; obstetrics; pediatrics; Iliana Neumann; MD; Anne Mounsey; MD; Niladri Das; MD; bradycardia; apnea; Neonatal Resuscitation Program; oronasopharyngeal suctioning; bulb syringe; meconium
Legacy Keywords
respiratory; obstetrics; pediatrics; Iliana Neumann; MD; Anne Mounsey; MD; Niladri Das; MD; bradycardia; apnea; Neonatal Resuscitation Program; oronasopharyngeal suctioning; bulb syringe; meconium
Sections
Article Source

PURLs Copyright

Copyright © 2014 Family Physicians Inquiries Network. All rights reserved.

Inside the Article

Article PDF Media

Suctioning neonates at birth: Time to change our approach

Article Type
Changed
Mon, 07/13/2020 - 11:46
Display Headline
Suctioning neonates at birth: Time to change our approach

 

PRACTICE CHANGER

Stop suctioning neonates at birth. There is no benefit to this practice, and it can cause bradycardia and apnea. Instead, wipe the baby’s mouth and nose with a towel to clear excess secretions and stimulate respiration.1

Strength of recommendation

B: Based on a single randomized equivalency trial.

Kelleher J, Bhat, R, Salas AA, et al. Oronasopharyngeal suction versus wiping of the mouth and nose at birth: a randomised equivalency trial. Lancet. 2013;382:326-330.

Illustrative case

A healthy neonate is born through clear amniotic fluid with no meconium. She is vigorous and has no major congenital anomalies. Does she need oronasopharyngeal suctioning?

No, she does not need suctioning. Although it is still standard practice to perform oronasopharyngeal suctioning with a bulb syringe immediately after delivery, multiple studies have found no benefit to routine suctioning.2-7 Guidelines from the Neonatal Resuscitation Program (NRP) and other organizations recommend against the practice, even for neonates born through meconium-stained amniotic fluid.8,9 Suctioning is done because some clinicians believe it reduces the risk of aspiration, especially if there is meconium, and to stimulate breathing, but the evidence suggests that suctioning can stimulate the vagus nerve, which can lead to bradycardia.2 Studies that compared babies who did and didn’t receive suctioning found that those who received it had lower Apgar scores and oxygen saturation levels.2-4

Wiping the neonate’s mouth and nose with a towel is an alternative to suctioning, but until now no trials have compared the outcomes of these 2 methods. Kelleher et al1 conducted an equivalency trial to determine if wiping the mouth and nose is as effective as oronasopharyngeal suctioning.

STUDY SUMMARY: No difference in breathing 
after wiping or suctioning


Kelleher et al1 studied neonates born after at least 35 weeks gestation, excluding those who had major congenital anomalies or were non-vigorous (depressed muscle tone or respiration, heart rate <100 beats/min, or both) and born into meconium-stained amniotic fluid, as well as those whom they anticipated would need advanced resuscitation. Neonates were randomly assigned to receive either oronasopharyngeal suctioning with a bulb syringe or wiping of the face and mouth with a towel, starting immediately after the umbilical cord was cut and lasting as long as needed while in the delivery room. The primary outcome was the mean respiratory rate in the first 24 hours after birth. The predefined range of clinical equivalence between the 2 groups was a respiratory rate within 4 breaths/min.

Of 506 neonates randomized, 15 were excluded because they were not vigorous and had meconium-stained fluid, and 3 were excluded when their parents withdrew consent. Baseline characteristics for the 2 groups—including maternal age, presence of chronic medical conditions, and body mass index; vaginal vs cesarean delivery; umbilical artery pH; and neonatal sex, ethnic origin, and birth weight—were similar.

There was no difference in respiratory rates between the suctioning and wiping groups within the first 24 hours of birth. In the first 24 hours after birth, the average respiratory rate in the wiping group was 51 breaths/min (standard deviation [SD] ± 8) vs 50 breaths/min (SD ± 6) in the suctioning group. There was no difference in respiratory rates between the 2 groups at 1, 8, or 16 hours after birth. There was also no difference between the 2 groups in Apgar scores or need for advanced resuscitation. More neonates in the wiping group than in the suctioning group were admitted to the neonatal intensive care unit (45 of 246 [18%] vs 30 of 242 [12%]; P=.07), but the study was not powered to assess this outcome.

WHAT'S NEW: Wiping is as effective as suctioning,
 but there are no adverse effects


This study gives us evidence that wiping the face, mouth, and nose is equivalent to suctioning newborns at delivery, and it supports the NRP recommendation against routine suctioning in vigorous neonates born at term. Wiping avoids the potential adverse effects on the respiratory mucosa, bradycardia, and lower Apgar scores associated with suctioning via bulb syringes.

CAVEATS: Wiping is not best 
if a neonate’s airway is obstructed


This study looked only at neonates born after 35 weeks’ gestation who did not have meconium-stained amniotic fluid or congenital abnormalities. Also, NRP guidelines do recommend clearing the airways with a bulb syringe or suction catheter if airway obstruction is evident or positive-pressure ventilation is required.8

Another caveat ... In this study,1 there were 98 treatment crossovers: 64 of the 246 neonates in the wiping group received suctioning, and 34 of the 242 neonates in the suctioning group received wiping. However, this was not likely to change the study’s overall conclusion because a per-treatment analysis also found that wiping and suctioning were equivalent.

 

 

CHALLENGES TO IMPLEMENTATION: “We’ve always done it this way”

Practice patterns in a delivery room can be difficult to change. As we work on improving our delivery room environment and changing ingrained habits, the evidence from this study should help support the use of wiping in place of suctioning. The transition from suctioning to wiping also would be facilitated by having easily accessible towels designated for wiping.

Acknowledgement
The PURLs Surveillance System was supported in part by Grant Number UL1RR024999 from the National Center For Research Resources, a Clinical Translational Science Award to the University of Chicago. The content is solely the responsibility of the authors and does not necessarily represent the official views of the National Center For Research Resources or the National Institutes of Health.

Files
References

 

1. Kelleher J, Bhat R, Salas AA, et al. Oronasopharyngeal suction versus wiping of the mouth and nose at birth: a randomised equivalency trial. Lancet. 2013;382:326-330.

2. Gungor S, Kurt E, Teksoz E, et al. Oronasopharyngeal suction versus no suction in normal and term infants delivered by elective cesarean section: a prospective randomized controlled trial. Gynecol Obstet Invest. 2006;61:9-14.

3. Gungor S, Teksoz E, Ceyhan T, et al. Oronasopharyngeal suction versus no suction in normal, term and vaginally born infants: a prospective randomized controlled trial. Aust N Z J Obstet Gynaecol. 2005;45:453-456.

4. Carrasco M, Martell M, Estol PC. Oronasopharyngeal suction at birth: effects on arterial oxygen saturation. J Pediatr. 1997;130:832-834.

5. Estol PC, Piriz H, Basalo S, et al. Oro-naso-pharyngeal suction at birth: effects on respiratory adaptation of normal term vaginally born infants. J Perinat Med. 1992;20:297-305.

6. Wiswell TE, Gannon CM, Jacob J, et al. Delivery room management of the apparently vigorous meconium-stained neonate: results of the multicenter, international collaborative trial. Pediatrics. 2000;105(1 pt 1):1-7.

7. Vain NE, Szyld EG, Prudent LM, et al. Oropharyngeal and nasopharyngeal suctioning of meconium-stained neonates before delivery of their shoulders: multicentre, randomized controlled trial. Lancet. 2004;364:597-602.

8. Kattwinkel J, Perlman JM, Aziz K, et al. Part 15: neonatal resuscitation: 2010 American Heart Association guidelines for cardiopulmonary resuscitation and emergency cardiovascular care. Circulation. 2010;122(18 suppl 3):S909-S919.

9. Perlman JM, Wyllie J, Kattwinkel J, et al; Neonatal Resuscitation Chapter Collaborators. Neonatal resuscitation: 2010 International Consensus on Cardiopulmonary Resuscitation and Emergency Cardiovascular Care Science with Treatment Recommendations. Pediatrics. 2010;126:e1319-1344.

Article PDF
Author and Disclosure Information

 

Iliana Neumann, MD
Anne Mounsey, MD
Niladri Das, MD

Department of Family Medicine, University of North Carolina at Chapel Hill (Drs. Neumann and Mounsey); Family Medicine Residency Program, University of Pittsburgh St. Margaret, Pa (Dr. Das)

PURLs EDITOR
James J. Stevermer, MD, MSPH
Department of Family and Community Medicine, University of Missouri-Columbia

Issue
The Journal of Family Practice - 63(8)
Publications
Topics
Page Number
461-462
Legacy Keywords
respiratory; obstetrics; pediatrics; Iliana Neumann; MD; Anne Mounsey; MD; Niladri Das; MD; bradycardia; apnea; Neonatal Resuscitation Program; oronasopharyngeal suctioning; bulb syringe; meconium
Sections
Files
Files
Author and Disclosure Information

 

Iliana Neumann, MD
Anne Mounsey, MD
Niladri Das, MD

Department of Family Medicine, University of North Carolina at Chapel Hill (Drs. Neumann and Mounsey); Family Medicine Residency Program, University of Pittsburgh St. Margaret, Pa (Dr. Das)

PURLs EDITOR
James J. Stevermer, MD, MSPH
Department of Family and Community Medicine, University of Missouri-Columbia

Author and Disclosure Information

 

Iliana Neumann, MD
Anne Mounsey, MD
Niladri Das, MD

Department of Family Medicine, University of North Carolina at Chapel Hill (Drs. Neumann and Mounsey); Family Medicine Residency Program, University of Pittsburgh St. Margaret, Pa (Dr. Das)

PURLs EDITOR
James J. Stevermer, MD, MSPH
Department of Family and Community Medicine, University of Missouri-Columbia

Article PDF
Article PDF

 

PRACTICE CHANGER

Stop suctioning neonates at birth. There is no benefit to this practice, and it can cause bradycardia and apnea. Instead, wipe the baby’s mouth and nose with a towel to clear excess secretions and stimulate respiration.1

Strength of recommendation

B: Based on a single randomized equivalency trial.

Kelleher J, Bhat, R, Salas AA, et al. Oronasopharyngeal suction versus wiping of the mouth and nose at birth: a randomised equivalency trial. Lancet. 2013;382:326-330.

Illustrative case

A healthy neonate is born through clear amniotic fluid with no meconium. She is vigorous and has no major congenital anomalies. Does she need oronasopharyngeal suctioning?

No, she does not need suctioning. Although it is still standard practice to perform oronasopharyngeal suctioning with a bulb syringe immediately after delivery, multiple studies have found no benefit to routine suctioning.2-7 Guidelines from the Neonatal Resuscitation Program (NRP) and other organizations recommend against the practice, even for neonates born through meconium-stained amniotic fluid.8,9 Suctioning is done because some clinicians believe it reduces the risk of aspiration, especially if there is meconium, and to stimulate breathing, but the evidence suggests that suctioning can stimulate the vagus nerve, which can lead to bradycardia.2 Studies that compared babies who did and didn’t receive suctioning found that those who received it had lower Apgar scores and oxygen saturation levels.2-4

Wiping the neonate’s mouth and nose with a towel is an alternative to suctioning, but until now no trials have compared the outcomes of these 2 methods. Kelleher et al1 conducted an equivalency trial to determine if wiping the mouth and nose is as effective as oronasopharyngeal suctioning.

STUDY SUMMARY: No difference in breathing 
after wiping or suctioning


Kelleher et al1 studied neonates born after at least 35 weeks gestation, excluding those who had major congenital anomalies or were non-vigorous (depressed muscle tone or respiration, heart rate <100 beats/min, or both) and born into meconium-stained amniotic fluid, as well as those whom they anticipated would need advanced resuscitation. Neonates were randomly assigned to receive either oronasopharyngeal suctioning with a bulb syringe or wiping of the face and mouth with a towel, starting immediately after the umbilical cord was cut and lasting as long as needed while in the delivery room. The primary outcome was the mean respiratory rate in the first 24 hours after birth. The predefined range of clinical equivalence between the 2 groups was a respiratory rate within 4 breaths/min.

Of 506 neonates randomized, 15 were excluded because they were not vigorous and had meconium-stained fluid, and 3 were excluded when their parents withdrew consent. Baseline characteristics for the 2 groups—including maternal age, presence of chronic medical conditions, and body mass index; vaginal vs cesarean delivery; umbilical artery pH; and neonatal sex, ethnic origin, and birth weight—were similar.

There was no difference in respiratory rates between the suctioning and wiping groups within the first 24 hours of birth. In the first 24 hours after birth, the average respiratory rate in the wiping group was 51 breaths/min (standard deviation [SD] ± 8) vs 50 breaths/min (SD ± 6) in the suctioning group. There was no difference in respiratory rates between the 2 groups at 1, 8, or 16 hours after birth. There was also no difference between the 2 groups in Apgar scores or need for advanced resuscitation. More neonates in the wiping group than in the suctioning group were admitted to the neonatal intensive care unit (45 of 246 [18%] vs 30 of 242 [12%]; P=.07), but the study was not powered to assess this outcome.

WHAT'S NEW: Wiping is as effective as suctioning,
 but there are no adverse effects


This study gives us evidence that wiping the face, mouth, and nose is equivalent to suctioning newborns at delivery, and it supports the NRP recommendation against routine suctioning in vigorous neonates born at term. Wiping avoids the potential adverse effects on the respiratory mucosa, bradycardia, and lower Apgar scores associated with suctioning via bulb syringes.

CAVEATS: Wiping is not best 
if a neonate’s airway is obstructed


This study looked only at neonates born after 35 weeks’ gestation who did not have meconium-stained amniotic fluid or congenital abnormalities. Also, NRP guidelines do recommend clearing the airways with a bulb syringe or suction catheter if airway obstruction is evident or positive-pressure ventilation is required.8

Another caveat ... In this study,1 there were 98 treatment crossovers: 64 of the 246 neonates in the wiping group received suctioning, and 34 of the 242 neonates in the suctioning group received wiping. However, this was not likely to change the study’s overall conclusion because a per-treatment analysis also found that wiping and suctioning were equivalent.

 

 

CHALLENGES TO IMPLEMENTATION: “We’ve always done it this way”

Practice patterns in a delivery room can be difficult to change. As we work on improving our delivery room environment and changing ingrained habits, the evidence from this study should help support the use of wiping in place of suctioning. The transition from suctioning to wiping also would be facilitated by having easily accessible towels designated for wiping.

Acknowledgement
The PURLs Surveillance System was supported in part by Grant Number UL1RR024999 from the National Center For Research Resources, a Clinical Translational Science Award to the University of Chicago. The content is solely the responsibility of the authors and does not necessarily represent the official views of the National Center For Research Resources or the National Institutes of Health.

 

PRACTICE CHANGER

Stop suctioning neonates at birth. There is no benefit to this practice, and it can cause bradycardia and apnea. Instead, wipe the baby’s mouth and nose with a towel to clear excess secretions and stimulate respiration.1

Strength of recommendation

B: Based on a single randomized equivalency trial.

Kelleher J, Bhat, R, Salas AA, et al. Oronasopharyngeal suction versus wiping of the mouth and nose at birth: a randomised equivalency trial. Lancet. 2013;382:326-330.

Illustrative case

A healthy neonate is born through clear amniotic fluid with no meconium. She is vigorous and has no major congenital anomalies. Does she need oronasopharyngeal suctioning?

No, she does not need suctioning. Although it is still standard practice to perform oronasopharyngeal suctioning with a bulb syringe immediately after delivery, multiple studies have found no benefit to routine suctioning.2-7 Guidelines from the Neonatal Resuscitation Program (NRP) and other organizations recommend against the practice, even for neonates born through meconium-stained amniotic fluid.8,9 Suctioning is done because some clinicians believe it reduces the risk of aspiration, especially if there is meconium, and to stimulate breathing, but the evidence suggests that suctioning can stimulate the vagus nerve, which can lead to bradycardia.2 Studies that compared babies who did and didn’t receive suctioning found that those who received it had lower Apgar scores and oxygen saturation levels.2-4

Wiping the neonate’s mouth and nose with a towel is an alternative to suctioning, but until now no trials have compared the outcomes of these 2 methods. Kelleher et al1 conducted an equivalency trial to determine if wiping the mouth and nose is as effective as oronasopharyngeal suctioning.

STUDY SUMMARY: No difference in breathing 
after wiping or suctioning


Kelleher et al1 studied neonates born after at least 35 weeks gestation, excluding those who had major congenital anomalies or were non-vigorous (depressed muscle tone or respiration, heart rate <100 beats/min, or both) and born into meconium-stained amniotic fluid, as well as those whom they anticipated would need advanced resuscitation. Neonates were randomly assigned to receive either oronasopharyngeal suctioning with a bulb syringe or wiping of the face and mouth with a towel, starting immediately after the umbilical cord was cut and lasting as long as needed while in the delivery room. The primary outcome was the mean respiratory rate in the first 24 hours after birth. The predefined range of clinical equivalence between the 2 groups was a respiratory rate within 4 breaths/min.

Of 506 neonates randomized, 15 were excluded because they were not vigorous and had meconium-stained fluid, and 3 were excluded when their parents withdrew consent. Baseline characteristics for the 2 groups—including maternal age, presence of chronic medical conditions, and body mass index; vaginal vs cesarean delivery; umbilical artery pH; and neonatal sex, ethnic origin, and birth weight—were similar.

There was no difference in respiratory rates between the suctioning and wiping groups within the first 24 hours of birth. In the first 24 hours after birth, the average respiratory rate in the wiping group was 51 breaths/min (standard deviation [SD] ± 8) vs 50 breaths/min (SD ± 6) in the suctioning group. There was no difference in respiratory rates between the 2 groups at 1, 8, or 16 hours after birth. There was also no difference between the 2 groups in Apgar scores or need for advanced resuscitation. More neonates in the wiping group than in the suctioning group were admitted to the neonatal intensive care unit (45 of 246 [18%] vs 30 of 242 [12%]; P=.07), but the study was not powered to assess this outcome.

WHAT'S NEW: Wiping is as effective as suctioning,
 but there are no adverse effects


This study gives us evidence that wiping the face, mouth, and nose is equivalent to suctioning newborns at delivery, and it supports the NRP recommendation against routine suctioning in vigorous neonates born at term. Wiping avoids the potential adverse effects on the respiratory mucosa, bradycardia, and lower Apgar scores associated with suctioning via bulb syringes.

CAVEATS: Wiping is not best 
if a neonate’s airway is obstructed


This study looked only at neonates born after 35 weeks’ gestation who did not have meconium-stained amniotic fluid or congenital abnormalities. Also, NRP guidelines do recommend clearing the airways with a bulb syringe or suction catheter if airway obstruction is evident or positive-pressure ventilation is required.8

Another caveat ... In this study,1 there were 98 treatment crossovers: 64 of the 246 neonates in the wiping group received suctioning, and 34 of the 242 neonates in the suctioning group received wiping. However, this was not likely to change the study’s overall conclusion because a per-treatment analysis also found that wiping and suctioning were equivalent.

 

 

CHALLENGES TO IMPLEMENTATION: “We’ve always done it this way”

Practice patterns in a delivery room can be difficult to change. As we work on improving our delivery room environment and changing ingrained habits, the evidence from this study should help support the use of wiping in place of suctioning. The transition from suctioning to wiping also would be facilitated by having easily accessible towels designated for wiping.

Acknowledgement
The PURLs Surveillance System was supported in part by Grant Number UL1RR024999 from the National Center For Research Resources, a Clinical Translational Science Award to the University of Chicago. The content is solely the responsibility of the authors and does not necessarily represent the official views of the National Center For Research Resources or the National Institutes of Health.

References

 

1. Kelleher J, Bhat R, Salas AA, et al. Oronasopharyngeal suction versus wiping of the mouth and nose at birth: a randomised equivalency trial. Lancet. 2013;382:326-330.

2. Gungor S, Kurt E, Teksoz E, et al. Oronasopharyngeal suction versus no suction in normal and term infants delivered by elective cesarean section: a prospective randomized controlled trial. Gynecol Obstet Invest. 2006;61:9-14.

3. Gungor S, Teksoz E, Ceyhan T, et al. Oronasopharyngeal suction versus no suction in normal, term and vaginally born infants: a prospective randomized controlled trial. Aust N Z J Obstet Gynaecol. 2005;45:453-456.

4. Carrasco M, Martell M, Estol PC. Oronasopharyngeal suction at birth: effects on arterial oxygen saturation. J Pediatr. 1997;130:832-834.

5. Estol PC, Piriz H, Basalo S, et al. Oro-naso-pharyngeal suction at birth: effects on respiratory adaptation of normal term vaginally born infants. J Perinat Med. 1992;20:297-305.

6. Wiswell TE, Gannon CM, Jacob J, et al. Delivery room management of the apparently vigorous meconium-stained neonate: results of the multicenter, international collaborative trial. Pediatrics. 2000;105(1 pt 1):1-7.

7. Vain NE, Szyld EG, Prudent LM, et al. Oropharyngeal and nasopharyngeal suctioning of meconium-stained neonates before delivery of their shoulders: multicentre, randomized controlled trial. Lancet. 2004;364:597-602.

8. Kattwinkel J, Perlman JM, Aziz K, et al. Part 15: neonatal resuscitation: 2010 American Heart Association guidelines for cardiopulmonary resuscitation and emergency cardiovascular care. Circulation. 2010;122(18 suppl 3):S909-S919.

9. Perlman JM, Wyllie J, Kattwinkel J, et al; Neonatal Resuscitation Chapter Collaborators. Neonatal resuscitation: 2010 International Consensus on Cardiopulmonary Resuscitation and Emergency Cardiovascular Care Science with Treatment Recommendations. Pediatrics. 2010;126:e1319-1344.

References

 

1. Kelleher J, Bhat R, Salas AA, et al. Oronasopharyngeal suction versus wiping of the mouth and nose at birth: a randomised equivalency trial. Lancet. 2013;382:326-330.

2. Gungor S, Kurt E, Teksoz E, et al. Oronasopharyngeal suction versus no suction in normal and term infants delivered by elective cesarean section: a prospective randomized controlled trial. Gynecol Obstet Invest. 2006;61:9-14.

3. Gungor S, Teksoz E, Ceyhan T, et al. Oronasopharyngeal suction versus no suction in normal, term and vaginally born infants: a prospective randomized controlled trial. Aust N Z J Obstet Gynaecol. 2005;45:453-456.

4. Carrasco M, Martell M, Estol PC. Oronasopharyngeal suction at birth: effects on arterial oxygen saturation. J Pediatr. 1997;130:832-834.

5. Estol PC, Piriz H, Basalo S, et al. Oro-naso-pharyngeal suction at birth: effects on respiratory adaptation of normal term vaginally born infants. J Perinat Med. 1992;20:297-305.

6. Wiswell TE, Gannon CM, Jacob J, et al. Delivery room management of the apparently vigorous meconium-stained neonate: results of the multicenter, international collaborative trial. Pediatrics. 2000;105(1 pt 1):1-7.

7. Vain NE, Szyld EG, Prudent LM, et al. Oropharyngeal and nasopharyngeal suctioning of meconium-stained neonates before delivery of their shoulders: multicentre, randomized controlled trial. Lancet. 2004;364:597-602.

8. Kattwinkel J, Perlman JM, Aziz K, et al. Part 15: neonatal resuscitation: 2010 American Heart Association guidelines for cardiopulmonary resuscitation and emergency cardiovascular care. Circulation. 2010;122(18 suppl 3):S909-S919.

9. Perlman JM, Wyllie J, Kattwinkel J, et al; Neonatal Resuscitation Chapter Collaborators. Neonatal resuscitation: 2010 International Consensus on Cardiopulmonary Resuscitation and Emergency Cardiovascular Care Science with Treatment Recommendations. Pediatrics. 2010;126:e1319-1344.

Issue
The Journal of Family Practice - 63(8)
Issue
The Journal of Family Practice - 63(8)
Page Number
461-462
Page Number
461-462
Publications
Publications
Topics
Article Type
Display Headline
Suctioning neonates at birth: Time to change our approach
Display Headline
Suctioning neonates at birth: Time to change our approach
Legacy Keywords
respiratory; obstetrics; pediatrics; Iliana Neumann; MD; Anne Mounsey; MD; Niladri Das; MD; bradycardia; apnea; Neonatal Resuscitation Program; oronasopharyngeal suctioning; bulb syringe; meconium
Legacy Keywords
respiratory; obstetrics; pediatrics; Iliana Neumann; MD; Anne Mounsey; MD; Niladri Das; MD; bradycardia; apnea; Neonatal Resuscitation Program; oronasopharyngeal suctioning; bulb syringe; meconium
Sections
PURLs Copyright

Copyright © 2014 Family Physicians Inquiries Network. All rights reserved.

Disallow All Ads
Alternative CME
Use ProPublica
Hide sidebar & use full width
render the right sidebar.
Conference Recap Checkbox
Not Conference Recap
Clinical Edge
Display the Slideshow in this Article
Article PDF Media
Media Files